You are on page 1of 194

Sample Papers (with MS) BY CBSE and DOE

Session 2022-23 and 2023-24


Class XII Mathematics
Assembled By Deepika Bhati
To Get More material for
Maths IX - X Click below

CBSE MATHS
IX-X

To Get More Material for XI-


XII Click below

CBSE MATHS
XI-XII
CBSE SAMPLE QUESTION PAPER
Class:-XII

Session 2023-24

Mathematics (Code-041)

Time: 3 hours Maximum marks: 80

General Instructions:

1. This Question paper contains - five sections A, B, C, D and E. Each section is compulsory. However, there are
internal choices in some questions.

2. Section A has 18 MCQ’s and 02 Assertion-Reason based questions of 1 mark each.

3. Section B has 5 Very Short Answer (VSA)-type questions of 2 marks each.

4. Section C has 6 Short Answer (SA)-type questions of 3 marks each.

5. Section D has 4 Long Answer (LA)-type questions of 5 marks each.

6. Section E has 3 source based/case based/passage based/integrated units of assessment of 4 marks each with

sub-parts.
___________________________________________________________________________________________

Section –A
(Multiple Choice Questions)
Each question carries 1 mark

1, when i  j
Q1. If A   aij  is a square matrix of order 2 such that aij   , then A2 is
 0, when i  j
1 0  1 1 1 1  1 0
(a)   (b)   (c)   (d)  
1 0  22  0 0  22 1 0  22  0 1  22
Q2. If A and B are invertible square matrices of the same order, then which of the following is not correct?
1
 AB
|A| 1
(a) AB
-1
 (b) 
|B| | A | |B|

(c)  AB   B 1 A1 (d)  A  B   B 1  A 1


1 1

Q3. If the area of the triangle with vertices   3 , 0  ,  3, 0  and  0, k  is 9 sq units, then the value/s of k will
be
(a) 9 (b)  3 (c) -9 (d) 6
 kx
 , if x  0
Q4. If f  x    x is continuous at x  0 , then the value of k is
 3, if x  0

(a) −3 (b) 0 (c) 3 (d) any real number
Page 1 of 8
 
   
Q5. The lines r  i  j  k   2i  3 j  6 k and r  2i  j  k   6i  9 j  18 k ; (where  &  are

scalars) are
(a) coincident (b) skew (c) intersecting (d) parallel
3 2
  dy  2   d 2 y 
Q6. The degree of the differential equation 1       2  is
  dx    dx 
3
(a) 4 (b) (c) 2 (d) Not defined
2
Q7. The corner points of the bounded feasible region determined by a system of linear constraints are
 0, 3  ,  1,1  and  3, 0  . Let Z  px  qy , where p , q  0 . The condition on p and q so that the
minimum of Z occurs at  3, 0  and  1,1  is

q
(a) p  2q (b) p  (c) p  3q (d) p  q
2
   
Q8. ABCD is a rhombus whose diagonals intersect at E . Then EA  EB  EC  ED equals to
   
(a) 0 (b) AD (c) 2BD (d) 2AD

 e
cos 2 x
Q9. For any integer n , the value of Sin 3 (2n + 1) x dx is

(a) -1 (b) 0 (c) 1 (d) 2

 0 2x  1 x 
 
Q10. The value of A , if A  1  2 x 0 2 x  , where x    , is
 
  x 2 x 0 

(a)  2 x  1 (c)  2 x  1 (d)  2 x  1


2 3 2
(b) 0

Q11. The feasible region corresponding to the linear constraints of a Linear Programming Problem is given
below.

Which of the following is not a constraint to the given Linear Programming Problem?
(a) x  y  2 (b) x  2 y  10 (c) x  y  1 (d) x  y  1

Page 2 of 8
   
Q12. If a  4i  6 j and b  3 j  4k , then the vector form of the component of a along b is

(a)
5

18  
3i  4k  (b)
18 
25

3 j  4k  (c)
5

18  
3i  4k  (d)
25

18  
4i  6 j 
Q13. Given that A is a square matrix of order 3 and A  2, then adj  2 A  is equal to
(a) 26 (b)  4 (c) 28 (d) 28
1 1 1
Q14. A problem in Mathematics is given to three students whose chances of solving it are , ,
2 3 4
respectively. If the events of their solving the problem are independent then the probability that the
problem will be solved, is
1 1 1 3
(a) (b) (c) (d)
4 3 2 4
Q15. The general solution of the differential equation ydx – xdy  0;  Given x , y  0  , is of the form

(a) xy  c (b) x  c y 2 (c) y  c x (d) y  cx 2 ;


(Where ' c ' is an arbitrary positive constant of integration)

Q16. The value of  for which two vectors 2 i  j  2 k and 3i   j  k are perpendicular is
(a) 2 (b) 4 (c) 6 (d) 8
Q17. The set of all points where the function f  x   x  x is differentiable, is

(a)  0,   (b)   , 0  (c)   , 0    0,   (d)   ,  

1 1 1
Q18. If the direction cosines of a line are  , ,  , then
c c c

(a) 0  c  1 (b) c  2 (c) c   2 (d) c   3

ASSERTION-REASON BASED QUESTIONS

In the following questions, a statement of Assertion (A) is followed by a statement of Reason (R).
Choose the correct answer out of the following choices.

(a) Both (A) and (R) are true and (R) is the correct explanation of (A).
(b) Both (A) and (R) are true but (R) is not the correct explanation of (A).
(c) (A) is true but (R) is false.
(d) (A) is false but (R) is true.
d
Q19. Let f  x  be a polynomial function of degree 6 such that
dx
 f  x     x  13  x  3 2 , then
ASSERTION (A): f  x  has a minimum at x  1.

d
REASON (R): When
dx
 f  x    0,  x   a  h,a  and dxd  f  x    0,  x   a, a  h ; where
' h ' is an infinitesimally small positive quantity, then f  x  has a minimum at x  a,
provided f  x  is continuous at x  a .

Page 3 of 8
Q20. ASSERTION (A): The relation f : 1,2, 3,4   x , y, z , p defined by f   1, x  ,  2, y  ,  3, z  is a
bijective function.
REASON (R): The function f : 1,2, 3   x , y, z , p such that f   1, x  ,  2, y  ,  3, z  is one-one.

Section –B

[This section comprises of very short answer type questions (VSA) of 2 marks each]

  33 
Q21. Find the value of sin 1  cos  .
  5 
OR
Find the domain of sin1 x 2  4 .  
Q22. Find the interval/s in which the function f :    defined by f  x   x e x , is increasing.

1
Q23. If f  x   2
; x   , then find the maximum value of f  x  .
4x  2x  1
OR
Find the maximum profit that a company can make, if the profit function is given by
P  x   72  42 x  x 2 , where x is the number of units and P is the profit in rupees.

1  2 x 
Q24. Evaluate :  1
log 
 2 x
 dx .

Q25. Check whether the function f :    defined by f  x   x 3  x, has any critical point/s or not ?
If yes, then find the point/s.

Section – C
[This section comprises of short answer type questions (SA) of 3 marks each]

2x2  3
Q26. Find :  x x
2 2
9 
dx ; x  0.

Q27. The random variable X has a probability distribution P  X  of the following form, where ' k ' is some
real number:

 k , if x  0
 2k , if x  1

P X   
 3k , if x  2
0, otherwise

(i) Determine the value of k .

(ii) Find P  X  2  .

Page 4 of 8
(iii) Find P  X  2 .

x
Q28. Find :  dx; x   0,1 .
1  x3
OR

Evaluate:  4
log  1  tan x  dx .
0

x
 x 
Q29. Solve the differential equation: ye y dx   xe y  y 2  dy ,
   y  0 .
 
OR

dy  
Solve the differential equation:  cos 2 x   y  tan x ;  0  x  2 .
dx  

Q30. Solve the following Linear Programming Problem graphically:


Minimize: z  x  2 y ,
subject to the constraints: x  2 y  100, 2 x  y  0, 2 x  y  200, x , y  0.
OR
Solve the following Linear Programming Problem graphically:
Maximize: z   x  2 y ,
subject to the constraints: x  3, x  y  5, x  2 y  6, y  0.
y 2
d2 y  a 
Q31. If  a  bx  e x  x then prove that x   .
dx 2  a  bx 

Section –D
[This section comprises of long answer type questions (LA) of 5 marks each]

Q32. Make a rough sketch of the region  x, y  : 0  y  x 2



 1, 0  y  x  1, 0  x  2 and find the

area of the region, using the method of integration.


Q33. Let  be the set of all natural numbers and R be a relation on    defined by

 a , b  R  c, d   ad  bc for all  a , b  ,  c , d      . Show that R is an equivalence relation on

   . Also, find the equivalence class of  2,6  , i.e.,  2,6   .

OR
x
Show that the function f :    x   :  1  x  1 defined by f  x   , x   is one-one and
1 x
onto function.
Q34. Using the matrix method, solve the following system of linear equations :

Page 5 of 8
2 3 10 4 6 5 6 9 20
   4,    1,    2.
x y z x y z x y z

Q35. Find the coordinates of the image of the point  1, 6 , 3  with respect to the line

   
r  j  2k   i  2 j  3k ; where '  ' is a scalar. Also, find the distance of the image from the
y  axis.
OR

 
An aeroplane is flying along the line r   i  j  k ; where '  ' is a scalar and another aeroplane is flying


 
along the line r  i  j   2 j  k ; where '  ' is a scalar. At what points on the lines should they reach, so

that the distance between them is the shortest? Find the shortest possible distance between them.

Section –E
[This section comprises of 3 case- study/passage based questions of 4 marks each with sub parts.
The first two case study questions have three sub parts (i), (ii), (iii) of marks 1,1,2 respectively.
The third case study question has two sub parts of 2 marks each.)
Q36. Read the following passage and answer the questions given below:
In an Office three employees Jayant, Sonia and Oliver process incoming copies of a certain form. Jayant
processes 50% of the forms, Sonia processes 20% and Oliver the remaining 30% of the forms. Jayant
has an error rate of 0.06 , Sonia has an error rate of 0.04 and Oliver has an error rate of 0.03 .
Based on the above information, answer the following questions.

(i) Find the probability that Sonia processed the form and committed an error.
(ii) Find the total probability of committing an error in processing the form.
(iii) The manager of the Company wants to do a quality check. During inspection, he selects a form at
random from the days output of processed form. If the form selected at random has an error, find the
probability that the form is not processed by Jayant.
OR

Page 6 of 8
(iii) Let E be the event of committing an error in processing the form and let E1 , E2 and E 3 be the
3
events that Jayant, Sonia and Oliver processed the form. Find the value of  P  E E .
i 1
i

Q37. Read the following passage and answer the questions given below:
Teams A , B , C went for playing a tug of war game. Teams A, B , C have attached a rope to a metal ring
and is trying to pull the ring into their own area.

Team A pulls with force F1  6iˆ  0 ˆj kN ,

Team B pulls with force F2  4iˆ  4 ˆj kN ,

Team C pulls with force F3   3 iˆ  3 ˆj kN ,

(i) What is the magnitude of the force of Team A ?


(ii) Which team will win the game?
(iii) Find the magnitude of the resultant force exerted by the teams.
OR
(iii) In what direction is the ring getting pulled?

Q38. Read the following passage and answer the questions given below:

The relation between the height of the plant  ' y ' in cm  with respect to its exposure to the sunlight

1 2
is governed by the following equation y  4 x  x , where ' x ' is the number of days exposed to the
2

sunlight, for x  3.

(i) Find the rate of growth of the plant with respect to the number of days exposed to the sunlight.

Page 7 of 8
(ii) Does the rate of growth of the plant increase or decrease in the first three days?
What will be the height of the plant after 2 days?
*************************************************************************************

Page 8 of 8
SAMPLE QUESTION PAPER
MARKING SCHEME
CLASS XII
MATHEMATICS (CODE-041)
SECTION: A (Solution of MCQs of 1 Mark each)
Q no. ANS HINTS/SOLUTION

1 (d)  0 1  2  1 0
A   , A   0 1 .
1 0  
2 (d)
 A  B
1
 B1  A1 .
3 (b) 3 0 1
1
Area  3 0 1 , given that the area  9 sq unit .
2
0 k 1

3 0 1
1
  9  3 0 1 ; expanding along C 2 , we get  k  3.
2
0 k 1

4 (a) Since, f is continuous at x  0 ,

therefore, L. H . L  R. H . L  f  0   a finite quantity .

lim f  x   lim f  x   f  0
x0 x0

 kx
 lim  lim 3  3  k  3.
x 0 x x 0

5 (d) Vectors 2i  3 j  6k &6i  9 j  18k are parallel and the fixed point i  j  k on the

 
line r  i  j  k   2i  3 j  6k does not satisfy the other line

 
r  2i  j  k   6i  9 j  18k ; where  &  are scalars.

6 (c)   dy  2   d 2 y 
3 2

The degree of the differential equation 1       2  is 2


  dx    dx 

7 (b) Z  px  qy     i 

At  3,0  , Z  3 p     ii  and at  1,1 , Z  p  q      iii 

From  ii  &  iii  , 3 p  p  q  2 p  q .

Page 1 of 19
 
8 (a) Given, ABCD is a rhombus whose diagonals bisect each other. EA  EC and
 
EB  ED but since they are opposite to each other so they are of opposite signs
   
 EA   EC and EB   ED .

     


 EA  EC  O .....  i  and EB  ED  O ....  ii 
    
Adding (i) and (ii), we get EA  EB  EC  ED  O .

9 (b) f  x   e cos x sin 3  2n  1 x


2

f   x  e sin3  2n  1  x 
cos2   x 

2
f (  x )   e cos x sin 3 (2n  1) x
 f ( x )   f ( x )

So,  ecos x sin3 (2n  1) x dx  0
2



10 (b) Matrix A is a skew symmetric matrix of odd order.  A  0.


11 (c) We observe,  0,0  does not satisfy the inequality x  y  1

So, the half plane represented by the above inequality will not contain origin
therefore, it will not contain the shaded feasible region.
12 (b) 
   a .b   18
Vector component of a along b   2 b 

 b 
 25
3 j  4k .  
 
13 (d) adj  2 A    2 A  23 A    26 A  26   2   28 .
2 2 2 2

14 (d) Method 1:
1 1
Let A, B , C be the respective events of solving the problem. Then, P  A  , P  B  
2 3
1
and P  C   . Here, A, B , C are independent events.
4
Problem is solved if at least one of them solves the problem.
Required probability is  P  A  B  C   1  P A P B P C      
Page 2 of 19
 1  1  1 1 3
 1   1    1   1    1   .
 2  3  4 4 4
Method 2:
The problem will be solved if one or more of them can solve the problem. The probability is
        
P ABC  P ABC  P ABC  P ABC  P ABC  P ABC  P  ABC    
1 2 3 1 1 3 1 2 1 1 1 3 1 2 1 1 1 1 1 1 1 3
 . .  . .  . .  . .  . .  . .  . .  .
2 3 4 2 3 4 2 3 4 2 3 4 2 3 4 2 3 4 2 3 4 4
Method 3:
Let us think quantitively. Let us assume that there are 100 questions given to A. A
1
solves  100  50 questions then remaining 50 questions is given to B and B solves
2
1 2
50   16.67 questions . Remaining 50  questions is given to C and C solves
3 3
2 1
50    8.33 questions.
3 4
Therefore, number of questions solved is 50  16.67  8.33  75 .
75 3
So, required probability is  .
100 4
15 (c) Method 1:
ydx  xdy  x 1
ydx  xdy  0  2
 0  d    0  x  y  y  cx.
y  y c
Method 2:
dy dx dy dx
ydx  xdy  0  ydx  xdy 
y

x
; on integrating  y
  x
log e y  log e x  log e c
since x , y , c  0 , we write log e y  log e x  log e c  y  cx .
16 (d) Dot product of two mutually perpendicular vectors is zero.
 2  3   1   2  1  0    8.
17 (c) Method 1:
2 x, x  0
f  x  x  x  
 0 ,x  0

There is a sharp corner at x  0 , so f  x  is not differentiable at x  0 .

Method 2:
Page 3 of 19
Lf '  0   0 & Rf '  0   2 ; so, the function is not differentiable at x  0

For x  0, f  x   2 x (linear function) & when x  0, f  x   0 (constant function)

Hence f  x  is differentiable when x    ,0    0,   .

18 (d)  1  1  1
2 2
1
2 2

We know, l  m  n  1           1  3    1  c   3 .
2 2 2

c c c c


19 (a) d
dx
 f  x     x  1  x  3 
3 2

Assertion : f  x  has a minimum at x  1 is true as

d d
dx
 f  x    0,  x   1  h,1 and
dx
 f  x    0,  x  1,1  h ; where,
' h ' is an infinitesimally small positive quantity , which is in accordance with
the Reason statement.
20 (d) Assertion is false. As element 4 has no image under f , so relation f is not a function.
Reason is true. The given function f : 1, 2, 3   x , y , z , p is one – one, as for each

a  1, 2,3 , there is different image in  x , y, z , p under f .

Section –B
[This section comprises of solution of very short answer type questions (VSA) of 2 marks each]

21   33    3   3    3  1
sin1  cos     sin cos  6 
1 1 1
  sin cos    sin sin   
  5   5   5  2 5 
 3 
   . 1
2 5 10
21 OR  
1  x 2  4  1  3  x 2  5  3  x  5 1

 x    5,  3    3, 5  . So, required domain is   5,  3    3, 5  . 1

22 f  x   x e x  f '  x   e x  x  1 1

1
When x   1,   ,  x  1  0 & e x  0  f '  x   0  f  x  increases in this interval.

or, we can write f  x   x e x  f '  x   e x  x  1 1


2
For f  x  to be increasing, we have f '  x   e x  x  1  0  x  1 as e x  0,  x   1

Hence, the required interval where f  x  increases is  1,   . 1


2
23 1
Method 1 : f  x  
4x  2x  1 ,
2

Page 4 of 19
 2 1 1 3  1 3 3
2
1
Let g  x   4 x  2 x  1  4  x  2 x     4  x     1
2

4 16  4 4 4 4 2
 
1
4
maximum value of f  x   . 2
3
1
Method 2 : f  x   , let g  x   4 x 2  2 x  1
4x  2x  1
2

d 1 d2

dx
 g  x    g '  x   8 x  2 and g '  x   0 at x   also 2  g  x    g"  x   8  0
4 dx
1

1 1 1
 g  x  is minimum when x   so , f  x  is maximum at x  
4 4 2
 1 1 4
maximum value of f  x   f     2
 . 1
 4  1  1 3
4     2    1 2
 4  4
1
Method 3 : f  x  
4x  2x  1
2

  8 x  2 1
On differentiating w.r.t x ,we get f '  x   ....  i 
4 x  2
2
2
 2x  1
1 1
For maxima or minima , we put f '  x   0  8 x  2  0  x   .
4 2
Again, differentiating equation (i) w.r.t. x ,we get


 4 x2  2 x  1   8   8 x  2 2   4 x 
 2 x  1  8 x  2  
2
2

f " x      1
 4 x  2 x  1
4
 2
 2
1  1
At x   , f "     0
4  4
1
f  x  is maximum at x   .
4
1
 maximum value of f  x  is f    
1 1 4
2
 . 2
 4  1  1 3
4    2    1
 4  4
1
Method 4: f  x  
4x  2x  1
2

  8 x  2 1
On differentiating w.r.t x ,we get f '  x   ....  i 
4 x 
2
2
 2x  1 2

1 1
For maxima or minima , we put f '  x   0  8 x  2  0  x   .
4 2
 1 1
When x    h  ,   , where ' h ' is infinitesimally small positive quantity.
 4 4 

4 x  1  8 x  2  8 x  2  0    8 x  2   0 and  4 x 2  2 x  1  0  f '  x   0
2

Page 5 of 19
 1 1 
and when x    ,   h  , 4 x  1  8 x  2  8 x  2  0    8 x  2   0
 4 4  1
1 2
   0  f '  x   0 . This shows that x   is the point of local maxima.
2
and 4 x 2  2 x  1
4
1
maximum value of f  x  is f    
1 1 4
2
 . 2
 4  1  1 3
4    2    1
 4   4
23 OR For maxima and minima, P '  x   0  42  2 x  0 1
2
 x  21 and P "  x   2  0
1
So, P  x  is maximum at x  21 . 2
The maximum value of P  x   72   42  21    21   513 i.e., the maximum profit is 513.
2
1

24 2 x
Let f  x   log  
2 x

2 x 2 x 
We have, f   x   log     log  2  x    f  x  1
2 x   
1 2 x
So, f  x  is an odd function.   log   dx  0. 1
1
2 x
25 f  x   x 3  x , for all x   .
1
d 1
dx
 f  x    f '  x   3 x 2  1; for all x   , x 2  0  f '  x   0 2
1
Hence, no critical point exists.
2

Section –C
[This section comprises of solution short answer type questions (SA) of 3 marks each]

26 2x2  3 1
We have, . Now , let x 2  t

x2 x2  9  2

2t  3 A B 1 5
So,   , we get A  & B  1
t  t  9 t t  9 3 3

2x2  3 1 dx 5 dx 1
 x x2 2
9 
dx  

3 x2 3 x 2
9 2

1 5  x
  tan 1    c , where ' c ' is an arbitrary constant of integration. 1
3x 9 3
27 1 1
We have, (i)  P  X   1  k  2 k  3k  1  k  6 .
i

1
Page 6 of 19
1 1
(ii) P  X  2   P  X  0   P  X  1  k  2k  3k  3   .
6 2 1
(iii) P  X  2   0.

28 3
3 12 1
Let x 2  t  dt  x dx
2 2

x 2 dt 1
 1 x 3
dx  
3 1 t2 2

2 1 1
 sin  t   c
3

2  3 1
 sin1  x 2   c , where ' c ' is an arbitrary constant of integration.
3  
28 OR 
Let I   4 log e  1  tan x  dx ------(i)
0


   a a 1
 I   4 loge  1  tan   x   dx , Using,  f ( x )dx   f (a  x )dx
0
 4  0 0

  
 1  tan x   2 
 I   4 log e  1   dx   4
log e   dx   4
log e 2 dx  I ( Using ------(i) 1
0
 1  tan x  0
 1  tan x  0

1
 
 2I  log e 2  I  log e 2.
4 8

29  xy
x
 x x
 ydx  xdy 
Method 1: ye dx   xe  y  dy  e  ydx  xdy   y dy  e y 
y 2 y 2
  dy 1
   y2 
 
x
 x 1
 e d    dy
y

 y
x x
 x
  e d     dy  e y  y  c , where ' c ' is an arbitrary constant of integration.
y

 y 1
x

dx xe y  y 2
Method 2: We have ,  x
dy
y .e y

dx x y 1
   x ……………. (i) 2
dy y
ey
1
dx dv
Let x  vy   v  y. ; 2
dy dy

Page 7 of 19
dv y 1
So equation (i) becomes v  y v v
dy e 2
dv y 1
y  v
dy e 2
1
 e v dv  dy
2

e dv   dy  e v  y  c  e x / y  y  c 1
v
On integrating we get,
2
where ' c ' is an arbitrary constant of integration.

29 OR The given Differential equation is

dy
 cos x  dx
2
 y  tan x

Dividing both the sides by cos 2 x , we get

dy y tan x
 
dx cos x cos 2 x
2

dy
dx
  
 y sec2 x  tan x sec2 x ........  i  1
2
dy
Comparing with  Py  Q
dx

P  sec 2 x , Q  tan x .sec 2 x


1
The Integrating factor is, IF  e   e
P dx sec2 x dx
 e tan x 2
On multiplying the equation  i  by e tan x , we get

d
dx
     
y .e tan x  e tan x tan x sec2 x  d y .e tan x  e tan x tan x sec2 x dx   1

On integrating we get , y .e tan x   t .e t dt  c1 ; where, t  tan x so that dt  sec2 x dx

 te t  e t  c   tan x  e tan x  e tan x  c

 
 y  tan x  1  c . e  tan x , where ' c1 '& ' c ' are arbitrary constants of integration.
1
30 The feasible region determined by the
constraints, x  2 y  100, 2 x  y  0, 2 x  y  200, x , y  0 , is given below.

Page 8 of 19
1
1
2

A  0, 50  , B  20, 40  , C  50, 100  and D  0, 200  are the corner points of the feasible
region.
1
The values of Z at these corner points are given below.

Corner point Corresponding value of


Z  x  2y

A  0, 50  100 Minimum

B  20, 40  100 Minimum


1
C  50, 100  250
2

30 OR D  0, 200  400

The minimum value of Z is 100 at all the points on the line segment joining the points  0,50 

and  20,40  .

The feasible region determined by the constraints, x  3, x  y  5, x  2 y  6, y  0.


is given below.

1
1
2

Page 9 of 19
Here, it can 1
be seen that
the

feasible region is unbounded.

The values of Z at corner points A  3, 2  , B  4, 1 and C  6, 0  are given below. 1


Corner point Corresponding value of Z   x  2 y 2

A  3, 2  1 ( may or may not be the maximum value)

B  4, 1  -2

C  6, 0  -6

Since the feasible region is unbounded, Z  1 may or may not be the maximum value.

Now, we draw the graph of the inequality, – x  2 y  1 , and we check whether the resulting
open half-plane has any point/s, in common with the feasible region or not.

Here, the resulting open half plane has points in common with the feasible region.

Hence, Z  1 is not the maximum value. We conclude, Z has no maximum value.

31 y  x  1
 log e    log e x  log e  a  bx 
x  a  bx  2

On differentiating with respect to x , we get


dy
x y
dx 1 1 d 1 b
    a  bx    1
x 2
x a  bx dx x a  bx
dy 1 b  ax 1
x  y  x2    
dx  x a  bx  a  bx 2
On differentiating again with respect to x , we get

d 2 y dy dy  a  bx  a  ax  b  1
x   
dx 2 dx dx  a  bx 
2
2

Page 10 of 19
2
d2 y  a 
 x 2   .
dx  a  bx  1
2

Section –D
[This section comprises of solution of long answer type questions (LA) of 5 marks each]

32

To find the point of intersections of the curve y  x 2  1 and the line y  x  1 ,

we write x 2  1  x  1  x  x  1  0  x  0,1.

So, the point of intersections P  0,1  and Q  1,2  . 1

Area of the shaded region OPQRTSO = (Area of the region OSQPO + Area of the region
STRQS )
 
1 2
  x 2  1 dx    x  1 dx 1
0 1
1 2
 x3   x2  1
   x    x
3 0  2 1 2
 1     1 
   1   0   2  2     1  1
 3     2  2
23 23 1
 Hence the required area is sq units.
6 6
33 Let  a , b  be an arbitrary element of    . Then,  a , b      and a, b 

We have, ab  ba ; (As a, b   and multiplication is commutative on  )

  a , b  R  a , b  , according to the definition of the relation R on   

Thus  a , b  R  a , b  ,   a , b      .

So, R is reflexive relation on    . 1


Let  a , b  ,  c , d  be arbitrary elements of    such that  a , b  R  c , d  .

Page 11 of 19
Then,  a , b  R  c , d   ad  bc  bc  ad ; (changing LHS and RHS)

 cb  da; (As a, b, c, d   and multiplication is commutative on  )

  c , d  R  a , b  ; according to the definition of the relation R on   

Thus  a , b  R  c , d    c , d  R  a , b 

So, R is symmetric relation on    . 1


Let  a , b  ,  c , d  ,  e , f  be arbitrary elements of    such that

 a , b  R  c , d  and  c , d  R  e , f  .
 a , b  R  c , d   ad  bc 
Then    ad  cf    bc  de   af  be
 c , d  R  e , f   cf  de 
  a, b  R  e, f ; (according to the definition of the relation R on    )

Thus  a , b  R  c , d  and  c , d  R  e , f    a, b  R  e, f 
So, R is transitive relation on    .
1
As the relation R is reflexive, symmetric and transitive so, it is equivalence relation on    .
1
 2, 6     x , y      :  x , y  R  2, 6  2
1
  x , y      : 3 x  y 2

  x ,3 x  : x     1, 3  ,  2,6 ,  3,9  ,.........


1
33 OR
 x
 1  x , if x  0
We have, f  x   
 x , if x  0
 1  x
Now, we consider the following cases
x
Case 1: when x  0 , we have f  x  
1 x
Injectivity: let x, y   0 such that f  x   f  y  , then

x y
   x  xy  y  xy  x  y
1 x 1 y
So, f is injective function. 1
x 1
Surjectivity : when x  0 , we have f  x    0 and f  x   1   1,as x  0
1 x 1 x
y
y 1 y

Let y   0,1  , thus for each y  0,1 there exists x 
1 y
 0 such that f  x  
y
 y.
1
1 y

1
Page 12 of 19
So, f is onto function on  0,   to  0,1 .
x
Case 2: when x  0 , we have f  x  
1 x
Injectivity: Let x, y   i.e., x , y  0 , such that f  x  f  y  , then

x y
   x  xy  y  xy  x  y
1 x 1 y
So, f is injective function.
1
x x 1
Surjectivity : x  0 , we have f  x    0 also, f  x    1   1
1 x 1 x 1 x
1  f  x   0 .
y
Let y    1, 0  be an arbitrary real number and there exists x   0 such that,
1 y
y
 y  1 y
f  x  f    y.
 1 y  1 y
1 y
y
So, for y    1, 0  , there exists x   0 such that f  x   y .
1 y 1
Hence, f is onto function on   , 0  to   1, 0  .
Case 3:
x y
(Injectivity): Let x  0 & y  0 such that f  x   f  y   
1 x 1 y
 x  xy  y  xy  x  y  2 xy , here LHS  0 but RHS  0 , which is inadmissible.
Hence , f  x   f  y  when x  y.
1
Hence f is one-one and onto function.
34 The given system of equations can be written in the form AX  B,

 2 3 10  1 / x   4
     
Where, A   4 6 5  , X  1 / y  and B  1 
 6 9 20   1 / z   2

2 3 10 1
Now, A  4 6 5  2  120  45  3  80  30   10  36  36  2
6 9 20

 2  75   3   110   10  72   150  330  720  1200  0  A1 exists. 1


2
T
 75 110 72   75 150 75 
 adj A  150 100 0   110 100 30 
  
1
1
 75 30 24   72 0 24  2

Page 13 of 19
 75 150 75 
1 1 
Hence, A 
1
 adjA  110 100 30  1
A 1200 
 72 0 24 2

1
 
x  75 150 75   4 
1 1 
1
As, AX  B  X  A B     110 100 30  1  1
y 1200  2
   72 0 24   2
1
z
 
1 1
   
 300  150  150   x  600   2 
1  1 1  1 1
  440  100  60       400     2
1200 y 1200  3
 288  0  48     240   
1 1
z  5 
 

1 1 1 1 1 1
Thus,  ,  ,  Hence, x  2, y  3, z  5 . 1
x 2 y 3 z 5

35 Let P 1,6,3  be the given point, and let ' L ' be the foot of the perpendicular from ' P ' to the

given line AB (as shown in the figure below). The coordinates of a general point on the
given line are given by

x 0 y 1 z  2
    ;  is a scalar, i.e., x   , y  2  1 and z  3  2
1 2 3
Let the coordinates of L be   , 2  1,3  2  .
1
So, direction ratios of PL are   1, 2  1  6 and 3  2  3, i .e .   1, 2  5 and 3  1. 2
1
Direction ratios of the given line are 1, 2 and 3, which is perpendicular to PL . 2

Therefore,    1 1   2  5  2   3  1  3  0  14  14  0    1
1
So, coordinates of L are 1,3,5  .

Page 14 of 19
Let Q  x1 , y1 , z1  be the image of P 1,6,3  in the given line. Then, L is the mid-point of 1
PQ .

Therefore,
 x1  1  1,  y1  6   3 and  z1  3  5  x  1, y1  0 and z1  7
1
2 2 2
Hence, the image of P  1,6,3  in the given line is 1,0,7  .
1
Now, the distance of the point  1,0,7 from the y  axis is 12  72  50 units.

1
35 OR Method 1:

Given that equation of lines are


 
   
r   i  j  k ..............  i  and r  i  j   2 j  k ..............  ii 

The given lines are non-parallel lines as vectors i  j  k and 2 j  k are not parallel. There is a

unique line segment PQ ( P lying on line  i  and Q on the other line  ii  ), which is at right

angles to both the lines. PQ is the shortest distance between the lines. Hence, the shortest possible
distance between the aeroplanes  PQ .

 
Let the position vector of the point P lying on the line r   i  j  k where '  ' is a scalar, is
1
 
 i  j  k , for some  and the position vector of the point Q lying on the line 2

 
r  i  j   2 j  k ; where '  ' is a scalar, is i   1  2  j     k , for some  .
   1
Now, the vector PQ  OQ  OP   1    i   1  2    j       k ; (where ' O ' is the 2

origin), is perpendicular to both the lines, so the vector PQ is perpendicular to both the vectors
i  j  k and 2 j  k .

  1    .1   1  2    .  1       .1  0 &

  1    .0   1  2    .  2        .1  0 1
 2  3  3  0 & 2  5   3  0 2
1
2
Page 15 of 19
2
On solving the above equations , we get   and   0
3
1
So, the position vector of the points, at which they should be so that the distance between them is

the shortest, are


3

2   

i  j  k and i  j . 1

   1 1 2  2 2


1  1  2
2
2
PQ  OQ  OP  i  j  k and PQ             1
3 3 3  3  3  3 3

2
The shortest distance  units.
3
Method 2:

x y z
The equation of two given straight lines in the Cartesian form are   ........ i  and
1 1 1
x1 y1 z
  .........  ii 
0 2 1
The lines are not parallel as direction ratios are not proportional. Let P be a point on straight line 1
i and Q be a point on straight line  ii  such that line PQ is perpendicular to both of the lines. 2

Let the coordinates of P be   ,   ,   and that of Q be  1, 2  1,   ; where '  ' and '  ' are
1
scalars. 2
Then the direction ratios of the line PQ are    1,    2  1,    

Since PQ is perpendicular to straight line  i  , we have,


1
(  1).1  (    2  1).( 1)  (   ).1  0 2

 3  3  2......  iii 
1
Since , PQ is perpendicular to straight line  ii  , we have 2
0.    1      2  1 .( 2)       .1  0  3  5  2........  iv  1

2
Solving  iii  and  iv  , we get   0,   1
3
 2 2 2
Therfore , the Coordinates of P are  ,  ,  and that of Q are  1, 1, 0
 3 3 3 1

Page 16 of 19
2 2 2
 2  2  2 2
So, the required shortest distance is  1  3    1  3    0  3   3
units.
     

Section –E

[This section comprises solution of 3 case- study/passage based questions of 4 marks each with two sub
parts. Solution of the first two case study questions have three sub parts (i),(ii),(iii) of marks 1,1,2
respectively. Solution of the third case study question has two sub parts of 2 marks each.)

36 Let E1 , E2 , E3 be the events that Jayant, Sonia and Oliver processed the form, which are clearly

pairwise mutually exclusive and exhaustive set of events.

50 5 20 1 30 3
Then P  E1    , P  E2    and P  E3    .
100 10 100 5 100 10

Also, let E be the event of committing an error.

We have, P  E | E1   0.06 , P  E | E 2   0.04 , P  E | E 3   0.03.

(i) The probability that Sonia processed the form and committed an error is given by

1
P  E  E2   P  E2  . P  E | E2    0.04  0.008. 1
5

(ii) The total probability of committing an error in processing the form is given by

P  E   P  E1  . P  E | E1   P  E2  . P  E | E2   P  E3  . P  E | E3 

50 20 30
P E   0.06   0.04   0.03  0.047. 1
100 100 100

(iii) The probability that the form is processed by Jayant given that form has an error is given by
P  E | E1   P  E1 
P  E1 | E  
P  E | E1  . P  E1   P  E | E2  . P  E2   P  E | E3  . P  E3 

50
0.06  1
100 30
 
50 20 30 47
0.06   0.04   0.03 
100 100 100
Therefore, the required probability that the form is not processed by Jayant given that form has an
1
 
error  P E1 | E  1  P  E1 | E   1 
30 17
 .
47 47
1
3
(iii) OR  PE E  P E
i 1 | E   P  E 2 | E   P  E3 | E   1
i 1 1
Since, sum of the posterior probabilities is 1.

Page 17 of 19
3
( We have ,  P  E i E   P  E1 | E   P  E 2 | E   P  E 3 | E 
i 1

P  E  E1   P  E  E2   P  E  E3 

P E
P   E  E1    E  E2    E  E3  
 as Ei & E j ; i  j , are mutually exclusive events
P E
P  E  ( E1  E2  E3  PE  S PE
    1; ' S ' being the sample space )
PE PE PE

37 We have ,
  
 4   3    3   18  3 2 kN .
2 2 2
F1  62  02  6 kN , F2   42  32  4 2 kN , F3 
1
(i) Magnitude of force of Team A  6 kN .
   1
(ii) Sin ce a  c  3(i  j )and b  4 (i  j )
  
So, b and a  c are unlike vectors having same intial point 1
  
and b  4 2 & a  c  3 2
      1
Thus F 2  F 1  F 3 also F 2 and F 1  F 3 are unlike
Hence B will win the game
   
(iii) F  F1  F2  F3  6iˆ  0 ˆj  4iˆ  4 ˆj  3iˆ  3 ˆj   iˆ  ˆj
1

 1    1 
2 2
F   2 kN .
1
OR

F   iˆ  ˆj

1  3
     tan 1       ; where' ' is the angle made by the resultant force with the
1 4 4

 ve direction of the x  axis.

38 1
y  4x  x2
2
(i) The rate of growth of the plant with respect to the number of days exposed to sunlight
dy 2
is given by  4  x.
dx
dy
(ii) Let rate of growth be represented by the function g  x   .
dx

Page 18 of 19
d  dy 
Now, g '  x    1  0
dx  dx 

 g  x  decreases. 1

So the rate of growth of the plant decreases for the first three days. 1
1
 2   6 cm .
2
Height of the plant after 2 days is y  4  2 
2

Page 19 of 19
Additional Practice Question Paper (2023-24)
CLASS-XII
MATHEMATICS (041)
TIME: 3 Hours MM.80

General Instructions:
 This Question paper contains - five sections A, B, C, D and E. Each section is compulsory.
However, there are internal choices in some questions.
 Section A has 18 MCQ’s and 02 Assertion-Reason based questions of 1 mark each.
 Section B has 5 Very Short Answer (VSA)-type questions of 2 marks each.
 Section C has 6 Short Answer (SA)-type questions of 3 marks each.
 Section D has 4 Long Answer (LA)-type questions of 5 marks each.
 Section E has 3 source based/case based/passage based/integrated units of assessment of 4
marks each with sub-parts.
Section –A
(Multiple Choice Questions)
Each question carries 1 mark
Q1. The value of x – y + z from the following equation is
 x  y  z  9 
x  z   5 
   
 y  z  7 
(a) -3 (b) -1

(c) 1 (d) 3

Q2. 5 0 0 
If A be a 3  3 square matrix such that A  adj A   0 5 0  then the value of Adj A is
0 0 5 

(a) 5 (b) 25

(c) 125 (d) 625

Q3. 
If A and B are symmetric matrices of same order, then ABT  2 BAT is a 
(a) Skew symmetric matrix (b)Symmetric matrix

(c) Neither Symmetric matrix nor Skew symmetric matrix (d) Null matrix

Q4. In the interval (1,2) the function f  x   2 x  1  3 x  2 is

(a) Strictly Increasing (b) Strictly Decreasing

(c) Neither Increasing nor Decreasing (d) Remains constant

Q5. If the set A contains 5 elements and the set B contains 6 elements, then the number of both
one-one and onto mapping from A to B is

(a) 720 (b) 120

(c) 30 (d) 0

Q6. 𝑑3𝑦 𝑑𝑦 5
The sum of order & degree of the differential equation = (1 + ) is
𝑑𝑥 3 𝑑𝑥

(a) 3 (b) 4

(c) 5 (d) 8
Q7. The solution set of the inequation 3x  2 y  3 is

(a) half plane containing the origin (b) half plane not containing the origin

(c) the point being on the line 3x  2 y  3 (d) None of these

Q8. The two vectors ĵ  kˆ and 3 iˆ  ˆj  4kˆ represents the two sides AB and AC, respectively of
 ABC . The length of the median through A is
34 48
(a) (b)
2 2
(c) 18 (d) 52

Q9.  /2

 x
3
The value of sin 4 x dx is
 /2


(a ) 0 (b)
2
2
(c )  (d )
4
Q10. 2 3 
If A    be such that A1  kA , then k is equal to
 5 2 

(a) 19 (b) 1/19

(c) -1/19 (d) - 19

Q11. The corner points of the feasible region for the Linear Programming Problem are (0, 2),
(3, 0), (6, 0), (6, 8) and (0, 5). Let the objective function is Z  4x  6 y then the minimum
value of the objective function occurs at

(a) (0, 2) only (b) (3, 0) only

(c) The mid-point on the line segment joining the points (0,2) and (3,0)

(d) Any point on the line segment joining the points (0,2) and (3,0)

Q12. If the projection of  iˆ  ˆj  4kˆ on 2 iˆ  6 ˆj  3kˆ is 4 units, then the value of  is equal to

(a) -9 (b) -5

(c) 5 (d) 9
Q13.  2 1  4 5
then  AB  is equal to
1
If A    and B 1   
 5 3  3 4 

 15 19   11 14 
(a)  26 33  (b)  29 37 
   
 37 14   37  14 
(c)  29 11 (d)  29 11 
   

Q14. In a hockey match, both teams A and B scored same number of goals up to the end of this
game, so to decide the winner, the referee asked both the captains to throw a die alternately
and decided that the team, whose captain gets a six first, will be declared the winner. If the
captain of team A was asked to start, then probability of B winning the match is.

(a) 1/6 (b) 5/6

(c) 5/11 (d) 6 / 11


Q15. The value of  for which the vectors 3iˆ  6 ˆj  kˆ and 2iˆ  4 ˆj   kˆ are parallel is
2 3
(a) (b)
3 2
5 2
(c) (d)
2 5
Q16. dy
The integrating factor of the differential equation  x log x   y  2 log x is
dx
(a) ex (b) log x

(c) log (log x) (d) x

Q17. The function f  x   x x has a stationary point at


1
(a) x=e (b) x
e

(c) x=1 (d) x e

Q18. The direction ratios of the line 3x + 1 = 6y − 2 = 1 – z are


(a) 3, 6, 1 (b) 3, 6, -1

(c) 2, 1, 6 (d) 2, 1, -6

ASSERTION-REASON BASED QUESTIONS


The following questions consist of two statements – Assertion (A) and Reason (R).
Answer these questions selecting the appropriate option given below:
(a) Both A and R are true and R is the correct explanation for A.
(b) Both A and R are true and R is not the correct explanation for A.
(c) A is true but R is false.
(d) A is false but R is true.

Assertion (A) :The Differential coefficient of sec  tan 1 x  with respect to x is


Q19. x
1  x2
Reason (R) :The Differential coefficient of the function with respect to xis the first order
derivative of the function .

Q20. Assertion (A) : The vector equation of the line passing through the points (6,-4,5) and (3,4,1)
is r  (6 iˆ  4 ˆj  5kˆ)  λ(3 iˆ  8 ˆj  4kˆ) .
Reason (R) : The vector equation of the line passing through the points a and b is
r  a  λ(b  a ) .
Section – B
(This section comprises of very short answer type questions (VSA) of 2 marks each)
Q21. If cos 1 α  cos 1 β  cos 1 γ  3π , then find the value of α  β  γ   β  γ  α   γ  α  β  .
OR
 1  sin x  1  sin x  
Reduce cot 1   where  x   in to simplest form.
 1  sin x  1  sin x  2
Q22. The two equal sides of an isosceles triangle with fixed base b are decreasing at the rate
of 3cm/sec. How fast is the area decreasing when the two equal sides are equal to the base?
OR
The volume of the cube increases at a constant rate. Prove that the increase in its surface area
varies inversely as the length of the side.
Q23. If a  b  c  0 , a  3 , b  5 and c  7 then find the angle between a and b
Q24. x  2 y 1 z  3
Find the point(s) on the line   at a distance 5 𝑢𝑛𝑖𝑡𝑠 from the point (1,3,3).
3 2 2

Q25. Find the area of the region bounded by the curve y 2  4 x , y -axis and line y  3 .
Section – C
(This section comprises of short answer type questions (SA) of 3 marks each)
Q26. Solve the following Linear Programming Problem graphically:
Minimize Z = 3x + 9y
Subject to the constraints
x + 3y ≤ 60
x + y ≥ 10
x≤y
x ≥ 0, y ≥ 0.

Q27. Two numbers are selected at random (without replacement) from positive integers 2, 3, 4, 5, 6
and 7. Let X denote the larger of the two numbers obtained, Find the probability distribution
of X.
OR
A and B are two independent events. The probability that both A and B occur is 1/6 and the
probability that neither of them occur is 1/3. Find the probability of the occurrence of A.

Q28. cos x
Find  1  sin x  2  sin x  dx

OR
sin 
Find  sin 2   2cos   3
d

Q29.  x x

Solve the differential equation 2 ye dx   y  2 xe y  dy  0 .
y
 
 
OR
𝑑𝑦
Solve the differential equation − 3𝑦 cot 𝑥 = sin 2𝑥
𝑑𝑥

Q30. Draw a rough sketch of the curve y  1  x  1 , x  3 , x  3, y  0 and find the area of the
region bounded by them using integration.

Q31. d2y x2  y 2
If x  a sin t  b cos t , y  a cos t  b sin t , then provethat   ( ).
dx 2 y3
Section – D
(This section comprises of long answer type questions (LA) of 5 marks each)
Q32. 1 2 3
If A = [ 2 3 −3], find 𝐴−1 and hence solve the system of equations
−3 2 −4

𝑥 + 2𝑦 − 3𝑧 = −4 ;2𝑥 + 3𝑦 + 2𝑧 = 14 ; 3𝑥 − 3𝑦 − 4𝑧 = −15

Q33. x 3 y 3 z
Find the equations of the lines through the origin which intersect the line   at
2 1 1

an angle of each.
3
OR
x  2 y  3 z 1
Find the equation of the line which intersect the lines   and
1 2 4
x 1 y  2 z  3
  and passes through the point (1, 1, 1).
2 3 4

Q34.  x tan x
Evaluate  0 sec x  tan x
dx .

OR


Evaluate  log 1  cos x  dx
0
Q35. Consider f : R   5,   given by f  x   9 x 2  6 x  5 where R is the set of all non-
negative real numbers. Prove that f is one- one and onto function.

Section – E

(This section comprises of 3 case- study/passage-based questions of 4 marks each with sub parts.
The first two case study questions have three sub parts (i), (ii), (iii) of marks 1,1,2 respectively.
The third case study question has two sub parts of 2 marks each.)

Q36. The use of electric vehicles will curb air pollution in the long run. The use of electric vehicles
is increasing every year and estimated number of electric vehicles in use at any time t is given
by the function
V(t) = 𝑡 3 − 3𝑡 2 + 3𝑡 − 100
Where t represents time and t = 1, 2, 3, --------- corresponds to year 2021, 2022, 2023 -------
respectively.

Based on the above information answer the following:


(i) Can the above function be used to estimate number of vehicles in the year 2020?
Justify.
(ii) Find the estimated number of vehicles in the year 2040.

(iii) Prove that the function V(t) is an increasing function.

Q37. Senior students tend to stay up all night and therefore are not able to wake up on time in
morning. Not only this but their dependence on tuitions further leads to absenteeism in
school. Of the students in class XII, it is known that 30% of the students have 100%
attendance. Previous year results report that 80% of all students who have 100% attendance
attain A grade and 10% irregular students attain A grade in their annual examination. At the
end of the year, one student is chosen at random from the class XII.

Using above information answer the following:

(i). Find the conditional probability that a student attains A grade given that he is not
100 % regular student.
(ii) Find the probability of attaining A grade by the students of class XII
.
(iii) Find the probability that student is 100% regular given that he attains A grade.
OR
Find the probability that student is irregular given that he attains A grade.

Q38. In a park, an open tank is to be constructed using metal sheet with a square base and vertical
sides so that it contains 500 cubic meters of water.

Using above information answer the following:

(i) Find the minimum surface area of the tank.

(ii) Find the percentage increase in volume of the tank, if size of square base of tank
become twice and height remains same.
MARKING SCHEME

Additional Practice Question Paper (2023-24)

CLASS-XII
MATHEMATICS (041)

SECTION: A
(Solution of MCQs of 1 Mark each)
Q.No. Solution Marks
1. x yz 9 , xz 5 , yz 7
On solving above equations, we get x  2 , y  4 , z  3
x  y  z 1
Correct Answer is Option (c) 1 1
2. A(AdjA)  5I
A(AdjA)  A I
 A 5
| Adj A | A  25
2

Correct Answer is Option (b) 25 1


3. Let  ABT  2BAT   C
T

Consider CT   ABT  2BAT   BAT  2 ABT  C or  C


T

 C is neither Symmetric matrix nor Skew symmetric matrix


Correct Answer is Option (c) Neither Symmetric matrix nor Skew
symmetric matrix 1
4. If 1  x  2 then f ( x)  2( x 1)  3( x  2)  x  4
 f / ( x)  1
Hence f ( x) is Strictly decreasing function
Correct Answer is Option (b) Strictly Decreasing 1
5. Set A contains 5 elements and the set B contains 6 elements. For one-one
function each element in set B is assigned to only one element in set A
Correct Answer is Option (d) 0 1
6. Order = 3, Degree = 1
Correct Answer is Option (b) 4 1
7. Correct Answer is Option (b) half plane not containing the origin 1
8. 3 5
AD = iˆ  kˆ
2 2
34
AD 
2
34
Correct Answer is Option (a)
2 1
9. f ( x)  x 3 sin 4 x
f ( x)  ( x) 3 [sin(  x)] 4   x 3 [ sin x] 4   x 3 sin 4 x   f ( x)
 f ( x) is an odd function
 /2
 x sin 4 x dx  0
3


 /2
1
Correct Answer is Option (a) 0
10. A1  kA
1  2 3 2 3 
  k 
19  5 2   5 2 
1
k 
19
Correct Answer is Option (b) 1/19 1
11. Corner Points Value of Z
A (0, 2) Z = 0 + 12 = 12
B (3, 0) Z = 12 + 0 = 12
C (6, 0) Z = 24 + 0 = 24
D (6,8) Z = 24 + 48 = 72
E (0,5) Z = 0 + 30 = 30
Minimum value of Z = 12
Correct Answer is Option (d)Any point on the line segment joining the
points (0,2) and (3,0) 1
12. According to the Question
(iˆ  ˆj  4kˆ).(2iˆ  6 ˆj  3kˆ)
4
2iˆ  6 ˆj  3kˆ

2  6  12
4
4  36  9
  5
Correct Answer is Option (c) 5 1
13.  4 5  3 1  37 14 
 AB   B 1 A1  
1
  
 3 4   5 2   29 11 
 37 14 
Correct Answer is Option (d)   1
 29 11 
14. B will win in second attempt or fourth attempt or sixth attempt or so on
 P( B winning )  P(AB)  P(A B A B)  P(A B A B A B )  ...........
5 1 5 5 5 1 5 5 5 5 5 1
 .  . . .  . . . . .  .............
6 6 6 6 6 6 6 6 6 6 6 6
5
 36  5
1  25 11
36
Correct Answer is Option (c) 5/11 1
15. According to the Question
3 6 1
 
2 4 
2
 
3
Correct Answer is Option (a) 2/3 1
16 
1
dx
Integrating Factor  e  elog(log x )  log x
x log x

Correct Answer is Option (b) log x 1


17. f  ( x)  x (1  log x)
x

For Stationary point f  ( x)  0


1
x x (1  log x)  0  log x  1  x  e1 
e
Correct Answer is Option (b) 1/e 1

18. 3x 1  6 y  2  1  z
3( x 1/ 3)  6( y 1/ 3)  ( z 1)
x  1/ 3 y  1/ 3 z  1
 
1/ 3 1/ 6 1
x  1/ 3 y  1/ 3 z  1
 
2 1 6
Correct Answer is Option (d) 2, 1, - 6. 1
19. Correct Answer is Option (a)Both A and R true and R is the correct
explanation for A. 1
20. Correct Answer is Option (d)A is false but R is true. 1

Section –B
[This section comprises of solution of very short answer type questions
(VSA) of 2 marks each]
21. cos 1 α  cos 1 β  cos 1 γ  3π
 cos –1α  π , cos –1 β  π & cos –1γ  π
 α  β  γ  1 1
α  β  γ  β γ  α   γ α  β 
  1 1  1   1 1  1   1 1  1
 2 – 2  22 1

OR
  x x
2
 x x 
2
  sin  cos    sin  cos  
  2 2  2 2 
cot 1   1
  
2 2
 x x   x x
  sin  cos    sin  cos  
  2 2  2 2 

 x x  x x 
  sin 2  cos 2    sin 2  cos 2  
 cot 1     

 x x   x
  sin  cos    sin  cos   x 
  2 2  2 2  

 x
 2sin 
 cot 1  2  cot 1  tan x  1/2
x 
 2

 2 cos 
 2
  x   x
 cot 1  cot(  )   
 2 2  2 2 1/2
22. In isosceles∆𝐴𝐵𝐶, let AB = AC = a and BC = b (given),

da
=  3cm/sec.
dt

b2
AD = a2 
4

1 1 b2
A = Area of ∆ 𝐴𝐵𝐶= = (BC) (AD) =    b  a 2 
2 2 4 1

1/2
dA  b   1   2 b   da 
2
 =     a   .  2a 
dt 22 4  dt 

b  2a  3 3ab
 =
b2 b2
4 a2  2 a2 
4 4

 dA  3b 2 3b 2
  = = =  3b
 dt  at a b b2 2 3b
2 b  2

4 2
∴ Area is decreasing at the rate of 3b cm/sec. 1
OR
Let the side of a cube be x unit.
Volume of cube = V = 𝑥 3
𝑑𝑉 𝑑𝑥
= 3𝑥 2 𝑑𝑡 = 𝑘 (𝑐𝑜𝑛𝑠𝑡𝑎𝑛𝑡)
𝑑𝑡
𝑑𝑥 𝑘
= 3𝑥 2 1
𝑑𝑡
Surface area = S = 6𝑥 2
𝑑𝑆 𝑑𝑥
= 12 𝑥 𝑑𝑡
𝑑𝑡
𝑑𝑆 𝑘 𝑘
= 12𝑥 3𝑥 2 = 4 (𝑥 )
𝑑𝑡
Hence, the surface area of the cube varies inversely as length of side 1
23. abc  0
a  b  c
 a  b . a  b    c. c  1/2
a . a  a .b  b . a  b .b  c . c
2 2 2
a  2 a .b  b  c
2 2 2 1/2
2 a b cos   c  a  b
2  3 5 cos  49  9  25 1/2
15 1 
 cos      1/2
30 2 3
24. x  2 y 1 z  3
Let P(3  2,2 1,2  3) be any point on a line   which
3 2 2
is at a distance of 5 units from the point Q(1,3,3) .
According to the Question
PQ  5
( PQ) 2  25
(3  2  1)2  (2  1  3) 2  (2  3  3) 2  25 1
17 2  34  0
  0 or 2
Required Point is (-2,-1,3) or (4,3,7) 1

25.
1 Mark
For
Correct
Figure

3
3 y2 y3 
Required Area = 
0 4
dy  
12  0
1/2

27 9
= 0  square units
12 4 1/2

Section –C
[This section comprises of solution short answer type questions (SA) of 3
marks each]
26. x  3 y  60 x  y  10 x y
x  3 y  60 x  y  10 x y
x 0 60 x 0 10 x 0 10
y 20 0 y 10 0 y 0 10

1.5
Marks
For
Correct
Figure

Corner points are A (0, 10), B (5, 5), C (15, 15) and D (0, 20) 1/2
Corner Points Value of Z
A (0, 10) Z = 0 + 90 = 90
B (5, 5) Z = 15 + 45 = 60 1/2
C (15, 15) Z = 45 + 135 = 180
D (0,20) Z = 0 + 180 = 180
1/2
Minimum value of Z = 60

27.
65
n  S  6 C2   15 1
2 1
Let X denote the larger of the two numbers obtained
X = 3, 4, 5, 6, 7 1/2
The Probability Distribution is
X P (X)
3 1/15
4 2/15
5 3/15
6 4/15
1.5
7 5/15

OR

Let P( A)  x and P(B)  y


According to the Question
1 1
P( A  B)  and P(A B)  1
6 3
1 1
P( A) P( B)  and P(A ) P( B) 
6 3
1 1
xy  and (1  x)(1  y)  1
6 3
1 1 1
on solving we get x  or
2 3

28. cos x
I dx
1  sin x  2  sin x 
cos x
 dx put sin x = t
 sin x  1 sin x  2 
cos x dx = dt
dt
I
 t  1 t  2  1

 1 1 
    dt
1
 t 1 t  2 
  log t 1  log t  2

t 2
 log c
t 1
sin x  2 1
 log c
sin x  1
OR
sin 
I d
sin   2cos   3
2

sin 
 d
1  cos2   2cos   3
sin  d 1
 put cos  = t
4  2cos   cos 2 
- sin  d  = dt

 dt dt
  
4  2t  t 2  t 2  2t  4
dt dt 1
   
 t  2t  1  5  5   t  1
2 2 2

 t 1  1  cos   1 
  sin 1    c   sin  c 1
 5  5 

29. x
 x

2 y e y dx   y  2 x e y  dy  0
 
 
x
 x

2 y e y dx   2 x e y  y  dy
 
 
 x

dx  2 x e y  y 
 Put x = vy
dy  x
y


 2y e 
dx dv 1
v y
dy dy
dv 2vy ev  y
v y 
dy 2 yev
dv 2vy ev  y  2vy ev
y 
dy 2 yev
dv 1

dy 2 y ev
1
 2e dv    y dy
v 1

2ev   log y  c
x

2e  log y  c
y
1

OR
dy
 3 y cot x  sin 2 x
dx
dy
Compare with  Py  Q
dx
P  3cot x , Q  sin 2x 1/2
I .F  e  e
P dx 3cot x dx
 e3logsin x

=𝑒 𝑙𝑜𝑔(𝑠𝑖𝑛𝑥)−3 = (𝑠𝑖𝑛𝑥)−3 = 𝑠𝑖𝑛13𝑥 1

The solution of given differential equation is


y  I .F    Q  I .F  dx
 1  1
y  3    sin 2 x. 3 dx
 sin x  sin x
y 1
3
  2sin x cos x. 3 dx
sin x sin x
y 1
sin 3 x 
 2cot x cosecx dx
y
 2cosec x  C
sin 3 x
y  2sin 2 x  c sin 3 x 1/2

30.

1.5
Mark
For
Correct
Figure

1 3
Required Area =   x dx   ( x  2)dx
3 1
1/2

−1 3
−𝑥 2 𝑥2
=[ 2
] + [ 2 + 2𝑥]
−3 −1

1  9 1 
= (1  9)  (  6)  (  2) 
2  2 2 
= 4 12  16squareunits 1

31. x  a sin t  b cos t y  a cos t  b sin t


dx dy
 a cos t  b sin t  y  a sin t  b cos t   x 1
dt dt
dy  x
 1/2
dx y
dy
y  1    x  .
d2y dx 1

 y
2
dx 2
 x
y  x 
  y
 y
2

 y 2  x2  ( y2  x2 )
 

y3 y3 1/2
Section –D
[This section comprises of solution of long answer type questions (LA) of
5 marks each]

32. 1 2 3
A   2 3 3
 3 2 4 
A  1 12  6  2  8  9   3  4  9 
1/2
 6  34  39  67  0
 6 14 15

Adj. A  17 5 9  2

13 8 1 
 6 14 15
1 
1
A   17 5 9  1/2
67
13 8 1 
The matrix form of the equations is
1 2 3  x   4 
2 3 2   y    14 

 3 3 4   z   15
At X B
X   At  B
1

  A1  B
t
1/2
 6 17 13   4  1/2
  14 5 8  14 
1 
67
 15 9 1  15
 24  238  195 
  56  70  120 
1 
67
60  126  15 
 67  1 
1    
 134    2 
67
 201 3 

x = 1, y = 2, z=3 1
33. x 3 y 3 z
Let P(2  3,   3, ) be any point on line  
2 1 1

Let the line through origin and making an angle of with the given line be
3
along OP. Then direction ratios are proportional to 2  3  0,   3  0,   0
i.e. 2  3,   3,  1
Also, direction ratios of the given line are proportional to 2,1,1.

 (2  3)(2)  (  3)(1)  ( )(1)


 cos  1
3 (2  3)2  (  3)2  ( ) 2 22  12  12

1 6  9
 
2 6  18  18 6
2

1 3(2  3)
 
2 6  2  3  3

  2  3  3  (2  3)
Squaring both sides, we get

 2  3  3  (2  3)2
  2  3  2  0
   1, 2
Therefore, the coordinates of point P(1,2, 1) or P(1,1, 2) 2
Hence Equations of required lines are
x y z x y z
  and   1
1 2 1 1 1 2

OR
The lines are
x  2 y  3 z 1 x 1 y  2 z  3
  and  
1 2 4 2 3 4
Let P(  2,2  3,4 1) be any point on line (1) and Q(2  1,3  2,4  3)
be any point on line (2). Also, the given point is A(1,1,1) . 1
For some definite values of  and  , the required line passes through
A, P and Q
The direction ratios of AP are   3,2  2,4  2
The direction ratios of AQ are 2,3 1,4  2
  3 2  2 4  2 1
  
2 3  1 4  2
  3 2  2 2  1
    k (let )
2 3  1 2  1
1
   3  2k , 2  2  3k  k , 2 1  2k  k
 3  3  3
 k  , 2  2  3( )  k , 2  1  2( )k
2 2 2
 3   13
 k  ,k , k 2
2 2
  13
 2 9
2
Also k    2  11 1
Hence The direction ratios of AP are 6,20,34 i.e. 3,10,17 1/2

Therefore, Equation of required line is


x 1 y 1 z 1
  1/2
3 10 17
34.  x tan x
Let I =  dx ……… (1)
0 sec x  tan x

 f  x  dx   f  a  x  dx
a a
Using
0 0

    x  tan    x  dx
I=   
0 sec   x  tan   x
 

   x   tan x  dx
 I= 
0
 sec x  tan x
    x  tan x dx
 I = ……… (2) 1
0sec x  tan x
  x    x  tan x
1   2   2 I  0 dx
sec x  tan x 1

  sin x
2 0 1  sin x
 I= dx

  sin x 1  sin x 
2 0 1  sin x 1  sin x 
= dx

   sin x sin 2 x 
2 0  cos 2 x cos 2 x 
=    dx 1

 
=   tan x sec x  tan 2 x  dx
2 0
 
=  sec x tan x  sec 2 x  1 dx 1
2 0

= sec x  tan x  x 0

2

=  1  0     1  0  0 
2

∴ I =    2 1
2

OR


Let I =  log 1  cos x  dx ...(1)
0


 I=  log 1  cos    x   dx
0

 I   log 1  cos x  dx ...(2)
0
Adding (1) and (2)


2 I   log 1  cos x 1  cos x   dx
0

log 1  cos 2 x  dx
1 
2 0
I=

1   1
 I = 
2 0
log sin 2 x dx   log sin x dx
0

Since log sin    x    log sin x

 /2
∴ I = 2 log sin x dx ...(3)
0

 /2  
 I = 2 log sin   x  dx
0
2 

 /2
 I=2  0
log cos x dx ...(4)
Adding (3) and (4)
 /2 1
2 I  2 log sin x cos x dx
0

 /2  sin 2 x 
 I=  0
log 
 2 
 dx

 /2  /2
= 0
log sin 2 x dx  log 2  1. dx
0

 
 I = I1  log 2  x 0 2  I1  log 2 ...(5) 1
2

 /2
Now I1   log sin 2x dx
0

Let 2 x  t  2dx  dt

1  1 
= 
2 0
log sin t dt   log sin x dx
2 0
(Changing t to x)
1  /2
=  2 log sin x dx
2 0

1
 I1  I 1
2

1 
From (5)  I= I  log 2
2 2
1 
 I   log 2
2 2

1
∴ I =  log 2 .

35. One – one: Let x1 , x2  R such that


f ( x1 )  f ( x2 )
9 x12  6 x1  5  9 x22  6 x2  5
 9  x12  x22   6  x1  x2   0 1

  x1  x2  9  x1  x2   6  0

 x1  x2  0 or 9 x1  9 x2  6  0 which is not possible


 x1  x2
∴ f is one-one. 1
Onto: Let y  9 x 2  6 x  5
 9 x 2  6 x  (5  y )  0

6  36  4(9)(5  y ) 6  36 1  5  y 1/2
 x 
2(9) 18
6(1  y  6) 1  y  6
x 
6(3) 3

1  y  6
Now, x  R  x  0 and so x is rejected
3
1  y  6 1
x 
3

1  y  6
Now x0 0
3
 y  6 1 y  6 1 1
 y  5

∴ R f   y : y   5,    codomain of f.
1/2
∴ f is onto.
Hence f is one one and onto function.

Section –E
[This section comprises solution of 3 case- study/passage-based questions
of 4 marks each with two sub parts. Solution of the first two case study
questions have three sub parts (i),(ii),(iii) of marks 1,1,2 respectively.
Solution of the third case study question has two sub parts of 2 marks
each.)
36. V(t) = 𝑡 3 − 3𝑡 2 + 3𝑡 − 100
(i) No, the above function cannot be used to estimate number of vehicles in
the year 2020 because for 2020 we have t = 0 and
V(0) = 0 − 0 + 0 − 100 = −100
Which is not possible 1
(ii) V(20) = (20)3 − 3(20)2 + 3(20) − 100
Therefore, the estimated number of vehicles in the year 2040 are 6760. 1
(iii)
𝑉 ′ (t) = 3𝑡 2 − 6𝑡 + 3 1
= 3(𝑡 2 − 2𝑡 + 1)
= 3(𝑡 − 1)2 ≥ 0.
Hence V (t ) is always increasing function. 1

37. Let E1 is the event that a student is regular


E2 is the event that a student is irregular
A is the event that a student attains grade A
30 70
P( E1 )  , P( E2 ) 
100 100

80 10
P(A / E1 )  , P(A / E2 ) 
100 100
10 1
(i) Required Probability = P(A / E2 )   1
100 10
(ii ) Required Probability  P(A )
 P( E1 ) P(A / E1 )  P( E2 ) P(A / E2 )
30 80 70 10 31 1
 .  . 
100 100 100 100 100
(iii ) Required Probability  P(E1 / A )
P( E1 ) P(A / E1 )
 1
P( E1 ) P(A / E1 )  P( E2 ) P(A / E2 )
30 80
. 1
100 100 24
 
30 80 70 10 31
.  .
100 100 100 100

OR
(iii ) Required Probability  P(E 2 / A )
P( E2 ) P(A / E2 )
 1
P( E1 ) P(A / E1 )  P( E2 ) P(A / E2 )
70 10
.
100 100 7
  1
30 80 70 10 31
.  .
100 100 100 100
38. (i) Let length, breadth and height of the tank are x, x and y respectively
According to the Question
500
∴ x y  500  y  2
2

x 1/2
Surface Area  S  x  4 xy
2

500 2000
S  x 2  4 x( 2
)  x2  1/2
x x

dS 2000
  2x  2
dx x
dS 2000
For maxima or minima,  0  2 x  2  0  x  10 m
dx x
d 2S 4000
Now 2
 2 3 1/2
dx x

 d 2S  4000
and  2   2 0
 dx  at x 10 (10)3
Surface Area is minimum when x  10 m
2000
 Minimum Surface Area = 100  = 300 m2 1/2
10

(ii) If x  10 m then y  5 m
and Volume of the tank  x 2 y  (10)2 (5)  500 m3 1/2
New Volume  (2 x)2 y  4 x 2 y  4(10)2 (5)  2000 m3 1/2
 Increase in Volume of the tank  2000  500  1500 m3 1/2
 % Increase in Volume of the tank  300% 1/2
To Get More material for
Maths IX - X Click below

CBSE MATHS
IX-X

To Get More Material for XI-


XII Click below

CBSE MATHS
XI-XII
CBSE
Additional Practice Questions
Subject: Mathematics (041)
Class: XII 2023-24
Time Allowed: 3 Hours Maximum Marks: 80
General Instructions:
1. This Question paper contains - five sections A, B, C, D and E. Each section is compulsory.
However, there are internal choices in some questions.
2. Section A has 18 MCQs and 02 Assertion-Reason based questions of 1 mark each.
3. Section B has 5 Very Short Answer (VSA)-type questions of 2 marks each.
4. Section C has 6 Short Answer (SA)-type questions of 3 marks each.
5. Section D has 4 Long Answer (LA)-type questions of 5 marks each.
6. Section E has 3 source based/case based/passage based/integrated units of assessment (4
marks each) with sub parts.

SECTION A
(This section comprises of Multiple-choice questions (MCQ) of 1 mark each.)

Serial
No. Question Marks
1 For any 2 × 2 matrix P, which of the following matrices can be Q such that PQ 1
= QP?

(a)

(b)

(c)

(d)
2 V is a matrix of order 3 such that |adj V| = 7. 1

Which of these could be |V|?

(a)
(b)
(c)
(d)

3 The points D, E and F are the mid-points of AB, BC and CA respectively. 1

(Note: The figure is not to scale.)

What is the area of the shaded region?

(a) 2 sq units
3
(b) 2 sq units
1
(c) 2 sq units
(d) (2√26 - 1) sq units

4 If f(x) = cos-1 √x, 0 < x < 1, which of the following is equal to f'(x)? 1

(a)
(b)

(c)

(d)

5 A function f: R -> R is defined by: 1

Which of the following statements is true about the function at the point
1
x = ln 2 ?

(a) f(x) is not continuous but differentiable.


(b) f(x) is continuous but not differentiable.
(c) f(x) is neither continuous nor differentiable.
(d) f(x) is both continuous as well as differentiable.

6 In which of these intervals is the function f(x) = 3x 2 - 4x strictly decreasing? 1

(a) (-∞, 0)
(b) (0, 2)
2
(c) (3 , ∞)
(d) (-∞, ∞)

7 Which of these is equal to ∫ 𝑒 (𝑥 log 5) 𝑒 𝑥 𝑑𝑥, where C is the constant of 1


integration?

(a)
(b)
(c)
(d)
8 Shown below is the curve defined by the equation y = log (x + 1) for x ≥ 0. 1

Which of these is the area of the shaded region?

(a) 6log(2) - 2
(b) 6log(2) - 6
(c) 6log(2)
(d) 5log(2)

9 In which of the following differential equations is the degree equal to its order? 1

(a)

(b)

(c)

(d)
10 Kapila is trying to find the general solution of the following differential 1
equations.

Which of the above become variable separable by substituting y = b.x, where


b is a variable?

(a) only (i)


(b) only (i) and (ii)
(c) all - (i), (ii) and (iii)
(d) None of the above

11 1

(a) only (i)


(b) only (ii)
(c) only (i) and (ii)
(d) only (ii) and (iii)

12 1

3
(a) λ = , σ = 0
5
5
(b) λ = 3 , σ = 5
(c) λ = 3, σ = 0
(d) (cannot be found as there are two unknowns and only one equation)
13 1

(a) (2, √2, 2)


(b) (√2, 2, √2)
1 1 1
(c) (2 , , )
√2 2
(d) (2√2, 2√2, 2√2)

14 A line m passes through the point (-4, 2, -3) and is parallel to line n, given by: 1

The vector equation of line m is given by:

Which of the following could be the possible values for p, q and r?

(a) p = 4, q = (-2), r = 3
(b) p = (-4), q = (-2), r = 3
(c) p = (-2), q = 3, r = (-6)
(d) p = 8, q = 4, r = (-3)

15 L1 and L2 are two skew lines. 1


How many lines joining L1 and L2 can be drawn such that the line is
perpendicular to both L1 and L2 ?

(a) exactly one


(b) exactly two
(c) infinitely many
(d) (there cannot be a line joining two skew lines such that it is perpendicular
to both)

16 A linear programming problem (LPP) along with the graph of its constraints is 1
shown below. The corresponding objective function is Minimize: Z = 3x + 2y.
The minimum value of the objective function is obtained at the corner point (2,
0).

The optimal solution of the above linear programming problem _________.

(a) does not exist as the feasible region is unbounded.

(b) does not exist as the inequality 3x + 2y < 6 does not have any point in
common with the feasible region.

(c) exists as the inequality 3x + 2y > 6 has infinitely many points in common with
the feasible region.

(d) exists as the inequality 3x + 2y < 6 does not have any point in common with
the feasible region.
17 The feasible region of a linear programming problem is bounded. The 1
corresponding objective function is Z = 6x - 7y.
The objective function attains __________ in the feasible region.

(a) only minimum


(b) only maximum
(c) both maximum and minimum
(d) either maximum or minimum but not both

18 M and N are two events such that P(M ∩ N) = 0. 1

Which of the following is equal to P(M|(M ∪ N))?

(a)

(b)

(c)

(d)

19 X = {0, 2, 4, 6, 8}. 1
P is a relation on X defined by P = {(0, 2), (4, 2), (4, 6), (8, 6), (2, 4), (0, 4)}.

Based on the above information, two statements are given below - one labelled
Assertion (A) and the other labelled Reason (R). Read the statements carefully
and choose the option that correctly describes statements (A) and (R).

Assertion (A): The relation P on set X is a transitive relation.

Reason (R): The relation P has a subset of the form {(a, b), (b, c), (a, c)},
where a, b, c ∈ X.

(a) Both (A) and (R) are true and (R) is the correct explanation for (A).
(b) Both (A) and (R) are true but (R) is not the correct explanation for (A).
(c) (A) is true but (R) is false.
(d) (A) is false but (R) is true.

20 Two statements are given below - one labelled Assertion (A) and the other 1
labelled Reason (R). Read the statements carefully and choose the option that
correctly describes statements (A) and (R).
Assertion (A): The maximum value of the function f(x) = x 5 , x ∈ [-1, 1], is
attained at its critical point, x = 0.

Reason (R): The maximum of a function can only occur at points where
derivative is zero.

(a) Both (A) and (R) are true and (R) is the correct explanation for (A).
(b) Both (A) and (R) are true but (R) is not the correct explanation for (A).
(c) (A) is false but (R) is true.
(d) Both (A) and (R) are false.

SECTION B
(This section comprises of very short answer type-questions (VSA) of 2 marks
each.)

Serial
No. Question Marks
21 Find the domain of the function y = cos-1 (|x - 1|). Show your steps. 2

OR

Draw the graph of the following function: 2

y = 2sin-1 (x), -π ≤ y ≤ π

22 2

23 If x = cot t and y = cosec2 t, find: 2


Show your steps.

24 Iqbal, a data analyst in a social media platform is tracking the number of active 2
users on their site between 5 pm and 6 pm on a particular day.

The user growth function is modelled by N(t) = 1000e0.1t, where N(t)


represents the number of active users at time t minutes during that period.

Find how fast the number of active users are increasing or decreasing at 10
minutes past 5 pm. Show your steps.

OR

The population of rabbits in a forest is modelled by the function below: 2

Determine whether the rabbit population is increasing or not, and justify your
answer.

25 Solve the integral: 2

Show your steps.

SECTION C
(This section comprises of short answer type questions (SA) of 3 marks each)

Serial
No. Question Marks
3
26
Solve the integral:

Show your work.

27 Evaluate the integral: 3

Show your steps.

OR

Using the properties of definite integrals, prove the following: 3

State the property used.

28 When an object is thrown vertically upward, it is under the effect of gravity 3


and air resistance. For small objects, the force due to air resistance is
numerically equal to some constant k times v, where v is the velocity of the
object (in m/s) at time t (s).

This situation can be modelled as the differential equation shown below.

A tennis ball of mass 0.050 kg is hit upwards with a velocity of 10 m/s. An air
resistance numerically equal to 0.4v acts on the ball.

(i) Model the above situation using a differential equation.


(ii) Write an expression for the velocity of the ball in terms of the time.

Show your work.


29 Shown below is a curve. 3

L1 is the tangent to any point (x, y) on the curve.


L2 is the line that connects the point (x, y) to the origin.

The slope of L1 is one third of the slope of L2 .

Find the equation of the curve. Show your work.

OR

(i) Solve the differential equation and show that the solution represents a
family of circles.

(ii) Find the radius of a circle belonging to the above family that passes
through the origin.

Show your work.

30 Each unit of Product A that a company produces, is sold for Rs 100 with a 3
production cost of Rs 60 and each unit of Product B is sold for Rs 150 with
production cost of Rs 90. On a given day, the company has a budget of Rs
8000 to spend on production. The production process makes it such that they
can only produce a maximum of 100 units each day. Also, the number of
product B produced cannot be more than twice as many of Product A.

Frame a linear programming problem to determine how many units of


Product A and B should the company produce in a day in order to maximize
their profit?
(Note: No need to find the feasible region and optimal solution.)

OR

Shown below is the feasible region of a maximisation problem whose 3


objective function is given by Z = 5x + 3y.

i) List all the constraints the problem is subjected to.


ii) Find the optimal solution of the problem.

Show your work.


31 A company follows a model of bifurcating the tasks into the categories shown 3
below.
At the beginning of a financial year, it was noticed that:

♦ 40% of the total tasks were urgent and the rest were not.
♦ half of the urgent tasks were important, and
♦ 30% of the tasks that were not urgent, were not important

What is the probability that a randomly selected task that is not important is
urgent? Use Bayes' theorem and show your steps.

SECTION D
(This section comprises of long answer-type questions (LA) of 5 marks each)

Serial
No. Question Marks
32 The Earth has 24 time zones, defined by dividing the Earth into 24 equal 5
longitudinal segments. These are the regions on Earth that have the same
standard time. For example, USA and India fall in different time zones, but
Sri Lanka and India are in the same time zone.

A relation R is defined on the set U = {All people on the Earth} such that R =
{(x, y)| the time difference between the time zones x and y reside in is 6
hours}.

i) Check whether the relation R is reflexive, symmetric and transitive.


ii) Is relation R an equivalence relation?

Show your work.


OR

A function f : R - {-1, 1} -> R is defined by: 5

i) Check if f is one-one.
ii) Check if f is onto.

Show your work.

33 Abdul threw a basketball in the direction of the basketball hoop which 5


traversed a parabolic path in a vertical plane as shown below.

(Note: The image is for representation purpose only.)

The equation of the path traversed by the ball is y = ax 2 + bx + c with respect


to a xy-coordinate system in the vertical plane. The ball traversed through the
points (10, 16), (20, 22) and (30, 25). The basketball hoop is at a horizontal
distance of 70 feet from Abdul. The height of the basketball hoop is 10 feet
from the floor to the top edge of the rim.

Did the ball successfully go through the hoop? Justify your answer.

(Hint: Consider the point where Abdul is standing as the origin of the
xy-coordinate system.)

34 Shown below are concrete elliptical water pipes, each 10 feet in length. 5
The graph given above represents the inner circumference of the elliptical
pipe, where x and y are in feet. Assume that the water flows uniformly and
fully covers the inner cross-sectional area of the pipe.

Find the volume of water in the pipe at a given instant of time, in terms of π.
Use the integration method and show your steps.

(Note: Volume = Area of the base × Height)

35 i) Find the vector and cartesian equations of the straight line passing through 5
the point (-5, 7, -4) and in the direction of (3, -2, 1).

ii) Find the point where this straight line crosses the xy-plane.

Show your work.


OR

Given below are two lines L1 and L2 : 5

L1 : 2x = 3y = -z
L2 : 6x = -y = -4z

i) Find the angle between the two lines.


ii) Find the shortest distance between the two lines.

Show your work.


SECTION E
(This section comprises of 3 case-study/passage-based questions of 4 marks
each with two sub-questions. First two case study questions have three sub
questions of marks 1, 1, 2 respectively. The third case study question has two
sub questions of 2 marks each.)

Serial
No. Question Marks

36 Answer the questions based on the given information.


The flight path of two airplanes in a flight simulator game are shown below.
The coordinates of the airports P and Q are given.

Airplane 1 flies directly from P to Q.


Airplane 2 has a layover at R and then flies to Q.

(Note: Assume that the flight path is straight and fuel is consumed uniformly
throughout the flight.)
1
i) Find the vector that represents the flight path of Airplane 1. Show your
steps.

ii) Write the vector representing the path of Airplane 2 from R to Q. Show 1
your steps.
iii) What is the angle between the flight paths of Airplane 1 and Airplane 2 2
just after takeoff? Show your work.

OR

iii) Consider that Airplane 1 started the flight with a full fuel tank. 2

Find the position vector of the point where a third of the fuel runs out if the
entire fuel is required for the flight. Show your work.

37 Answer the questions based on the given information.

Rubiya, Thaksh, Shanteri, and Lilly entered a spinning zone for a fun game,
but there is a twist: they don't know which spinner will appear on their
screens until it is their turn to play. They may encounter one of the following
spinners, or perhaps even both:

Different combinations of numbers will lead to exciting prizes. Below are


some of the rewards they can win:

♦ Get the number '5', from Spinner A and '8' from Spinner B, and you'll win a
music player!
♦ You win a photo frame if Spinner A lands on a value greater than that of
Spinner B!
i) Thaksh spun both the spinners, A and B in one of his turns. 1

What is the probability that Thaksh wins a music player in that turn? Show
your steps.

ii) Lilly spun spinner B in one of her turns. 1

What is the probability that the number she got is even given that it is a
multiple of 3? Show your steps.

iii) Rubiya spun both the spinners. 2

What is the probability that she wins a photo frame? Show your work.

OR

iii) As Shanteri steps up to the screen, the game administrator reveals that for 2
her turn, the probability of seeing Spinner A on the screen is 65%, while that
of Spinner B is 35%.

What is the probability that Shanteri gets the number '2'? Show your steps.

38 Answer the questions based on the given information.

Two metal rods, R1 and R2 , of lengths 16 m and 12 m respectively, are


insulated at both the ends. Rod R1 is being heated from a specific point while
rod R2 is being cooled from a specific point.

The temperature (T) in Celsius within both rods fluctuates based on the
distance (x) measured from either end. The temperature at a particular point
along the rod is determined by the equations T = (16 - x)x and T = (x -
12)x for rods R1 and R2 respectively, where the distance x is measured in
meters from one of the ends.

i) Find the rate of change of temperature at the mid point of the rod that is
being heated. Show your steps. 2

ii) Find the minimum temperature attained by the rod that is being cooled. 2
Show your work.
CBSE
Additional Practice Questions-Marking Scheme
Subject: Mathematics (041)
Class: XII 2023-24

Time Allowed: 3 Hours Maximum Marks: 80

SECTION A
Multiple Choice Questions of 1 mark each.
Q
No. Answer/Solution Marks
1 (b) 1

2 (c) 1

3 3 1
(b) 2 sq units

4 (d) 1

5 (b) f(x) is continuous but not differentiable. 1

6 (a) (-∞, 0) 1

7 (a) 1

8 (a) 6log(2) – 2 1

9 (c) 1
10 (a) only (i) 1

11 (d) only (ii) and (iii) 1

12 3 1
(a) λ = 5 , σ = 0

13 (b) (√2, 2, √2) 1


14 (d) p = 8, q = 4, r = (-3) 1

15 (a) exactly one 1

16 (d) exists as the inequality 3x + 2y < 6 does not have any point in common 1
with the feasible region.

17 (c) both maximum and minimum 1

18 (c) 1

19 (d) (A) is false but (R) is true. 1

20 (d) Both (A) and (R) are false. 1

SECTION B
Very short answer questions of 2 marks each.
Q
No. Answer/Solution Marks
21 Since the domain of inverse of cosine function is [-1, 1], finds the domain of the given 1.5
function as follows:

-1 ≤ x - 1 ≤ 1
So, 0 ≤ x ≤ 2

And,

-1 ≤ 1 - x ≤ 1
=> 1 ≥ x - 1 ≥ -1
So, 2 ≥ x ≥ 0

Concludes the domain of cos-1 (|x - 1|) as [0, 2]. 0.5

OR

Draws the graphs of y = 2sin-1 (x) as shown below.

2.0
22

0.5

Obtains x = 3 and w = 2 using the relationship obtained in the previous step as


follows:

2x = 6 => x = 3 0.5
2w = 4 => w = 2

Writes any value of y and z that satisfies the third relationship obtained in the first
step. For example, y = 0 and z = -1. 0.5

0.5

(Award full marks for any other matrix that satisfies the relationship.)
23 0.5

0.5

0.5

0.5

(Award full marks if any alternate method is used.)


24 Writes that the rate at which the number of active users is increasing or decreasing at 0.5
𝑑
a given time is given by 𝑑𝑡 N(t).

Finds the derivative of N(t) as:


𝑑 1.0
N(t) = 1000 (0.1) e0.1t
𝑑𝑡

Finds the rate of change of active users at 10 minutes past 5 pm as:


𝑑
N(10) = 1000(0.1)e(0.1)(10) = 100e
𝑑𝑡

Concludes that the number of active users are increasing at a rate of 100e people per
0.5
minute at 5:10 pm on that day.

OR

Finds the derivative of the given function as:

1.0

Writes that the above quantity is greater than 0 for any value of t. 0.5

Concludes that the rabbit population is increasing. 0.5


25 Substitutes (k - x) by u to get dx = -du and rewrites the given integral as:

0.5

Integrates the expression in above step as:

1.0
where, C is the constant of integration.

Substitutes u = (k - x) back in the above expression and writes:

0.5

where, C is the constant of integration.


SECTION C
Short answer questions of 3 marks each.
Q
No. Answer/Solution Marks
26 Rewrites the numerator of the given integral as:

𝑑
3x + 5 = A 𝑑𝑥 (x 2 + 4x + 7) + B
0.5
=> 3x + 5 = A(2x + 4) + B

Finds the values of A and B by comparing the coefficients of like terms as:
3
2A = 3 => A = 2
1.0
4A + B = 5 => B = -1

Substitutes the values of A and B in the given integral and integrates the same as:

1.5

where, C is the constant of integration.

27 Takes u = (25 + cos θ).

Finds du as:
0.5
du = -sin θ dθ

𝜋
Finds the change in limit when θ = 0 and θ = 2 to u = 26 and u = 25 respectively. 0.5

Rewrites the given integral using the above substitution and integrates the same as:
1.5

0.5

Applies the limit to find the value of the given definite integral as
26 ×26
log .
25 ×27

(Award full marks if the problem is solved correctly by taking u = 26 + cos θ.)
1.0
OR

States the property that is going to be used as:

Takes 2a = π and f(x) = h(sin x).

Finds f(2a - x) as:

f(2a - x) = f(π - x) = h(sin(π - x)) = h(sin x) = f(x) 2.0

Thus confirms that the property listed in the above step can be applied to the given
integral.

Hence concludes that:

28 (i) Models the situation and rearranges terms to form a linear differential equation as
follows:
0.5
𝑑𝑦
(ii) Considering the obtained equation as linear of the form + Py = Q with P = 8 and
𝑑𝑥
hence takes the integrating factor as:
0.5

Multiplies the differential equation by the integrating factor as follows:

0.5

Integrates both sides to obtain the general solution of the differential equation as
follows:

1.0

Uses the initial condition v(0) = 10 m/s to find the value of C as follows:

Hence, writes the expression for the velocity of the ball as a function of time as
follows:

0.5

29 Frames the differential equation using the given conditions as follows:

1.0

Rearranges the terms to separate the variables as follows:


0.5

Integrates both sides to obtain the general solution of the curve. The working may look
as follows:
1.0

Substitutes (8, 2) to obtain the equation of the curve as follows:


0.5

OR

(i) Separates the variables and rearranges the terms of the differential equation as
follows:
0.5

Integrates both sides to obtain the following:


1.0

0.5
Writes that the solution is a general solution of a circle and hence it represents a family
of circles.

(ii) Substitutes x = 0 and y = 0 into the general solution to obtain C 2 = 0 and writes the
particular solution as:
0.5
x2 + y2 - 4x + 2y = 0

Rearranges the terms to rewrite the particular solution as (x - 2)2 + (y + 1)2 = 5 to find
the radius as √5 units. The working may look as follows:

x 2 + y2 - 4x + 2y = 0
Adding 5 to both sides and rearranging terms,
0.5
=> (x 2 - 4x + 4) + (y2 + 2y + 1) = 5
=> (x - 2)2 + (y + 1)2 = 5
30 Finds the profit on selling the products as:

Profit for each unit of Product A sold = 100 - 60 = Rs 40


0.5
Profit for each unit of Product B sold = 150 - 90 = Rs 60.

Takes x and y to be the numbers of Product A and Product B to be produced in a day


respectively and frames the objective function as:

Maximise Z = 40x + 60y 1.0

Writes the constraints of the given linear programming problem as:

60x + 90y ≤ 8000


x + y ≤ 100 1.5
y ≤ 2x or -2x + y ≤ 0
x, y ≥ 0

OR

i) Uses the graph of the feasible region and lists the constraints of the given
maximisation problem as:

3x + 2y ≤ 12
x + 2y ≤ 8 1.5
x, y ≥ 0

ii) Finds the value of the objective function at corner points as:

Corner point z = 5x + 3y
(0, 0) 0
1.0
(0, 4) 12
(2, 3) 19
(4, 0) 20

Concludes that the objective function attains maximum value at (4, 0) and hence (4, 0) 0.5
is the optimal solution.

31 Applies Bayes' theorem and writes:


1.0

Substitutes respective probabilities in the expression obtained above to find the


required probability as follows:

1.5

Simplifies the above expression to get the probability that a randomly selected task that
10 0.5
is not important is urgent as 19 or 52.63%.

SECTION D
Long answer questions of 5 marks each.
Q
No. Answer/Solution Marks
32 i) Writes that for no x ∈ U, (x, x) ∈ R as the difference in time between x & x is
0 hours.

Concludes that R is not reflexive. 1.5

Writes that, whenever the difference in time between x and y is 6 hours, the
difference in time between y and x is also 6 hours. That is, (x, y) ∈ R =>
(y, x) ∈ R.
1.5
Concludes that R is symmetric.

Writes that, if the difference in time between x and y is 6 hours, and the
difference in time between y and z is also 6 hours, then the difference in time
between x and z could be either 0 hours or 12 hours. That is, (x, y) ∈ R &
(y, z) ∈ R but (x, z) ∉ R.

Concludes that R is not transitive. 1.5

ii) From the above steps, concludes that R is not an equivalence relation. 0.5

OR

i) Assumes f(x) = f(y) and evaluates the same as:

1.5

Uses the above step to conclude that x = y or xy = -1. 0.5

Takes a pair of numbers x and y such that xy = -1 to show that f is not one-one.

For example,
1 2 2
for x = and y = -2, f(x) = − and f(y) = − .
2 3 3
1.0
ii) Equates f(x) to y and solves the same to express x in terms of y as:

1.5

Writes that for any y ∈ R (codomain), there exists x ∈ R - {-1, 1} (domain) 0.5
such that f(x) = y. Hence concludes that f is onto.

33 Writes the system of equations as:

100a + 10b + c = 16
400a + 20b + c = 22 0.5
900a + 30b + c = 25

Writes the above system of equations in the form AX = B as:

0.5

Finds |A| as 1(18000 - 12000) - 1(3000 - 9000) + 1(2000 - 4000) = -2000 and 0.5
writes that A-1 exists as |A| ≠ 0.

Finds A-1 as:

1.5

(Award 1 mark if only all the cofactors are found correctly.)


3 21
Finds the values of a, b and c as − 200 , 20 and 7 respectively by solving
X = A-1 B as:

1.0

Finds the equation of the path traversed by the ball as: 0.5
3 21
y = − 200 x 2 + 20 x + 7.

Writes that when x = 70 feet, y = 7 feet. So, the ball went by 7 feet above the
floor that means 3 feet below the basketball hoop. So, the ball did not go 0.5
through the hoop.

34 Finds the equation of the ellipse as:

0.5

Expresses y in terms of x as:


0.5

Integrates the above equation with respect to x from limit 0 to 6, that gives the
area of one quarter of the ellipse. The working may look as follows:
0.5

Applies the formula of integration and simplifies as:


1.0

Applies the limit and solves further as:


0.5

Simplifies the above expression to get the area of one-quarter of the base as 6π 1.0
sq feet.
Finds the area of the whole ellipse as 4 × 6π = 24π sq feet. 0.5
Finds the volume of water as 24π × 10 = 240π cubic feet. 0.5
35 0.5

Writes the vector equation of the given straight line as:


1.0

Writes the cartesian equation of the given straight line as:


1.0

ii) Simplifies the vector form obtained in step 2 as:


1.0

Writes that at the point where the line crosses xy-plane, its z-coordinate is zero
and equates the z-coordinate of the above equation to zero as:

λ-4=0 0.5
=> λ = 4

Substitutes λ = 4 in the vector form to get the required point as (7, -1, 0). 1.0

OR

Rewrites the equation of L1 in cartesian form as:


𝑥 𝑦 𝑧
= 2 = −6 0.5
3

Rewrites the equation of L2 in cartesian form as:


0.5
𝑥 𝑦 𝑧
= −12 = −3
2

i) Identifies the direction cosines of both the lines as (3, 2, -6) and 0.5
(2, -12, -3).

Finds the cosine of the angle between the two lines as:
1.5

(Award 0.5 marks if only the formula of the cosine of the angle between the
two lines is written correctly.)
Concludes that the angle between the two lines is 90°. 0.5
ii) Rewrites the equations of L1 and L2 in vector form as:

1.0

Writes that both the lines pass through the origin hence intersect at the origin.

(Award full marks if the inference about both lines passing through the origin
is drawn without writing the vector forms.)

Writes that since both the lines intersect at the origin, the shortest distance 0.5
between the two lines is 0 units.

SECTION E
Case-based questions of 4 marks each.
Q
No. Answer/Solution Marks
36 Writes the vectors for points P and Q as follows:
i)

0.5

Finds the vector representing the flight path of Airplane 1 as:

0.5

36 Uses vector subtraction to find the vector representing the flight path from R to
ii) Q as:

1.0
36 Finds the cosine of the angle between the vectors representing the flight paths
iii) of Airplane 1 and Airplane 2 as:

1.5

Finds the angle between the flight paths as:

0.5

OR

Considers a point S which divides PQ internally in the ratio 1:2. 0.5

Finds the position vector of point S as:

1.5

(Award 0.5 marks if only the formula is written correctly.)

37 Finds the required probability as:


i)

1.0

37 Uses the conditional probability and finds the required probability as follows:
ii)

1.0
37 Finds the probability of getting 2 from spinner A and getting 1 from spinner B
iii) as:
0.5

Finds the probability of getting 5 from spinner A and getting either 1, 2, 3 or 4


from spinner B as:

1.0

Writes that P1 and P2 are mutually exclusive and hence, finds the probability
that she wins a photo frame as:

0.5

OR

Uses the theorem of total probability and writes:

P(getting 2) = [P(Spinner A) × P(Getting 2|Spinner A)] +


[P(Spinner B) × P(Getting 2|Spinner B)] 0.5

Finds the required probability by substituting the required probability as:

1.5

38 Identifies that the rod being heated is R1 and finds the rate of change of
i) temperature at any distance from one end of R1 as:
1.0

0.5
Finds the mid-point of the rod as x = 8 m.
Finds the rate of change of temperature at the mid point of R1 as:

0.5

38 Identifies that the rod being cooled is R2 and finds the rate of change of
ii) temperature at any distance x m as:

𝑑𝑇
Equates to 0 to get the critical point as x = 6. 1.0
𝑑𝑥

Finds the second derivative of T as:

0.5

Finds the minimum temperature attained by the rod R2 as


0.5
T(6) = (6 - 12)6 = -36 °C.
Practice Questions
Session 2022-23
Class XII
Mathematics (Code-041)

Time Allowed: 3 Hours Maximum Marks: 80

General Instructions:
1. This Question paper contains - five sections A, B, C, D and E. Each section is
compulsory. However, there are internal choices in some questions.
2. Section A has 18 MCQs and 02 Assertion-Reason based questions of 1 mark each.
3. Section B has 5 Very Short Answer (VSA)-type questions of 2 marks each.
4. Section C has 6 Short Answer (SA)-type questions of 3 marks each.
5. Section D has 4 Long Answer (LA)-type questions of 5 marks each.
6. Section E has 3 source based/case based/passage based/integrated units of
assessment (4 marks each) with sub parts.

SECTION A
(This section comprises of Multiple-choice questions (MCQ) of 1 mark each.)
Q.No Question Marks
Q.1 Shown below is the graph of a function f(x) whose domain is R – (-1,1). Some portion of
the graph is hidden behind the star. 1

Which of the following is f(x)?

A. tan-1 x
B. cot-1 x
C. sec-1 x
D. cosec-1 x
Q.2 P and Q are matrices such that both (P + Q) and (PQ) are defined. 1

Which of the following is true about P and Q?

A. P and Q can be any matrices but of the same order.


B. P and Q must be square matrices of the same order.
C. P and Q must be square matrices not necessarily of the same order.
D. Order of P and Q must be of the form m × k and k × n respectively, with no
condition on m and n.

Q.3 1

Under which of the following conditions will |A| be equal to 0?

i) a - 2p = b - 2q = c - 2r = 0
ii) x = y = z = 0
iii) a : b : c = x : y : z

A. only ii)
B. only i) and ii)
C. only i) and iii)
D. all - i), ii) and iii)

Q.4 If abc = 2, what is the value of the determinant below? 1

A. -48
B. -24
C. 48
D. (cannot be found without the values of a, b and c)

1
Q.5
For what value of k is the function f continuous at x = 0?

A. 4
B. 1
1
C. 4
1
D. 8

Q.6 What is the integral of the following expression? 1

A. − tan 𝑥 + 𝐴, where A is a constant.


1
B. − tan 𝑥 − 𝐵, where B is a constant.
1
C. tan 𝑥 + 𝐶, where C is a constant.
2
1 1
D. 𝑠𝑒𝑐 𝑥 𝑡𝑎𝑛 − 𝐷, where D is a constant.
𝑥

Q.7 What is the value of the following integral? 1

A. -4
B. -2
C. 0
D. 4

Q.8 Which of the following is CLOSEST to the area under the parabola given by y = 4x2, 1
bounded by the x-axis, and the lines x = (-1) and x = (-2)?

A. 6 sq units
B. 8 sq units
C. 9 sq units
D. 12 sq units

Q.9 Which of the following differential equation has an order of 2 and a degree of 3? 1
A.

B.

C.

D.

Q.10 Following is a differential equation. 1


𝑑𝑦
= 4𝑒 3𝑥
𝑑𝑥

7
If y(0) = , which of the following is a particular solution of the differential equation?
3

4 4
A. 𝑒 3𝑥 − 𝑒7
3 3
29
B. 12𝑒 3𝑥 − 3
4
C. 𝑒 3𝑥 + 1
3
5
D. 4𝑒 3𝑥 − 3

Q.11 1
A. only ii)
B. only iii)
C. only i) and ii)
D. all - i), ii) and iii)

Q.12 1

A.

B.

C.

D.

Q.13 1
A. 8
B. 4
C. 2
D. 0

1
Q.14 A line makes an angle of 135° with the positive direction of the x-axis, and an angle of
300° with the positive direction of the y-axis.

Which of the following could be the angle it makes with the negative direction of the z-
axis?

A. 45°
B. 60°
C. (Such a line does not exist.)
D. (A unique angle made with the z-axis cannot be determined.)

Q.15 1

A. -9
B. -8
C. 7
D. 8

Q.16 A linear programming problem (LPP) along with its constraints is given below. 1

Minimize: 𝑍 = 3x + 2y

Subject to:
5x − 10y ≥ 0
x + y ≤ 1
x ≤ 4
x ≥ 0, y ≥ 0

Which of the following is true about the above LPP?

A. It has no solution.
B. It has a unique solution.
C. It has two distinct solutions.
D. It has infinitely many solutions.

Q.17 M and N are two events such that P(M|N) = 0.3, P(M)= 0.2 and P(N) = 0.4. 1

Which of the following is the value of P(M ∩ N')?

A. 0.8
B. 0.12
C. 0.1
D. 0.08

1
Q.18 The constraints of a linear programming problem along with their graphs is shown below:

x + 2y ≥ 3
x ≥ 10
y ≥ 0
Which of the following objective functions has an optimal solution with respect to the
above set of constraints?

A. Minimise Z = x + y
B. Minimise Z = 0.5x + y
C. Maximise Z = x + y
D. Maximise Z = 2x + y

Q.19
1
Based on the above function, two statements are given below - one labelled Assertion (A)
and the other labelled Reason (R). Read the statements carefully and choose the option that
correctly describes statements (A) and (R).

Assertion (A): The function f is not onto.

Reason (R): 3 ∈ R (co-domain of f) has no pre-image in the domain of f.

A. Both A and R are true and R is the correct explanation of A.


B. Both A and R are true but R is not the correct explanation of A.
C. A is true but R is false.
D. Both A and R are false.

1
Q.20 Two statements are given below - one labelled Assertion (A) and the other labelled Reason
(R). Read the statements carefully and choose the option that correctly describes
statements (A) and (R).

Assertion (A): The function f(x) = |x − 6|(cos x) is differentiable in 𝑅 − {6}.

Reason (R): If a function f is continuous at a point c, then it is also differentiable at that


point.

A. Both (A) and (R) are true and (R) is the correct explanation for (A).
B. Both (A) and (R) are true but (R) is not the correct explanation for (A).
C. (A) is true but (R) is false.
D. (A) is false but (R) is true.
SECTION B

(This section comprises of very short answer type-questions (VSA) of 2 marks each.)

2
Q.21

1
Based on the above equation, find 𝑡𝑎𝑛−1 (𝑥) using the principal values of the inverse
trigonometric functions. Show your work.

OR

i) Find the domain of the function below.

ii) Find the range (principal value branch) of the function below.

Show your work.

Q.22

𝑑𝑦 2
Q.23 Find if 𝑦 = (𝑒 𝑠𝑒𝑐𝑥 + 𝑥)4 . Show your work.
𝑑𝑥
Q.24 2

OR

In the figure below, QRST and QRTP are parallelograms.

Using the vectors shown for RQ and RS, prove that the area of QRST is equal to the area
of QRTP.

Q.25 2

SECTION C
(This section comprises of short answer type questions (SA) of 3 marks each)

Q.26 Check whether the following statement is true or false. 3

Show your work with valid reason.

OR
1 3
Q.27 The anti-derivative of a function of the form (3x − 1)f(x), (x ≠ ), is given by
3
13 3
3𝑥 4 − 𝑥3 + 𝑥 2 + 𝐶, where C is the constant of integration.
3 2

Find the value of f(6). Show your steps.

Q.28 Evaluate the following definite integral and show your work. 3

Q.29 Find the particular solution when x = y = 0 for the following differential equation. 3

Show your steps.

OR

Find the general solution of the following differential equation.

Show your steps.

Q.30 Frame the below optimisation problem as a linear programming problem and determine its 3
feasible region graphically.

Bhavani Singh, a farmer, decides to raise hens and cows to make some extra money apart
from his agricultural income. He wants to raise no more than 16 animals including no
more than 10 hens. On an average it will cost him Rs 25 and Rs 75 per day to raise one
hen and one cow respectively. He will make an average profit of Rs 12 from each hen and
Rs 40 from each cow every day. He has a budget of Rs 900 per day to raise the animals.
How many of each type of animals should he raise to maximise his profit?
Q.31 Nikhil has a bag of marbles that contains exactly 8 green marbles and 6 red marbles. He 3
takes out three marbles successively without replacing any of the marbles.

What is the probability that all three marbles taken out are green in colour? Show your
steps.

OR

In a recreational event at a school, there were 8 students, 6 parents, and 4 teachers. To play
a game, two members were selected randomly one after the other.

Find the probability distribution of the number of students. Show your steps.

SECTION D
(This section comprises of long answer-type questions (LA) of 5 marks each)

Q.32 Sravan is a nutritionist. He wants to create a mixture of orange juice, beetroot juice and 5
kiwi juice that can provide 1860 mg of vitamin C, 22 mg of iron and 760 mg of calcium.
The quantity of each nutrient per litre of juice is shown below.

Using the matrix method, find how many litres of each juice Sravan should add into the
mixture. Show your work.
2 𝑦2
Q.33 Shown below is an ellipse whose equation is 𝑥 + = 1. 5
81 36

Find the area of the shaded region in terms of π. Show your steps.

Q.34 5
OR

Q.35 Integrate the given function and show your steps. 5

OR

Evaluate the integral and show your steps.


SECTION E
(This section comprises of 3 case-study/passage-based questions of 4 marks each
with two sub-questions. First two case study questions have three sub questions of
marks 1, 1, 2 respectively. The third case study question has two sub questions of 2
marks each.)

Q.36 Answer the questions based on the given information.

Port Blair, the capital city of Andaman and Nicobar Islands is directly connected to
Chennai and Vishakapatnam via ship route. The ships sail from Chennai/Vishakapatnam
to Port Blair and vice versa.

Swaraj Dweep and Shaheed Dweep are two popular tourist islands in Andaman Islands.
One has to take a ferry from Port Blair to reach these islands. There are ferries that sail
frequently between the three islands - Port Blair (PB), Swaraj Dweep (SwD) and
Shaheed Dweep (ShD).

Shown below is a schematic representation of the ship routes and ferry routes.

(Note: The image is for representation purpose only.)

X is the set of all 5 places and Y is the set of 3 places in Andaman Islands.

That is, X = {C, V, PB, SwD, ShD} and Y = {PB, SwD, ShD}.

A relation R defined on the set X is given by, R = {(𝑥1 , 𝑥2 ): there is a direct ship or
direct ferry from 𝑥1 to 𝑥2 }.

A function 𝑓: Y → X is defined by, 𝑓(𝑃𝐵) = 𝑉, 𝑓(𝑆𝑤𝐷) = 𝑃𝐵, 𝑓(𝑆ℎ𝐷) = 𝑆𝑤𝐷.


i) List all the elements of R. 1

ii) Is the relation R symmetric? Give a valid reason. 1

iii) Is the relation R transitive? Give a valid reason. 2

OR

Check whether the function f is one-one and onto. Give valid reasons.

Q.37 Answer the questions based on the given information.

A medicinal drug administered into a human body requires some time to produce its effect
on the body. The amount (in mg) of a certain medicinal drug in the bloodstream at t hours
after administering the drug to an individual is given by the function:

C(t) = −t 3 + 4.5t 2 + 54t , 0 ≤ t ≤ 10

Shown below is the graph of C(t) in the interval [0, 10].


i) Find the rate at which the amount of drug is changing in the bloodstream at 5 1
hours after administering the drug. Show your work.
ii) Show that the function C(t) is strictly increasing in the interval (3, 4).
2
OR

The amount of the drug in the bloodstream at which the action of the drug is
maximum is denoted by Cmax.
How long after administering the drug is Cmax attained? Show your work and
give valid reasons.

iii) Find the amount of drug in the bloodstream at the time when the effect of the
drug is maximum. Show your work. 1

Q.38 Answer the questions based on the given information.

A school conducted a survey of their school staff to find their beverage preferences. Each
of them picked either tea or coffee as their first preference and then with sugar or without
sugar as their second preference as shown in the below tree diagram.

Some of the insights from the survey are given below.

♦ 60% percent of the staff prefer coffee.


♦ 90% of those who prefer coffee prefer it with sugar.
♦ 20% of those who prefer tea prefer it without sugar.

i) What is the probability that a person selected randomly from the staff prefers a
2
beverage with sugar? Show your steps.

ii) What is the probability that a person from the staff selected at random prefers
2
coffee given that it is without sugar? Show your steps.
Practice Questions - Marking Scheme
Session 2022-23
Class XII
Mathematics (Code – 041)

SECTION A - Multiple Choice Questions - 1 Mark each

Q.No. Answer/Solution Marks


Q.1 C. sec-1 x 1
Q.2 B. P and Q must be square matrices of the same order. 1
Q.3 D. all - i), ii) and iii) 1
Q.4 A. -48 1
Q.5 1 1
C. 4
Q.6 1 1
B. −𝑡𝑎𝑛 𝑥 − B, where B is a constant.
Q.7 D. 4 1
Q.8 C. 9 sq units 1
Q.9 1

B.
Q.10 4
C. 3 𝑒 3𝑥 + 1 1
Q.11 A. only ii) 1
Q.12 1
C.
Q.13 D. 0 1
Q.14 B. 60° 1
Q.15 D. 8 1
Q.16 B. It has a unique solution. 1
Q.17 D. 0.08 1
Q.18 A. Minimise Z = x + y 1
Q.19 C. (A) is true but (R) is false. 1
Q.20 C. (A) is true but (R) is false. 1

SECTION B - VSA questions of 2 marks each

Q.21 2𝜋 0.5
Solves the RHS to obtain as follows:
3

1 1
Equates the LHS to obtain x = − as follows:
√3
1 𝜋
Finds 𝑡𝑎𝑛−1 (𝑥) as − 3 as follows: 0.5

OR

2 4
i) Finds the domain as (−∞, 5] ∪ [5 , ∞)as follows: 1

ii) Finds the range as [−2, 3𝜋 − 2] as follows: 1

Q.22 1 0.5
Writes the expression for C as 2 (A − A′).

Finds A' as: 0.5


Finds C as:
1

Q.23 Differentiates y with respect to x using chain rule as: 1

𝑑𝑦 𝑑 sec 𝑥
= 4(𝑒 sec 𝑥 + 𝑥)3 (𝑒 + 𝑥)
𝑑𝑥 𝑑𝑥

Simplifies the above differential as:


1
𝑑𝑦
= 4(𝑒 sec 𝑥 + 𝑥)3 (𝑒 sec 𝑥 𝑠𝑒𝑐𝑥𝑡𝑎𝑛𝑥 + 1)
𝑑𝑥

Q.24 0.5

0.5

OR

Uses the cross-product of vectors and writes: 0.5

Uses the cross-product of vectors and writes:


0.5
Simplifies RHS of the above equation as:
1

Concludes that the area of parallelogram QRST is equal to the area of


parallelogram QRTP.

Q.25 i) Expands the vector form to get the following: 0.5

Eliminates λ by equating the like coefficients of the position vectors of the 0.5
x, y and z axes to get the cartesian equation as follows:

ii) Assumes the coordinates of B as (𝑥2 , y2 , z2 ) and compares the cartesian


form of the equation from step 2 with the regular form of the cartesian 0.5
equation to find:

𝑥2 = 2𝑥1 , 𝑦2 = 3𝑦1 and 𝑧2 = 4𝑧1

Substitutes values 𝑥1 = (-2), 𝑦1 = 5 and 𝑧1 = (-3) in the equations from step 0.5
3 to get coordinates of B as (-4, 15, -12).
SECTION C - Short Answer Questions of 3 Marks each

Q.26 𝑑𝑢 1
Finds 𝑑𝜃 as:

𝑑𝑣
Finds 𝑑𝜃 as: 1

𝑑𝑢
Uses parametric differentiation and finds 𝑑𝑣 as: 0.5

𝜋
Concludes that the given statement is true as 𝑒 2 is a constant. 0.5

OR

Rewrites the given equation by taking logarithm on both sides as: 1

m(log x) − n(log y) = (m − n)(log x + log y)

Differentiates the above equation as: 1

𝑚 𝑛 1 1
𝑑𝑥 − 𝑑𝑦 = (𝑚 − 𝑛) ( 𝑑𝑥 + 𝑑𝑦)
𝑥 𝑦 𝑥 𝑦

Rearranges the above equation to get: 0.5


𝑛 𝑚
𝑑𝑥 = 𝑑𝑦
𝑥 𝑦

𝑑𝑦 𝑛𝑦
Finds 𝑑𝑥 to be 𝑚𝑥 . 0.5

Q.27 Interprets the question statement and writes it as: 0.5

Finds the derivative in the above step as: 1

(3x − 1)f(x) = 12𝑥 3 − 13𝑥 2 + 3x


1
Factorises the above cubic polynomial as:

(3x − 1)f(x) = (3x − 1)(4𝑥 2 − 3x)

and determines the value of f(x) as (4𝑥 2 − 3x).

Substitutes x = 6 in f(x) and evaluates f(6) as 126. 0.5

Q.28 Rewrites the integral using the identity cosec2 x = 1 + cot2 x as: 0.5

Substitutes cot x = u and hence cosec2 x dx = - du in the above step and 0.5
rewrites the integral as:

Integrates the above expression as: 0.5

0.5
Substitutes cot x in place of u in the above expression to get:

92
Substitutes the limits in the above expression to get 105 .
1

Q.29 Rearranges the given differential equation as: 0.5

Finds the integrating factor as follows as the equation obtained in the above step is of the 0.5
𝑑𝑦
form 𝑑𝑥 + y P(x) = Q(x).
Finds the solution as:
1.5

Substitutes x = y = 0 in the above equation and finds the value of C as 0.


0.5
Writes the particular solution as:

OR
𝑦
Rearranges the given equation in terms of as: 0.5
𝑥

𝑑𝑦
Considers y = vx and finds 𝑑𝑥 in terms of v as:
0.5
𝑑𝑦 𝑑𝑦
= v + x
𝑑𝑥 𝑑𝑥

Equates the RHS obtained in steps 1 and 2 to get:


0.5

Rearranges the terms using the variable separable method as:


0.5

Integrates on both sides to find the general solution as:

sec −1 𝑣 = log |x| + C 1


or
𝑦
sec −1 𝑥 = log |x| + C, where C is the constant of integration.
Q.30 Takes the number of hens and cows to be x and y respectively and 1.5
formulates the linear programming problem as follows:

Maximise Z = 12x + 40y

subject to constraints,
25x + 75y ≤ 900
x + y ≤ 16
x ≤ 10
x ≥ 0
y ≥ 0

Graphs the constraints and marks the feasible region as: 1.5
Q.31 Takes E, F and G to be the events of taking out green marbles in the first, 0.5
second and third draws respectively, and writes:

8
𝑃(𝐸) = 𝑃(𝑔𝑟𝑒𝑒𝑛 𝑚𝑎𝑟𝑏𝑙𝑒 𝑖𝑛 𝑓𝑖𝑟𝑠𝑡 𝑑𝑟𝑎𝑤) =
14

Finds the probability that the second marble taken out is green provided 0.5
first is also green as:

7
𝑃(𝐹|𝐸) = 𝑃(𝑔𝑟𝑒𝑒𝑛 𝑚𝑎𝑟𝑏𝑙𝑒 𝑖𝑛 𝑡ℎ𝑒 𝑠𝑒𝑐𝑜𝑛𝑑 𝑑𝑟𝑎𝑤) =
13

Finds the probability that the third marble taken out is green provided first 0.5
two are also green as:

6
𝑃(𝐺|𝐸𝐹) = 𝑃(𝑔𝑟𝑒𝑒𝑛 𝑚𝑎𝑟𝑏𝑙𝑒 𝑖𝑛 𝑡ℎ𝑖𝑟𝑑 𝑑𝑟𝑎𝑤) =
12

Finds the probability that all three marbles taken out are green in colour as: 1.5

𝑃(𝐸) × 𝑃(𝐹|𝐸) × 𝑃(𝐺|𝐸𝐹)

8 5 6
= × ×
14 13 12
10
=
91

OR

Assumes the number of students as a random variable X and writes that it


can take values of 0, 1 and 2.
0.5
Finds P(X = 0) as:

P(non-student and non-student)

10 9
= ×
18 17
90
=
306

Finds P(X = 1) as:

P(student and non-student) or P(non-student and student)


1.5
8 10 10 8
= × × ×
18 17 18 17
160
=
306

Finds P(X = 2) as: 0.5

P(student and student)

8 7
= ×
18 17
56
=
306
Writes the required probability distribution as:
0.5
X 0 1 2

P(X) 90 160 56
306 306 306

SECTION D - Long answer type questions (LA) of 5 marks each

Q.32 Assumes the number of litres of orange juice, beetroot juice and kiwi juice 0.5
as x, y and z, respectively to frame equations as follows:

500x + 20y + 800z = 1860


2x + 5y + 3z = 22
100x + 120y + 200z = 760

Writes the above system of equations in the matrix form using AX = B as 0.5

Finds |A| = 110000 ≠ 0 and hence writes that A is non-singular and has a 0.5
unique solution.

Finds adj A as:


1
Finds A-1 using |A| and adj A as: 1

Writes that X = A-1B and finds X as 1

Concludes that 2 litres of orange juice, 3 litres of beetroot juice and 1 litre 0.5
of kiwi juice should go into the mixture.

Q.33 Writes the endpoints of the ellipse as (−9, 0), (9, 0), (0, 6) and (0, −6) 1
respectively.

Expresses y in terms of x as:

Sets up the equation for the area of the shaded region as: 1

Evaluates the 1st part of the above equation as: 1


Applies the upper and the lower limit and finds the value of the integral as: 1

0.5
Evaluates the 2nd part of the equation from step 2 as:
1
Area of 2 triangles = 2 × × 9 × 6 = 54 sq units
2

Adds the area obtained in step 3 and 4 to find the area of the shaded region in 0.5
terms of π as:

(27𝜋 + 54) sq units or 27(𝜋 + 2) sq units.

Q.34 1

Notes that the lines are skewed and writes the formula to find the shortest 1
distance between the lines (d) as follows:

1.5

10 0.5
Substitutes values from above steps to find distance as units.
√59

OR

0.5

Assumes P (x, y, z) to be the point of intersection of the two lines. Finds 0.5
x = λ + 4, y = 3λ + 2 and z = 2λ + 1.

Takes Tara’s position as T(2, -2, 1) to find the direction ratios of TP as


(𝜆 + 2), (3𝜆 + 4) and (2𝜆). 1
Notes that the dot product of the direction ratios of the given line and TP
will be 0, since they are perpendicular, and cos 90° = 0.
1.5
Writes that 1(𝜆 + 2) + 3(3𝜆 + 4) + 2(2𝜆) = 0.

Solves the above equation to find 𝜆 = (−1).

Substitutes the value of λ to find x = 3, y = 7 and z = 2. 0.5

Finds the length of TP as √83 units, using the following formula: 1

Q.35 Rewrites the given integral as: 0.5

1
Substitutes x4 = u and hence 𝑥 3 𝑑𝑥 = du in the above integral to get: 1
4

Uses integration by parts to integrate the above expression as: 1

Integrates the above expression to get: 2

Substitutes x4 in place of u in the above expression to get: 0.5

OR

Expands the denominator using the identity (a3 - b3) as: 0.5

Rewrites the integral as a sum of two integrals using partial fractions as: 1
Solves the first integral as: 0.5

Rewrites the second integral as: 1

Solves the above integral as: 1.5

Concludes the final answer as: 0.5

SECTION E - Case Studies/Passage based questions of 4 Marks each

Q.36i) Lists all the elements of R as:


1
R = {(C, PB), (PB, C), (V, PB), (PB, V), (PB, SwD), (SwD, PB),
(PB, ShD), (ShD, PB), (SwD, ShD), (ShD, SwD)}

Q.36ii) Writes that the relation R is symmetric. 0.5

Gives a reason. For example, for every (x1, x2) ∈ R, (x2, x1) ∈ R as every 0.5
direct ship/direct ferry runs in both the directions.

Q.36iii) Writes that R is not transitive. 0.5

Gives a reason. For example,


1.5
(C, PB) ∈ R as there is a direct ship from Chennai to Port Blair.

(PB, SwD) ∈ R as there is a direct ferry from Port Blair to Swaraj Dweep.

But (C, SwD) ∉ R as there is no direct ship/ferry from Chennai to Swaraj Dweep.

OR
Writes that the function f is one-one. 0.5

Gives a reason. For example, no two elements of set Y are mapped to a common 0.5
element in set X.

Writes that the function f is not onto. 0.5

Gives a reason. For example, C ∈ X (co-domain of f) but it has no pre-image in 0.5


Y.

Q.37i) Finds the rate at which the amount of drug is changing in the blood stream 5 1
hours after the drug has been administered as:

𝐶 ′ (t) = −3𝑡 2 + 9t + 54
⇒ C′(5) = 24 mg/hr

Q.37ii) Equates the derivative 𝐶 ′ (t) to 0 and factorises 𝐶 ′ (t)as 3(3 + t)(6 − t). 0.5

Writes that for t ∈ (3, 4), 1.5

3 > 0,
(3 + t) > 0
and (6 − t) > 0
Therefore, C′(t) > 0.

Concludes that C(t) is strictly increasing in the interval (3, 4).


OR
0.5
Equates the derivative 𝐶 ′ (t) = −3𝑡 2 + 9t + 54 to 0 and finds the
critical points as t = 6 hours and t = (-3) hours.
0.5
Differentiate 𝐶 ′ (t) to get C"(t) as:

C"(t) = −6t + 9

Finds C"(6) as (−27) and writes that C(t) attains its maximum at t = 6 0.5
hours, as C"(6) = (−27) < 0.

Concludes that 6 hours after the drug is administered, Cmax is attained. 0.5

1
Q.37iii) Finds the value of C(t) at t = 6 hours as:

C(6) = −(6)3 + 4.5(6)2 + 54(6)

⇒ C(6) = 270
Writes the amount of drug in the bloodstream when the effect of the drug is
maximum as 270 mg.

Q.38i) Takes P(S), P(C) and P(T) as the probabilities that a person selected 1
randomly from the staff prefers sugar, coffee and tea respectively.

Finds P(T) = P(C′) = 1 − 0.6 = 0.4.

Finds P(S|T) = 1 − 0.2 = 0.8.

Uses theorem on total probability and finds the probability that a randomly 1
selected staff prefers a beverage with sugar as:

P(S) = P(C) × P(S|C) + P(T) × P(S|T)


86 43
= 0.6 × 0.9 + 0.4 × 0.8 = 0.86 or 100 𝑜𝑟 50

Q.38ii) Uses the sum of probabilities = 1 and finds the following probabilities: 0.5

♦ P(without sugar|coffee) = 1 − 0.9 = 0.1

♦ P(tea) = 1 − 0.6 = 0.4

Uses Bayes' theorem to find the probability that a staff selected at random
prefers coffee given that it is without sugar, P(coffee|without sugar) as: 1

0.6 × 0.1
=
0.6 × 0.1 + 0.4 × 0.2

(Award 0.5 marks if only the formula for Bayes′ theorem is written correctly. )

6 3
Simplifies the above expression and finds the required probability as 14 𝑜𝑟 7.
0.5
Sample Question Paper
Class XII
Session 2022-23
Mathematics (Code-041)

Time Allowed: 3 Hours Maximum Marks: 80

General Instructions :

1. This Question paper contains - five sections A, B, C, D and E. Each section is


compulsory. However, there are internal choices in some questions.
2. Section A has 18 MCQ’s and 02 Assertion-Reason based questions of 1 mark each.
3. Section B has 5 Very Short Answer (VSA)-type questions of 2 marks each.
4. Section C has 6 Short Answer (SA)-type questions of 3 marks each.
5. Section D has 4 Long Answer (LA)-type questions of 5 marks each.
6. Section E has 3 source based/case based/passage based/integrated units of
assessment (4 marks each) with sub parts.

SECTION A
(Multiple Choice Questions)
Each question carries 1 mark

Q1. If A =[aij] is a skew-symmetric matrix of order n, then


(a) 𝑎 = ∀ 𝑖, 𝑗 (b) 𝑎 ≠ 0 ∀ 𝑖, 𝑗 (c)𝑎 = 0, 𝑤ℎ𝑒𝑟𝑒 𝑖 = 𝑗 (d) 𝑎 ≠ 0 𝑤ℎ𝑒𝑟𝑒 𝑖 = 𝑗
Q2. If A is a square matrix of order 3, |𝐴′| = −3, then |𝐴𝐴′| =
(a) 9 (b) -9 (c) 3 (d) -3
Q3. The area of a triangle with vertices A, B, C is given by
(a) 𝐴𝐵⃗ × 𝐴𝐶⃗ (b) 𝐴𝐵⃗ × 𝐴𝐶⃗
(b) 𝐴𝐶⃗ × 𝐴𝐵⃗ (d) 𝐴𝐶⃗ × 𝐴𝐵⃗
, 𝑖𝑓 𝑥 ≠ 0
Q4. The value of ‘k’ for which the function f(x) = is continuous at x = 0 is
𝑘, 𝑖𝑓 𝑥 = 0
(a) 0 (b) -1 (c) 1. (d) 2
Q5. If 𝑓 (𝑥) = 𝑥 + , then 𝑓(𝑥) is
(a) 𝑥 + log |𝑥| + 𝐶 (b) + log |𝑥| + 𝐶 (c) + log |𝑥| + 𝐶 (d) − log |𝑥| + 𝐶
Q6. If m and n, respectively, are the order and the degree of the differential equation
= 0, then m + n =

(a) 1 (b) 2 (c) 3 (d) 4


Q7. The solution set of the inequality 3x + 5y < 4 is

(a) an open half-plane not containing the origin.


(b) an open half-plane containing the origin.
(c) the whole XY-plane not containing the line 3x + 5y = 4.
(d) a closed half plane containing the origin.

Page 1
Q8. The scalar projection of the vector 3𝚤̂ − 𝚥̂ − 2𝑘 𝑜𝑛 𝑡ℎ𝑒 𝑣𝑒𝑐𝑡𝑜𝑟 𝚤̂ + 2𝚥̂ − 3𝑘 is
(a) (b) (c) (d)

Q9. The value of ∫ dx is


(a) log4 (b) 𝑙𝑜𝑔 (c) 𝑙𝑜𝑔2 (d) 𝑙𝑜𝑔

Q10. If A, B are non-singular square matrices of the same order, then (𝐴𝐵 ) =
(a)𝐴 𝐵 (b)𝐴 𝐵 (c)𝐵𝐴 (d) 𝐴𝐵

Q11. The corner points of the shaded unbounded feasible region of an LPP are (0, 4),
(0.6, 1.6) and (3, 0) as shown in the figure. The minimum value of the objective
function Z = 4x + 6y occurs at

(a)(0.6, 1.6) 𝑜𝑛𝑙𝑦 (b) (3, 0) only (c) (0.6, 1.6) and (3, 0) only
(d) at every point of the line-segment joining the points (0.6, 1.6) and (3, 0)

2 4 2𝑥 4
Q12. If = , 𝑡ℎ𝑒𝑛 𝑡ℎ𝑒 𝑝𝑜𝑠𝑠𝑖𝑏𝑙𝑒 value(s) of ‘x’ is/are
5 1 6 𝑥
(a) 3 (b) √3 (c) -√3 (d) √3, −√3

Q13. If A is a square matrix of order 3 and |A| = 5, then |𝑎𝑑𝑗𝐴| =


(a) 5 (b) 25 (c) 125 (d)

Q14. Given two independent events A and B such that P(A) =0.3, P(B) = 0.6 and P(𝐴 ∩ 𝐵 ) is
(a) 0.9 (b) 0.18 (c) 0.28 (d) 0.1

Q15. The general solution of the differential equation 𝑦𝑑𝑥 − 𝑥𝑑𝑦 = 0 𝑖𝑠


(a) 𝑥𝑦 = 𝐶 (b) 𝑥 = 𝐶𝑦 (c) 𝑦 = 𝐶𝑥 (d) 𝑦 = 𝐶𝑥

Q16. If 𝑦 = 𝑠𝑖𝑛 𝑥, then (1 − 𝑥 )𝑦 𝑖𝑠 equal to


(a) 𝑥𝑦 (b) 𝑥𝑦 (c) 𝑥𝑦 (d) 𝑥

Page 2
Q17. If two vectors 𝑎⃗ 𝑎𝑛𝑑 𝑏⃗ are such that |𝑎⃗| = 2 , 𝑏⃗ = 3 𝑎𝑛𝑑 𝑎⃗. 𝑏⃗ = 4, 𝑡ℎ𝑒𝑛 𝑎⃗ − 2𝑏⃗ is
equal to
(a) √2 (b) 2√6 (c) 24 (d) 2√2

Q18. P is a point on the line joining the points 𝐴(0,5, −2) and 𝐵(3, −1,2). If the x-coordinate
of P is 6, then its z-coordinate is

(a) 10 (b) 6 (c) -6 (d) -10

ASSERTION-REASON BASED QUESTIONS


In the following questions, a statement of assertion (A) is followed by a statement of
Reason (R). Choose the correct answer out of the following choices.

(a) Both A and R are true and R is the correct explanation of A.


(b) Both A and R are true but R is not the correct explanation of A.
(c) A is true but R is false.
(d) A is false but R is true.

Q19. Assertion (A): The domain of the function 𝑠𝑒𝑐 2𝑥 is −∞, − ∪ [ , ∞)


Reason (R): 𝑠𝑒𝑐 (−2) = −
Q20. Assertion (A): The acute angle between the line 𝑟̅ = 𝚤̂ + 𝚥̂ + 2𝑘 + 𝜆(𝚤̂ − 𝚥̂) and the x-axis
is
Reason(R): The acute angle 𝜃 between the lines
𝑟̅ = 𝑥 𝚤̂ + 𝑦 𝚥̂ + 𝑧 𝑘 + 𝜆 𝑎 𝚤̂ + 𝑏 𝚥̂ + 𝑐 𝑘 and
| |
𝑟̅ = 𝑥 𝚤̂ + 𝑦 𝚥̂ + 𝑧 𝑘 + 𝜇 𝑎 𝚤̂ + 𝑏 𝚥̂ + 𝑐 𝑘 is given by 𝑐𝑜𝑠𝜃 =

SECTION B
This section comprises of very short answer type-questions (VSA) of 2 marks each

Q21. Find the value of 𝑠𝑖𝑛 [𝑠𝑖𝑛 ]


OR
Prove that the function f is surjective, where 𝑓: 𝑁 → 𝑁 such that
𝑛+1
, 𝑖𝑓 𝑛 𝑖𝑠 𝑜𝑑𝑑
𝑓(𝑛) = 2
𝑛
, 𝑖𝑓 𝑛 𝑖𝑠 𝑒𝑣𝑒𝑛
2
Is the function injective? Justify your answer.

Q22. A man 1.6 m tall walks at the rate of 0.3 m/sec away from a street light that is 4 m above
the ground. At what rate is the tip of his shadow moving? At what rate is his shadow
lengthening?

Q23. If 𝑎⃗ = 𝚤̂ − 𝚥̂ + 7𝑘 𝑎𝑛𝑑 𝑏⃗ = 5𝚤̂ − 𝚥̂ + 𝜆𝑘, then find the value of 𝜆 so that the vectors
𝑎⃗ + 𝑏⃗ 𝑎𝑛𝑑 𝑎⃗ − 𝑏⃗ are orthogonal.
𝑶𝑹

Page 3
Find the direction ratio and direction cosines of a line parallel to the line whose equations
are
6𝑥 − 12 = 3𝑦 + 9 = 2𝑧 − 2
Q24. If 𝑦√1 − 𝑥 + 𝑥 1 − 𝑦 = 1 , 𝑡ℎ𝑒𝑛 𝑝𝑟𝑜𝑣𝑒 𝑡ℎ𝑎𝑡 = −

Q25. Find |𝑥⃗| if (𝑥⃗ − 𝑎⃗). (𝑥⃗ + 𝑎⃗) = 12, where 𝑎⃗ is a unit vector.

SECTION C
(This section comprises of short answer type questions (SA) of 3 marks each)

Q26. Find: ∫

Q27. Three friends go for coffee. They decide who will pay the bill, by each tossing a coin and
then letting the “odd person” pay. There is no odd person if all three tosses produce the
same result. If there is no odd person in the first round, they make a second round of
tosses and they continue to do so until there is an odd person. What is the probability
that exactly three rounds of tosses are made?
OR
Find the mean number of defective items in a sample of two items drawn one-by-one
without replacement from an urn containing 6 items, which include 2 defective items.
Assume that the items are identical in shape and size.
Q28. Evaluate: ∫

OR

Evaluate: ∫ |𝑥 − 1| 𝑑𝑥

Q29. Solve the differential equation: 𝑦𝑑𝑥 + (𝑥 − 𝑦 )𝑑𝑦 = 0

OR
Solve the differential equation: 𝑥𝑑𝑦 − 𝑦𝑑𝑥 = 𝑥 + 𝑦 𝑑𝑥

Q30. Solve the following Linear Programming Problem graphically:

Maximize Z = 400x + 300y subject to 𝑥 + 𝑦 ≤ 200, 𝑥 ≤ 40, 𝑥 ≥ 20, 𝑦 ≥ 0

Q31. Find ∫ ( )
𝑑𝑥
SECTION D
(This section comprises of long answer-type questions (LA) of 5 marks each)

Q32. Make a rough sketch of the region {(𝑥, 𝑦): 0 ≤ 𝑦 ≤ 𝑥 , 0 ≤ 𝑦 ≤ 𝑥, 0 ≤ 𝑥 ≤ 2} and find
the area of the region using integration.
Q33. Define the relation R in the set 𝑁 × 𝑁 as follows:
For (a, b), (c, d) ∈ 𝑁 × 𝑁, (a, b) R (c, d) iff ad = bc. Prove that R is an equivalence
relation in 𝑁 × 𝑁.
OR

Page 4
Given a non-empty
empty set X, define the relation R in P(X) as follows:
For A, B ∈ 𝑃(𝑋), (𝐴, 𝐵) ∈ 𝑅 iff 𝐴 ⊂ 𝐵. Prove that R is reflexive, transitive and not
symmetric.

Q34. An insect is crawling along the line 𝑟̅ = 6𝚤̂ + 2𝚥̂ + 2𝑘 + 𝜆 𝚤̂ − 2𝚥̂ + 2𝑘 and another
insect is crawling along the line 𝑟̅ = −4𝚤̂ − 𝑘 + 𝜇 3𝚤̂ − 2𝚥̂ − 2𝑘 . At what points on the
lines should they reach so that the distance between them is the shortest? Find the shortest
possible distance between them.

OR
The equations of motion of a rocket are:
ar
𝑥 = 2𝑡, 𝑦 = −4𝑡, 𝑧 = 4𝑡, where the time t is given in seconds, and the coordinates of a
moving point in km. What is the path of the rocket? At what distances will the rocket be
from the starting point O(0,
(0, 0, 0) and from the following line in 10 seconds?
𝑟⃗ = 20𝚤̂ − 10𝚥̂ + 40𝑘 + 𝜇((10𝚤̂ − 20𝚥̂ + 10𝑘 )

2 −3 5
Q35. If A = 3 2 −4 , find 𝐴 . Use 𝐴 to solve the following system of equations
1 1 −2
2𝑥 − 3𝑦 + 5𝑧 = 11, 3𝑥 + 2
2𝑦 − 4𝑧 = −5, 𝑥 + 𝑦 − 2𝑧 = −3

SECTION E
(This
This section comprises of 3 case-study/passage-based questions of 4 marks each
with two sub-parts. First two case study questions have three sub-parts
sub parts (i), (ii), (iii)
of marks 1, 1, 2 respectively. The third case study question has two sub
sub-parts of 2
marks each.)

Q36. Case-Study 1: Read the following passage and answer the questions given below.

The temperature of a person during an intestinal illness is given by


𝑓(𝑥) = −0.1𝑥 + 𝑚𝑥 + 98 98.6,0 ≤ 𝑥 ≤ 12, m being a constant, where f(x) is the
temperature in °F at x days.
(i) Is the function differentiable in the interval (0, 12)? Justify your answer.
(ii) If 6 is the critical point of thee function, then find the value of the constant m.
(iii) Find the intervals in which the function is strictly increasing/strictly decreasing.
OR
(iii) Find the points of local maximum/local minimum, if any, in the interval (0, 12) as
well as the points of absolute maximum/absolute minimum in the interval [0, 12].
Also, find the corresponding local maximum/local minimum and the absolute
maximum/absolute minimum values of the function.

Q37. Case-Study 2: Read the following passage and answer the questions given below.

In an elliptical sport field the authority wants to design a rectangular soccer field
with the maximum possible area. The sport field is given by the graph of
+ = 1.
(i) If the length and the breadth of the rectangular field be 2x and 2y respectively,
then find the area function in terms of x.
(ii) Find the critical point of the function.
(iii) Use First derivative Test to find the length 2x and width 2y of the soccer field (in
terms of a and b) that maximize its area.
OR
(iii) Use Second Derivative Test to find the length 2x and width 2y of the soccer field
(in terms of a and b) that maximize its area.

Page 6
Q38. Case-Study 3: Read the following passage and answer the questions given below.

There are two antiaircraft guns, named as A and B. The probabilities that the shell fired
from them hits an airplane are 0.3 and 0.2 respectively. Both of them fired one shell at an
airplane at the same time.
(i) What is the probability that the shell fired from exactly one of them hit the plane?
(ii) If it is known that the shell fired from exactly one of them hit the plane, then what is
the probability that it was fired from B?

Page 7
Marking Scheme
Class XII
Mathematics (Code – 041)
Section : A (Multiple Choice Questions- 1 Mark each)
Question Answer Hints/Solution
No
1. (c) In a skew-symmetric matrix, the (i, j)th element is negative of
the (j, i)th element. Hence, the (i, i)th element = 0
2. (a) |𝐴𝐴 | = |𝐴||𝐴′| = (−3)(−3) = 9
3. (b) The area of the parallelogram with adjacent sides AB and AC =
𝐴𝐵⃗ × 𝐴𝐶⃗ . Hence, the area of the triangle with vertices A, B, C
= 𝐴𝐵⃗ × 𝐴𝐶⃗
4. (c) The function f is continuous at x = 0 if 𝑙𝑖𝑚 → 𝑓(𝑥) = 𝑓(0)
We have f(0) = k and
𝑙𝑖𝑚 → 𝑓(𝑥) = 𝑙𝑖𝑚 → = 𝑙𝑖𝑚 → =𝑙𝑖𝑚 →

=𝑙𝑖𝑚 → =1
Hence, k =1
5. (b) 𝑥 1
+ log |𝑥| + 𝐶 ∵ 𝑓(𝑥) = 𝑥+ 𝑑𝑥
2 𝑥
6. (c)
The given differential equation is 4 = 0. Here, m = 2
and n = 1
Hence, m + n= 3
7. (b) The strict inequality represents an open half plane and it
contains the origin as (0, 0) satisfies it.
8. (a) Scalar Projection of 3𝚤̂ − 𝚥̂ − 2𝑘 𝑜𝑛 𝑣𝑒𝑐𝑡𝑜𝑟 𝚤̂ + 2𝚥̂ − 3𝑘
̂ ̂ . ̂ ̂
= ̂ ̂
=

9. (c) ∫ = [𝑙𝑜𝑔(𝑥 + 1)] = (𝑙𝑜𝑔10 − 𝑙𝑜𝑔5) = 𝑙𝑜𝑔
= 𝑙𝑜𝑔2
10. (c) (𝐴𝐵 ) = (𝐵 ) 𝐴 = 𝐵𝐴
11. (d) The minimum value of the objective function occurs at two
adjacent corner points (0.6, 1.6) and (3, 0) and there is no point
in the half plane 4𝑥 + 6𝑦 < 12 in common with the feasible
region. So, the minimum value occurs at every point of the line-
segment joining the two points.
12. (d) 2 − 20 = 2𝑥 − 24 ⟹ 2𝑥 = 6 ⟹ 𝑥 = 3 ⟹ 𝑥 = ±√3
13. (b) |𝑎𝑑𝑗𝐴| = |𝐴| ⟹ |𝑎𝑑𝑗𝐴| = 25
14. (c) P 𝐴 ∩ 𝐵 = 𝑃 𝐴′) × 𝑃(𝐵′) (As A and B are independent,
( ) (
𝐴 𝑎𝑛𝑑 𝐵′ are also independent.)
= 0.7 × 0.4 = 0.28
15. (c) 𝑑𝑦 𝑑𝑥
𝑦𝑑𝑥 − 𝑥𝑑𝑦 = 0 ⟹ 𝑦𝑑𝑥 − 𝑥𝑑𝑦 = 0 ⟹ =
𝑦 𝑥
𝑑𝑦 𝑑𝑥
⟹ = + 𝑙𝑜𝑔𝐾, 𝐾 > 0 ⟹ log|𝑦| = log|𝑥| + log 𝐾
𝑦 𝑥
⟹ log|𝑦| = log |𝑥|𝐾 ⟹ |𝑦| = |𝑥|𝐾 ⟹ y=±𝐾𝑥 ⟹ 𝑦 = 𝐶𝑥

Page 1
16. (a) y = sin-1x
𝑑𝑦 1 𝑑𝑦
= ⟹ 1−𝑥 . =1
𝑑𝑥 √1 − 𝑥 𝑑𝑥
Again , differentiating both sides w. r. to x, we get
𝑑 𝑦 𝑑𝑦 −2𝑥
1−𝑥 + . =0
𝑑𝑥 𝑑𝑥 2√1 − 𝑥
Simplifying, we get (1 − 𝑥 )𝑦 = 𝑥𝑦
17. (b) 𝑎⃗ − 2𝑏⃗ = 𝑎⃗ − 2𝑏⃗ . 𝑎⃗ − 2𝑏⃗
𝑎⃗ − 2𝑏⃗ = 𝑎⃗. 𝑎⃗ − 4𝑎⃗. 𝑏⃗ + 4𝑏⃗ . 𝑏⃗
= |𝑎⃗| − 4𝑎⃗. 𝑏⃗+ 4 𝑏⃗
= 4 − 16 + 36 = 24
𝑎⃗ − 2𝑏⃗ = 24 ⟹ 𝑎⃗ − 2𝑏⃗ = 2√6
18. (b) The line through the points (0, 5, -2) and (3, -1, 2) is
= =
𝑥 𝑦−5 𝑧+2
𝑜𝑟, = =
3 −6 4
Any point on the line is (3𝑘, −6𝑘 + 5,4𝑘 − 2), where k is an
arbitrary scalar.
3𝑘 = 6 ⟹ 𝑘 = 2
The z-coordinate of the point P will be 4 × 2 − 2 = 6
19. (c) 𝑠𝑒𝑐 𝑥 is defined if 𝑥 ≤ −1 𝑜𝑟 𝑥 ≥ 1. Hence, 𝑠𝑒𝑐 2𝑥 will be
defined if 𝑥 ≤ − or 𝑥 ≥ .
Hence, A is true.
The range of the function 𝑠𝑒𝑐 𝑥 is [0, 𝜋] − { }
R is false.
20. (a) The equation of the x-axis may be written as 𝑟⃗ = 𝑡𝚤̂. Hence, the
acute angle 𝜃 between the given line and the x-axis is given by
|1 × 1 + (−1) × 0 + 0 × 0| 1 𝜋
𝑐𝑜𝑠𝜃 = = ⟹𝜃=
1 + (−1) + 0 × √1 + 0 + 0 √2 4

SECTION B (VSA questions of 2 marks each)


21. 13𝜋 𝜋 .1
𝑠𝑖𝑛 [𝑠𝑖𝑛 ] = 𝑠𝑖𝑛 [𝑠𝑖𝑛 2𝜋 − ]
7 7
= 𝑠𝑖𝑛 [𝑠𝑖𝑛 − ] = − 1
OR
Let 𝑦 ∈ 𝑁(codomain). Then ∃ 2𝑦 ∈ 𝑁(domain) such that
𝑓(2𝑦) = = 𝑦. Hence, f is surjective. 1
1, 2 ∈ 𝑁(domain) such that 𝑓(1) = 1 = 𝑓(2)
Hence, f is not injective. 1
22. Let AB represent the height of the street light from the ground. At
any time t seconds, let the man represented as ED of height 1.6 m
be at a distance of x m from AB and the length of his shadow EC
be y m.
Using similarity of triangles, we have = ⇒ 3𝑦 = 2𝑥 ½
.

Page 2
Differentiating both sides w.r.to t, we get 3 = 2
𝑑𝑦 2 𝑑𝑦
= × 0.3 ⇒ = 0.2 ½
𝑑𝑡 3 𝑑𝑡
At any time t seconds, the tip of his shadow is at a distance of
(𝑥 + 𝑦) m from AB.
The rate at which the tip of his shadow moving
𝑑𝑥 𝑑𝑦
= + 𝑚/𝑠 = 0.5 𝑚/𝑠 ½
𝑑𝑡 𝑑𝑡
The rate at which his shadow is lengthening
𝑑𝑦
= 𝑚/𝑠 = 0.2 𝑚/𝑠 1/2
𝑑𝑡
23. 𝑎⃗ = 𝚤̂ − 𝚥̂ + 7𝑘 𝑎𝑛𝑑 𝑏⃗ = 5𝚤̂ − 𝚥̂ + 𝜆𝑘
Hence 𝑎⃗ + 𝑏⃗ = 6 𝚤̂ − 2𝚥̂ + (7 + 𝜆)𝑘 and 𝑎⃗ − 𝑏⃗ = −4𝚤̂ + (7 − 𝜆)𝑘
𝑘 ½
⃗ ⃗ ⃗ ⃗
𝑎⃗ + 𝑏 𝑎𝑛𝑑 𝑎⃗ − 𝑏 will be orthogonal if, 𝑎⃗ + 𝑏 . 𝑎⃗ − 𝑏 = 0 ½
i.e., if, −24 + (49 − 𝜆 ) = 0 ⟹ 𝜆 = 25
i.e., if, 𝜆 = ±5 1

OR
The equations of the line are 6𝑥 − 12 = 3𝑦 + 9 = 2𝑧 − 2, which,
when written in standard symmetric form, will be
( )
= = ½

Since, lines are parallel, we have = =


Hence, the required direction ratios are , , or (1,2,3)) ½
and the required direction cosines are , , 1
√ √ √

24. 𝑦 1−𝑥 +𝑥 1−𝑦 = 1


Let 𝑠𝑖𝑛 𝑥 = 𝐴 and 𝑠𝑖𝑛 𝑦 = 𝐵.. Then x = sinA and y = sinB ½
𝑦 1 − 𝑥 + 𝑥 1 − 𝑦 = 1 ⟹ 𝑠𝑖𝑛𝐵𝑐𝑜𝑠𝐴 + 𝑠𝑖𝑛𝐴𝑐𝑜𝑠𝐵 = 1

𝜋
⟹ sin(𝐴 + 𝐵 ) = 1 ⟹ 𝐴 + 𝐵 = 𝑠𝑖𝑛 1 =
2
𝜋
⟹ 𝑠𝑖𝑛 𝑥 + 𝑠𝑖𝑛 𝑦 = ½
2
Differentiating w.r.to x, we obtain = − 1
25. Since 𝒂⃗ is a unit ve
vector, ∴ |𝑎⃗| = 1 ½
(𝑥⃗ − 𝑎⃗). (𝑥⃗ + 𝑎⃗) = 12.

⟹ 𝑥⃗. 𝑥⃗ + 𝑥⃗. 𝑎⃗ − 𝑎⃗. 𝑥⃗ − 𝑎⃗. 𝑎⃗ = 12 ½

⟹ |𝑥⃗| − |𝑎⃗| = 12. ½


⟹ |𝑥⃗| − 1 = 12
⟹ |𝑥⃗| = 13 ⟹ |𝑥⃗| = √13 1/2
SECTION C
(Short Answer Questions of 3 Marks each)

26. 𝑑𝑥
√3 − 2𝑥 − 𝑥
2
=∫ =∫
( ) ( )

= 𝑠𝑖𝑛 +𝐶 [∫ = 𝑠𝑖𝑛 +𝐶] 1


27. P(not obtaining an odd person in a single round) = P(All three of


them throw tails or All three of them throw heads)
= × × ×2= 1+1/2
P(obtaining an odd person in a single round)
= 1 − P(not obtaining an odd person in a single round) = ½
The required probability
= P(‘In first round there is no odd person’ and ‘In second round
there is no odd person’ and ‘In third round there is an odd person’)
= × × = 1
OR
Let X denote the Random Variable defined by the number of
defective items.

P(X=0) = × =

P(X=1) = 2 × × =

2
P(X=2) = × =
𝑥 0 1 2
𝑝 2 8 1
5 15 15
𝑝𝑥 0 8 2 1/2
15 15

Mean = ∑ 𝑝 𝑥 = = 1/2
28. Let I = ∫ =∫

𝑑𝑥 ..(i)
√ √ √

Page 4
𝒃 𝒃
Using ∫𝒂 𝒇(𝒙) 𝒅𝒙 = ∫𝒂 𝒇(𝒂 + 𝒃 − 𝒙)𝒅𝒙

I=∫ 𝑑𝑥


I=∫ dx ..(ii). 1
√ √
Adding (i) and (ii), we get
√ √
2I = ∫ 𝑑𝑥 + ∫ dx
√ √ √ √

1
2I = ∫ 𝑑𝑥

= [𝑥] = − =

Hence, I = ∫ = 1

OR
|𝑥 − 1| 𝑑𝑥 = (1 − 𝑥 )𝑑𝑥 + (𝑥 − 1)𝑑𝑥 1

= 𝑥−
1
+ −𝑥
4 1
0 1
= (1 − ) + (8 − 4) − ( − 1)
=5 1
29. 𝑦𝑑𝑥 + (𝑥 − 𝑦 )𝑑𝑦 = 0
𝒅𝒙
Reducing the given differential equation to the form + 𝑷𝒙 = 𝑸
𝒅𝒚
we get, + =𝑦 ½

I.F = 𝑒 ∫ =𝑒 =𝑒 =𝑦 1
The general solution is given by
𝑥. 𝐼𝐹 = 𝑄. 𝐼𝐹𝑑𝑦 ⟹ 𝑥𝑦 = 𝑦 𝑑𝑦 1

⟹ 𝑥𝑦 = + 𝐶 , which is the required general solution ½


OR
𝑥𝑑𝑦 − 𝑦𝑑𝑥 = 𝑥 + 𝑦 𝑑𝑥
It is a Homogeneous Equation as
𝑑𝑦 𝑥 +𝑦 +𝑦 𝑦 𝑦 𝑦
= = 1+( ) + = 𝑓 .
𝑑𝑥 𝑥 𝑥 𝑥 𝑥 ½
Put 𝑦 = 𝑣𝑥

= 𝑣+𝑥

Page 5
𝑑𝑣 ½
𝑣+𝑥 = 1+𝑣 +𝑣
𝑑𝑥
Separating variables, we get
𝑑𝑣 𝑑𝑥 ½
=
√1 + 𝑣 𝑥
Integrating, we get 𝑙𝑜𝑔 𝑣 + √1 + 𝑣 = 𝑙𝑜𝑔|𝑥| + 𝑙𝑜𝑔𝐾, 𝐾 > 0
𝑙𝑜𝑔 𝑦 + 𝑥 + 𝑦 = 𝑙𝑜𝑔𝑥 𝐾
⇒ 𝑦 + 𝑥 + 𝑦 = ±𝐾𝑥
⇒ 𝑦 + 𝑥 + 𝑦 = 𝐶𝑥 , which is the required general solution
1+1/2
30. We have Z= 400x +300y subject to
x + y ≤ 200,𝑥 ≤ 40, 𝑥 ≥ 20, 𝑦 ≥ 0
The corner points of the feasible region are C(20,0), D(40,0),
B(40,160), A(20,180)

Corner Point Z = 400x + 300y


C(20,0) 8000
D(40,0) 16000
B(40,160) 64000
A(20,180) 62000 1

Maximum profit occurs at x= 40, y=160


and the maximum profit =₹ 64,000 1
31. 𝑑𝑥 = ∫ 𝑥 + 𝑑𝑥 1
∫ ( ) ( )( )
Now resolving into partial fractions as
( )( )

2𝑥 + 1 𝐴 𝐵
= +
(𝑥 − 1)(𝑥 + 1) 𝑥−1 𝑥+1

We get = + 1
( )( ) ( ) ( )

Page 6
Hence, ∫ ( )
𝑑𝑥 = ∫ 𝑥 + 𝑑𝑥
( )( )

=∫ 𝑥+ + 𝑑𝑥
( ) ( )

= + log|𝑥 − 1| + log|𝑥 + 1| + 𝐶
𝑥 1
= + (log|(𝑥 − 1) (𝑥 + 1)| + 𝐶 1
2 2

SECTION D
(Long answer type questions (LA) of 5 marks each)

32.

(Correct
Fig: 1
Mark)

The points of intersection of the parabola 𝑦 = 𝑥 and the line


½
𝑦 = 𝑥 are (0, 0) and (1, 1).
Required Area = ∫ 𝑦 𝑑𝑥 + ∫ 𝑦 𝑑𝑥
Required Area = ∫ 𝑥 𝑑𝑥 + ∫ 𝑥 𝑑𝑥 2
1 2
= + = + = 1+1/2
0 1

33. Let (𝑎, 𝑏) ∈ 𝑁 × 𝑁. Then we have


ab = ba (by commutative property of multiplication of natural
numbers)
⟹ (𝑎, 𝑏)𝑅 (𝑎, 𝑏)
Hence, R is reflexive. 1
Let (𝑎, 𝑏), (𝑐, 𝑑) ∈ 𝑁 × 𝑁 such that (a, b) R (c, d). Then
ad = bc
⟹ 𝑐𝑏 = 𝑑𝑎 (by commutative property of multiplication of
natural numbers
⟹ (𝑐, 𝑑)𝑅(𝑎, 𝑏)
Hence, R is symmetric. 1+1/2
Let (𝑎, 𝑏), (𝑐, 𝑑), (𝑒, 𝑓) ∈ 𝑁 × 𝑁 such that

Page 7
(a, b) R (c, d) and (c, d) R (e, f).
Then ad = bc, cf = de
⟹ 𝑎𝑑𝑐𝑓 = 𝑏𝑐𝑑𝑒
⟹ 𝑎𝑓 = 𝑏𝑒
⟹ (𝑎, 𝑏)𝑅(𝑒, 𝑓)
Hence, R is transitive. 2
Since, R is reflexive, symmetric and transitive, R is an
equivalence relation on 𝑁 × 𝑁. ½
OR
Let 𝐴 ∈ 𝑃 𝑋 . Then 𝐴 ⊂ 𝐴
( )
⟹ (𝐴, 𝐴) ∈ 𝑅
Hence, R is reflexive. 1
Let 𝐴, 𝐵, 𝐶 ∈ 𝑃(𝑋) such that
(𝐴, 𝐵), (𝐵, 𝐶) ∈ 𝑅
⟹ 𝐴 ⊂ 𝐵, 𝐵 ⊂ 𝐶
⟹𝐴⊂𝐶
⟹ (𝐴, 𝐶) ∈ 𝑅
Hence, R is transitive. 2
∅, 𝑋 ∈ 𝑃(𝑋) such that ∅ ⊂ 𝑋. Hence, (∅, 𝑋) ∈ 𝑅. But, 𝑋 ⊄ ∅,
which implies that (𝑋, ∅) ∉ 𝑅.
Thus, R is not symmetric. 2

34. The given lines are non-parallel lines. There is a unique line-
segment PQ (P lying on one and Q on the other, which is at right
angles to both the lines. PQ is the shortest distance between the
lines. Hence, the shortest possible distance between the insects =
PQ
The position vector of P lying on the line
𝑟⃗ = 6𝚤̂ + 2𝚥̂ + 2𝑘 + 𝜆 𝚤̂ − 2𝚥̂ + 2𝑘
is (6 + 𝜆)𝚤̂ + (2 − 2𝜆)𝚥̂ + (2 + 2𝜆)𝑘 for some 𝜆 ½
The position vector of Q lying on the line
𝑟⃗ = −4𝚤̂ − 𝑘 + 𝜇 3𝚤̂ − 2𝚥̂ − 2𝑘
½
is (−4 + 3𝜇)𝚤̂ + (−2𝜇)𝚥̂ + (−1 − 2𝜇)𝑘 for some 𝜇
𝑃𝑄⃗ = (−10 + 3𝜇 − 𝜆)𝚤̂ + (−2𝜇 − 2 + 2𝜆)𝚥̂ + (−3 − 2𝜇 − 2𝜆)𝑘 ½
Since, PQ is perpendicular to both the lines
(−10 + 3𝜇 − 𝜆) + (−2𝜇 − 2 + 2𝜆)(−2) + (−3 − 2𝜇 − 2𝜆)2
= 0, ½
𝑖. 𝑒. , 𝜇 − 3𝜆 = 4 …(i)
and (−10 + 3𝜇 − 𝜆)3 + (−2𝜇 − 2 + 2𝜆)(−2) + (−3 − 2𝜇 −
2𝜆)(−2) = 0, ½
𝑖. 𝑒. ,17𝜇 − 3𝜆 = 20 …(ii)
solving (i) and (ii) for 𝜆 𝑎𝑛𝑑 𝜇, we get 𝜇 = 1, 𝜆 = −1. 1
The position vector of the points, at which they should be so that
the distance between them is the shortest, are
5𝚤̂ + 4𝚥̂ and −𝚤̂ − 2𝚥̂ − 3𝑘 ½
𝑃𝑄⃗ = −6𝚤̂ − 6𝚥̂ − 3𝑘
The shortest distance = 𝑃𝑄⃗ = √6 + 6 + 3 = 9 1
OR

Page 8
Eliminating t between the equations, we obtain the equation of the
path = = , which are the equations of the line passing
through the origin having direction ratios <2, -4, 4>. This line is
the path of the rocket. 1
When t = 10 seconds, the rocket will be at the point (20, -40, 40).
Hence, the required distance from the origin at 10 seconds = ½
20 + 40 + 40 𝑘𝑚 = 20 × 3 𝑘𝑚 = 60 𝑘𝑚 1
The distance of the point (20, -40, 40) from the given line
( ⃗ ⃗)× ⃗ ̂ ×( ̂ ̂ ) ̂
= = 𝑘𝑚 = 𝑘𝑚 2
⃗ ̂ ̂ ̂ ̂

= 𝑘𝑚 = 10√3 𝑘𝑚 1/2

35. 2 −3 5
A = 3 2 −4
1 1 −2
½
|A| = 2(0) + 3(−2) + 5(1) = −1
𝑎𝑑𝑗𝐴
𝐴 =
|𝐴|
0 −1 2 0 −1 2
𝑎𝑑𝑗𝐴 = 2 −9 23 , 𝐴 = 2 −9 23 3
( )
1 −5 13 1 −5 13
𝑥 0 −1 2 11
X=𝐴 𝐵⟹ 𝑦 = 2 −9 23 −5
( )
𝑧 1 −5 13 −3
0+5−6
= 22 + 45 − 69
( )
11 + 25 − 39
𝑥 1 −1
⟹ 𝑦 = −2 ⟹ 𝑥 = 1, 𝑦 = 2 , 𝑧 = 3. 1+1/2
𝑧 (−1)
−3

SECTION E(Case Studies/Passage based questions of 4 Marks each)


36. (i) f(𝑥 ) = −0.1𝑥 + 𝑚𝑥 + 98.6, being a polynomial function, is differentiable
everywhere, hence, differentiable in (0, 12) 1
(ii)𝑓 (𝑥 ) = − 0.2𝑥 + 𝑚
Since, 6 is the critical point,
𝑓 (6) = 0 ⟹ 𝑚 = 1.2 1

(iii) 𝑓(𝑥) = −0.1𝑥 + 1.2𝑥 + 98.6

𝑓 (𝑥 ) = − 0.2𝑥 + 1.2 = −0.2(𝑥 − 6)

In the Interval 𝒇 (𝒙) Conclusion


(0, 6) +ve f is strictly increasing
in [0, 6]
(6, 12) -ve f is strictly decreasing 1+1
in [6, 12]

Page 9
OR
(iii) 𝑓(𝑥) = −0.1𝑥 + 1.2𝑥 + 98.6,
𝑓 (𝑥) = − 0.2𝑥 + 1.2, 𝑓 (6) = 0,
𝑓 ′(𝑥) = − 0.2
𝑓 (6) = − 0.2 < 0
Hence, by second derivative test 6 is a point of local maximum. The local
maximum value = 𝑓(6) = − 0.1 × 6 + 1.2 × 6 + 98.6 = 102.2 1
We have 𝑓(0) = 98.6, 𝑓(6) = 102.2, 𝑓(12) = 98.6
6 is the point of absolute maximum and the absolute maximum value of the
function = 102.2. 1/2
0 and 12 both are the points of absolute minimum and the absolute minimum value
of the function = 98.6. 1/2
37. (i)

Let (𝑥, 𝑦) = 𝑥, √𝑎 − 𝑥 be the upper right vertex of the rectangle.


The area function 𝐴 = 2𝑥 × 2 √𝑎 − 𝑥
= 𝑥√𝑎 − 𝑥 , 𝑥 ∈ (0, 𝑎). 1
(ii) = 𝑥× + √𝑎 − 𝑥

4𝑏 𝑎 − 2𝑥 4𝑏 2 𝑥 + √ (𝑥 − √ )
= × =− × ½
𝑎 √𝑎 − 𝑥 𝑎 √𝑎 − 𝑥
=0⇒𝑥= .

𝑥= is the critical point. 1/2

(iii)For the values of x less than and close to , >0
√ √
1
and for the values of x greater than and close to , < 0.
√ √
Hence, by the first derivative test, there is a local maximum at the critical point
𝑥 = . Since there is only one critical point, therefore, the area of the soccer field

is maximum at this critical point 𝑥 = 1/2

Thus, for maximum area of the soccer field, its length should be 𝑎√2 and its width
should be 𝑏√2. ½
OR

Page
10
(iii) 𝐴 = 2𝑥 × 2 √𝑎 − 𝑥 , 𝑥 ∈ (0, 𝑎).
Squaring both sides, we get
𝑍=𝐴 = 𝑥 (𝑎 − 𝑥 ) = (𝑥 𝑎 − 𝑥 ), 𝑥 ∈ (0, 𝑎).
A is maximum when Z is maximum.
𝑑𝑍 16𝑏 32𝑏
= (2𝑥𝑎 − 4𝑥 ) = 𝑥 𝑎 + √2𝑥 (𝑎 − √2𝑥)
𝑑𝑥 𝑎 𝑎
=0⇒𝑥= .

𝑑 𝑍 32𝑏
= (𝑎 − 6𝑥 )
𝑑𝑥 𝑎
𝑑 𝑍 32𝑏
( ) = (𝑎 − 3𝑎 ) = −64𝑏 < 0
𝑑𝑥 √ 𝑎 1
Hence, by the second derivative test, there is a local maximum value of Z at the
critical point 𝑥 = . Since there is only one critical point, therefore, Z is

1/2
maximum at 𝑥 = , hence, A is maximum at 𝑥 = .
√ √
Thus, for maximum area of the soccer field, its length should be 𝑎√2 and its width
should be 𝑏√2. 1/2
38. (i)Let P be the event that the shell fired from A hits the plane and Q be the event
that the shell fired from B hits the plane. The following four hypotheses are
possible before the trial, with the guns operating independently:
𝐸 = 𝑃𝑄,𝐸 = 𝑃 𝑄,𝐸 = 𝑃 𝑄,𝐸 = 𝑃𝑄
Let E = The shell fired from exactly one of them hits the plane.
𝑃(𝐸 ) = 0.3 × 0.2 = 0.06, 𝑃 (𝐸 ) = 0.7 × 0.8 = 0.56, 𝑃(𝐸 ) = 0.7 × 0.2
= 0.14, 𝑃 (𝐸 ) = 0.3 × 0.8 = 0.24
𝐸 𝐸 𝐸 𝐸
𝑃 = 0, 𝑃 = 0, 𝑃 = 1, 𝑃 =1 1
𝐸 𝐸 𝐸 𝐸
𝐸 𝐸 𝐸 𝐸
𝑃(𝐸) = 𝑃(𝐸 ). 𝑃 + 𝑃(𝐸 ). 𝑃 + 𝑃(𝐸 ). 𝑃 + 𝑃(𝐸 ). 𝑃
𝐸 𝐸 𝐸 𝐸
= 0.14 + 0.24 = 0.38 1
( ).
(ii)By Bayes’ Theorem, P =
( ). ( ). ( ). ( ).

0.14 7 2
= =
0.38 19

NOTE: The four hypotheses form the partition of the sample space and it can be
seen that the sum of their probabilities is 1. The hypotheses 𝐸 𝑎𝑛𝑑 𝐸 are actually
eliminated as 𝑃 =𝑃 =0
Alternative way of writing the solution:
(i)P(Shell fired from exactly one of them hits the plane)
= P[(Shell from A hits the plane and Shell from B does not hit the plane) or (Shell
1
from A does not hit the plane and Shell from B hits the plane)]
= 0.3 × 0.8 + 0.7 × 0.2 = 0.38
1
(ii)P(Shell fired from B hit the plane/Exactly one of them hit the plane)
P(Shell ired from B hit the plane ∩ Exactly one of them hit the plane)
=
P(Exactly one of them hit the plane)

Page
11
KENDRIYA VIDYALAYA SANGATHAN, GURUGRAM REGION
(SUMMER STATION)
FIRST PRE BOARD EXAMINATION - 2023-24
CLASS: XII Maximum Marks: 80
SUBJECT: MATHEMATICS TIME ALLOWED: 3 Hours
General instruction
1 This question paper contains- five sections A, B, C, D and E. Each section is
compulsory. However, there are internal choices in some questions.
2. Section A has 18 MCQ’s and 02 Assertion- Reason based question of 1 mark
each.
3. Section B has 5 very short answer type questions (VSA) of 2 marks each.
4. Section C has 6 short answer type questions (SA) of 3 marks each.
5. Section D has 4 long answer type questions (LA) of 5 marks each.
6. Section E has 3 source base / case based/passage based / integrated units of
aassessment 4 marks each with sub parts.
Q.No. SECTION A Marks
(MULTIPLE CHOICE QUESTIONS)
1 −1 −1 1
If x = √asin t and y = √acos t then

dy dy
(a)x +y=0 (b) x =y
dx dx

dy
(c) y =x (d) none of the above
dx

2 3𝑥 2 1
∫ 6 𝑑𝑥 𝑖𝑠 𝑒𝑞𝑢𝑎𝑙 𝑡𝑜
𝑥 +1

(a) log (x6 +1) +c ( b) tan-1 x3 +c


(c) 3 tan -1x3 + c (d) log x2 +c
3 1 0 0 1
If 𝐴 = [0 1 0] then A2 + 2A is equal to
0 0 1

(a) 4A (b) 3A (c) 2A (d) A


4 A square matrix A = [𝑎𝑖𝑗 ] n × n is called a diagonal matrix 1
if 𝑎𝑖𝑗 = 0 for
(a) i = j (b) i < j (c) i > j (d) i ≠ j
5 The feasible region for LPP is shown shaded in the figure. 1
Let Z = 3 x – 4 y be the objective function, then maximum value
of Z is
(a) 12
(b) 8
(c) 0
(d) –18
6 The area of the feasible region for the following constraints 1
3y + x ≥ 3, x ≥ 0, y ≥ 0 will be
(a) Bounded (b) Unbounded (c) Convex (d) Concave

7 The direction cosines of the line which makes equal angles with 1
the coordinate axes are
1 1 1 1 1 1
(a) ( , , ) (b) (− ,− ,− )
√3 √3 √ 3 √3 √3 √3

1 1 1
(c) (± ,± ,± ) (d) none of the above
√3 √3 √3

8 √3 ⃗⃗ 1
If |𝑎⃗| = , |𝑏| = 4 and angle between 𝑎⃗⃗⃗⃗ 𝑎𝑛𝑑 𝑏⃗⃗⃗⃗ is 600 then
2
the value of 𝑎 ⃗⃗⃗⃗ is equal to
⃗⃗⃗⃗ . 𝑏
1
(a) √3 (b) (c) −√3 (d) none of the above
3 √

9 Order and degree of differential equation 1


1
2 4
𝑑2𝑦 𝑑𝑦
= [𝑦 + ( ) ]
𝑑𝑥 2 𝑑𝑥
(a) 4 and 2 (b) 1 and 2 (c) 1 and 4 (d) 2 and 4

10 The number of all possible matrices of order 3x3 with each 1


entry -1 or 1 is
(a) 512 (b) 81 (c) 27 (d) 18

𝑥−1 𝑦−3 𝑧+6 𝑥−1 𝑦−3 𝑧+6


11 If the lines = = 𝑎𝑛𝑑 = = are 1
𝑘 1 −2 1 −2 𝑘
perpendicular, then k is equal to
(a) 2 (b) 1 (c) -2 (d) 3

12 𝑑𝑦 1
Integrating factor of differential equation 𝑥 + 2𝑦 = 𝑥 2
𝑑𝑥
1 1
(a) (b) x2 (c) x (d)
𝑥2 𝑥

13 𝑑𝑦 1
The solution of differential equation 2𝑥 − 𝑦 = 3 represents:
𝑑𝑥
(a)straight lines (b) circles (c) parabola (d) ellipse

14 x 2 6 9 1
If | |= | | then x is equal
18 x 4 6

(a) 6 (b) -6 (c) ±6 (d) none of the above


15 The probability of obtaining an even prime number on each
die, when a pair of dice is rolled is 1
1 1 1
(a) 0 (b) (c) (d)
3 12 36
16 A unit vector perpendicular to both the vectors 1
𝑖̂ − 2𝑗̂ + 3𝑘̂ 𝑎𝑛𝑑 𝑖̂ + 2𝑗̂ − 𝑘̂ is

1 1
(a) ±
3
(𝑖̂ + 𝑗̂ + 𝑘̂) (b) ± (−𝑖̂ + 𝑗̂ + 𝑘̂)
√ √3
1 1
(c) ± (𝑖̂ − 𝑗̂ − 𝑘̂) (d) ± (𝑖̂ − 𝑗̂ + 𝑘̂)
√3 √3
17 1 2 1
If 𝐴 = [ ] , then find the value of |2A|
4 2
(a) -6 (b) -24 (c) 12 (d) -12

18 if α is the angle between any two vectors 1


𝑎⃗ 𝑎𝑛𝑑 𝑏⃗⃗ , 𝑡ℎ𝑒𝑛 |𝑎⃗. 𝑏⃗⃗| = |𝑎⃗ × 𝑏⃗⃗| when α is equal to

𝜋 𝜋
(a) 0 (b) (c) (d) 𝜋
4 2
ASSERTION – REASON BASED QUESTIONS
Directions: Each of these questions contains two statements,
Assertion and Reason. Each of these questions also has four
alternative choices, only one of which is the correct answer.
You have to select one of the codes (a), (b), (c) and (d) given
below.
(a) Assertion is correct, reason is correct; reason is a correct
explanation for assertion.
(b) Assertion is correct, reason is correct; reason is not a
correct explanation for assertion
(c) Assertion is correct, reason is incorrect
(d) Assertion is incorrect, reason is correct.
19 Assertion: A relation R = {(a,b) : |a-b|<2} defined on the set 1
A = {1, 2, 3, 4, 5} is reflexive.
Reason : A relation R on the set A is said to be reflexive if
(a,b)𝜖 𝑅 𝑎𝑛𝑑 (𝑏, 𝑐)𝜖𝑅 𝑓𝑜𝑟 𝑎𝑙𝑙 𝑎, 𝑏 𝜖 𝐴.

20 Assertion: The intervals in which f(x) = log sin x , 0 ≤ 𝑥 ≤ 𝜋 is 1


𝜋
Increasing is (0, ).
2
Reason: A function is increasing in (a,b) if f’ (x) > 0 for each
x𝜖(𝑎, 𝑏).
SECTION B
21 Find the value of 2
1 1
cos −1 + 2 sin−1
2 2
OR

tan−1 (tan )
6
22 𝑥 2 +𝑥+1 2
Find ∫ (𝑥+2)(𝑥 2 +1) 𝑑𝑥 .
23 𝑥 + 𝑘, 𝑖𝑓 𝑥 < 3 2
If function 𝑓(𝑥) = { 4 , 𝑥 = 3 is continuous function at
3𝑥 − 5, 𝑥 > 3
x=3, then find the value of k.
24 The volume of the cube is increasing at the rate of 9 cubic 2
centimeters per second. How fast is the surface area increasing
when the length of an edge is 10 centimeters?
OR
Find the maximum profit that a company can make, if the point
function is given by
p(x) = 41-72x-18x2

25 Find the intervals in which the function f given by 2


f(x) = 4x3 -6x2 -72x +30
(a) Strictly increasing (b) strictly decreasing

SECTION C
26 𝑑𝑦 3
Find , 𝑖𝑓 𝑦 𝑥 + 𝑥 𝑦 + 𝑥 𝑥 = 𝑎𝑏
𝑑𝑥
OR
𝑑2 𝑦
If x= a (cost +t sint) and y = a (sint-t cost), find .
𝑑𝑥 2

27 Let a pair of dice be thrown and the random variable X be the 3


sum of the numbers that appear on the two dice. Find the
expectation of X.
OR
A die is thrown twice and the sum of the numbers appearing is
observed to be 6. What is the conditional probability that the
number 4 has appeared at least once?
28 Find a particular solution of the differential equation 3
𝑑𝑦
+ 𝑦𝑐𝑜𝑡𝑥 = 4𝑥 𝑐𝑜𝑠𝑒𝑐𝑥 , 𝑥 ≠ 0, 𝑔𝑖𝑣𝑒𝑛 𝑡ℎ𝑎𝑡
𝑑𝑥 𝜋
𝑦 = 0 𝑤ℎ𝑒𝑛 𝑥 =
2
OR
Find a general solution of the differential equation
ex tan y dx + (1-ex) sec2 y dy =0

29 Evaluate the definite integrals 3


4
∫ [|𝑥 − 1| + |𝑥 − 2| + |𝑥 − 3|] 𝑑𝑥
1
30 Evaluate the integrals 3
𝑥+3
∫ 𝑑𝑥
√5 − 4𝑥 + 𝑥 2

31 Solve the following Linear Programming Problems graphically 3


Maximise Z = 5x + 3y
Subject to 3x + 5y ≤ 15, 5x + 2y ≤ 10, x ≥ 0, y ≥ 0.

SECTION D
32 Find the area of the region bounded by the curve y2 = x and the 5
lines x=1, x=4 and x-axis in the first quadrant.

33 Let A be the set of all the triangle in a plane and R be the


relation defined on R as R = {(T1 , T2 ): T1 is similar to T2}

1. Show that the relation R is an equivalence relation. 4


2. Consider three right angle triangle T1 with sides 3, 4, 5, +1
T2 with sides 5, 12, 13 and T3 with sides 6,8,10. Which
triangle among T1, T2, and T3 are related?
OR

Show that f :R→ {𝑥𝜖𝑅 ∶ −1 < 𝑥 < 1} 3


𝑥
𝑑𝑒𝑓𝑖𝑛𝑒𝑑 𝑏𝑦 𝑓(𝑥) = , x𝜖𝑅 is one – one and onto +2
|𝑥|1+
function.

34 Two factories decided to award their employee for three


values of (a) adaptable to new situation, (b) careful and alert in
difficult situations and (c) keeping calm in tense situations, at
the rate of ₹ x, ₹ y and ₹ z per person respectively. The first
factory decided to honour respectively 2, 4 and 3 employees
with total prize money of ₹ 29000. The second factory decided
to honour respectively 5, 2 and 3 employees with a total prize
money of ₹ 30500. If three prizes per person together cost
₹ 9500 then
(i) Represents the above situation by a matrix equation 1
and form linear equations using matrix multiplication,
(ii) Solve these equation using matrices. 4

35 By computing the shortest distance determine whether the 5


lines intersect or not. If not then find the shortest distance
between the lines.
𝑥−1 𝑦−2 𝑧−3 𝑥−2 𝑦−4 𝑧−5
= = 𝑎𝑛𝑑 = =
2 3 4 3 4 5
OR
Find the vector equation of the line passing through the point
(1, 2, – 4) and perpendicular to the two lines:

𝑥 − 8 𝑦 + 19 𝑧 − 10 𝑥 − 15 𝑦 − 29 𝑧 − 5
= = 𝑎𝑛𝑑 = =
3 −16 7 3 8 −5

SECTION E
This section comprises of 3 case study questions of 4 marks
having sub parts
36 A doctor is to visit a patient. From the past experience, it is
known that the probabilities that he will come by cab, metro,
bike or by other means of transport are respectively 0.3, 0.2,
0.1 and 0.4. The probabilities that he will be late are 0.25, 0.3,
0.35 and 0.1 if he comes by cab, metro, bike and other means
of transport respectively.
(i)What is the probability that the doctor is late by other 1
means?
(ii)When the doctor arrives late, what is the probability that he 1
comes by metro?
(iii) When the doctor arrives late, what is the probability that 2
he comes by bike or other means?
OR
2
When the doctor arrives late, what is the probability that he
comes by cab or metro?
37

Gitika house is situated at Shalimar Bag at O, going to Aloke’s


house she first travel 8 km in the east, here at point A a
hospital is situated. From the hospital she takes auto and goes
6 km in the north. Here at point B a school is situated. From 1
school she travels by bus to reach Aloke’s house which is 300 of 1
east and 6 km from point B. 2
(i) What is vector distance from Gitika’s house to school?
(ii) What is vector distance from school to Aloke’s house?
(iii) What is vector distance from Gitika’s house to Aloke’s 2
house?
OR
What is the total distance travel by Gitika from her house to
Aloke’s house?

38 A telephone company in a town has 500 subscribers on its list


and collect fixed charges of ₹ 300 per subscriber. The company
proposes to increase the annual subscription and it is believed
that every increase of ₹ 1, one subscriber will discontinue the
service.
2
(i) Based on above information find out
how much amount can be increased
for maximum revenue.
2
(ii) Find out maximum revenue
received by the telephone company.

================================================================
KENDRIYA VIDYALAYA SANGATHAN
GURUGRAM REGION
MARKING SCHEME
(SUMMER STATION)
FIRST PRE BOARD EXAMINATIONS 2023-24
CLASS XII
SUBJECT MATHEMATICS
Q.no. SECTION A Marks
1 dy 1
(a) x + y = 0
dx
2 (b) tan-1 x3 +c 1
3 (b) 3A 1
4 (d) i ≠ j 1
5 (c) 0 1
6 (b) Unbounded 1
7 1 1 1 1
(c) (± ,± ,± )
√3 √3 √3
8 (a) √3 1
9 (d) 2 and 4 1
10 (a) 512 1
11 (c) -2 1
12 (b) x2 1
13 (c) parabola 1
14 (c) ±6 1
1
15 (d) 1
36
1
16 (b) ± (−𝑖̂ + 𝑗̂ + 𝑘̂) 1
√3
17 (b) -24 1
𝜋
18 (b) 1
4
19 (c) Assertion is correct, reason is incorrect 1
20 (a) Assertion is correct, reason is correct; reason is a correct 1
explanation for assertion.
SECTION B
21 π π
cos −1 (cos ) + 2sin−1 (sin )
3 6 1
π π 2π
+2× = 1
3 6 3

OR
π
tan−1 (tan π + ) 1
6
π π π
∈ (− , )
6 2 2 1

22 𝑥 2 +𝑥+1 𝐴 𝐵𝑥+𝑐 ½
= +
(𝑥+2)(𝑥 2 +1) 𝑥+2 𝑥 2 +1
A=3/5 , B= 2/5 and C = 1/5
2 1 1/2
𝑥2 + 𝑥 + 1 3 𝑥+
2
= + 52 5
(𝑥 + 2)(𝑥 + 1) 5(𝑥 + 2) 𝑥 + 1
2
𝑥 +𝑥+1
∫ 𝑑𝑥
(𝑥 + 2)(𝑥 2 + 1)
3 1 1
= log|𝑥 + 2| + 𝑙𝑜𝑔|𝑥 2 + 1| + tan−1 𝑥 + 𝐶
5 5 5 1

23 lim f(x) = lim− f(x) = f(3) ½


x→3+ x→3

f(3) = 4 ½

lim f(x) = lim+ 3x − 5 = 4


x→3+ x→3 ½
lim f(x) = lim− x + k = 3 + k
x→0− x→3

3+k = 4 , k = 1 1/2
24 the volume of a cube with radius “x” is given by V = x3 ans
surface area = 6x2 ½
Hence, the rate of change of volume “V’ with respect to the
𝑑𝑉
time “t” is given by: = 9 cm3/s
𝑑𝑡
𝑑𝑉 𝑑 𝑑𝑥 𝑑𝑥 3
9= = 𝑥 = 3𝑥 2 . , By using the chain rule =
3
1/2
𝑑𝑡 𝑑𝑡 𝑑𝑡 𝑑𝑡 𝑥2
𝑑𝑠 𝑑(6𝑥 2 ) 𝑑𝑥 3 36
= = 12 = 12 × 2 =
𝑑𝑡 𝑑𝑡 𝑑𝑡 𝑥 𝑥
𝑑𝑠 2
At x= 10 , = 3.6 cm /s 1/2
𝑑𝑡
OR ½
p(x) = 41-72x-18x2
P’(x) = -72 -36
P’’ = -36
½
For maximum or minina or critical points p’(x) =0
½
-72 -36x =0 , x=-2
x=-2, p’’ (-2) = -36 <0 ½
hence x= -2 is a point of local maxima,
maximum profit = 113
½
3 2
25 f(x) = 4x -6x -72x +30
𝑓 ′ (𝑥) = 12𝑥 2 − 12𝑥 − 72 ½
For critical points 𝑓 ′ (𝑥) = 0, 1/2
12𝑥 2 − 12𝑥 − 72 = 0
12(x-3)(x+2) = 0
X= 2, x= -3 ½
(−∞, −2) and (3, ∞) the function is increasing 1/2
(-2,3) the function is decreasing

SECTION C
26 𝑦 + 𝑥 𝑦 + 𝑥 𝑥 = 𝑎𝑏
𝑥

𝑑 𝑑 𝑏
(𝑦 𝑥 + 𝑥 𝑦 + 𝑥 𝑥 ) = 𝑎
𝑑𝑥 𝑑𝑥 1/2
𝑑
(𝑦 𝑥 + 𝑥 𝑦 + 𝑥 𝑥 ) = 0
𝑑𝑥
𝑑𝑢 𝑑𝑣 𝑑𝑤 ½
+ + =0
𝑑𝑥 𝑑𝑥 𝑑𝑥
Taking log one by one for u = yx ,v= xy, w= xx

𝑑𝑢 𝑥 𝑑𝑦
= 𝑦𝑥 ( + 𝑙𝑜𝑔𝑦), ½
𝑑𝑥 𝑦 𝑑𝑥

𝑑𝑣 𝑦 𝑑𝑦
= 𝑥 𝑦 ( + 𝑙𝑜𝑔𝑥 ),
𝑑𝑥 𝑥 𝑑𝑥 ½
𝑑𝑤
= 𝑥 𝑥 (1 + 𝑙𝑜𝑔𝑥)
𝑑𝑥 1/2
𝑑𝑢 𝑑𝑣 𝑑𝑤
+ +
𝑑𝑥 𝑑𝑥 𝑑𝑥
𝑥 𝑑𝑦 𝑦 𝑑𝑦 ½
= 𝑦𝑥 ( + 𝑙𝑜𝑔𝑦) + 𝑥 𝑦 ( + 𝑙𝑜𝑔𝑥 ) + 𝑥 𝑥 (1 + 𝑙𝑜𝑔𝑥)
𝑦 𝑑𝑥 𝑥 𝑑𝑥
=0
OR
𝑑2𝑦
If x= a(cost +t sint) and y = a(sint-t cost), find 2
𝑑𝑥
𝑑𝑥 𝑑𝑦 1+1
= 𝑎𝑡𝑐𝑜𝑠𝑡 𝑎𝑛𝑑 = 𝑎𝑡 𝑐𝑜𝑠𝑡
𝑑𝑡 𝑑𝑡
𝑑𝑦 1/2
= 𝑡𝑎𝑛𝑡
𝑑𝑥
𝑑2𝑦 2
𝑑𝑡
= 𝑠𝑒𝑐 𝑡
𝑑𝑥 2 𝑑𝑥 1/2
𝑑2𝑦 𝑠𝑒𝑐 2 𝑡
=
𝑑𝑥 2 𝑎𝑡

27
xi 2 3 4 5 6 7 8 9 10 11 12
4 4 2
Pi(x) 1 2 3 5 6 5 3 2 1
36 36 36 36 36 36 36 36 36 36 36
1
E(X) = µ = ∑𝑛𝑖=1 𝑥𝑖 𝑃𝑖 = 7
OR
Let E be the event that ‘number 4 appears at least once’ and F ½
be the event that ‘the sum of the numbers appearing is 6’.
Then,
E = {(4,1), (4,2), (4,3), (4,4), (4,5), (4,6), (1,4), (2,4), (3,4), (5,4), ½
(6,4)}
F = {(1,5), (2,4), (3,3), (4,2), (5,1)} 1/2

P(E) = 11/36 and P(F) = 5/36 , ½


E∩F = {(2,4), (4,2)}, P(E∩F) = 2/36 1/2

𝑃(E∩F ) 2 1/2
P(E/F) = =
𝑃(𝐹) 5

28 𝑑𝑦
+ 𝑦𝑐𝑜𝑡𝑥 = 4𝑥 𝑐𝑜𝑠𝑒𝑐𝑥
𝑑𝑥
P= cot x , Q = 4x cosec x ½

I.F = 𝑒 ∫ cot 𝑥𝑑𝑥 = 𝑠𝑖𝑛𝑥 ½


y I.F. = ∫ 𝑄. 𝐼𝐹 𝑑𝑥 1/2

y sinx = ∫ 4x cosec x sinxdx =∫ 4x dx ½


𝜋2 ½
y sinx = 2x2 +C, C = −
2
2 𝜋2
y sinx = 2x − ½
2
OR
e tan y dx + (1-ex) sec2 y dy =0
x

(1-ex) sec2 y dy = - ex tan y dx ½


sec2 y −ex ½
dy = dx
tany 1−ex
log |tany|= log |1-ex| + log|c| 1
log |tany|- log |1-ex| = log|c| ½

Taking exponential both side 1/2


tany = c (1-ex)
4
29
∫ [|𝑥 − 1| + |𝑥 − 2| + |𝑥 − 3|] 𝑑𝑥
1

4 4 4
∫ |𝑥 − 1|𝑑𝑥 + ∫ |𝑥 − 2|𝑑𝑥 + ∫ |𝑥 − 3|𝑑𝑥 ½
1 1 1
4 2 4 3
= ∫1 (𝑥 − 1)𝑑𝑥 + ∫1 −(𝑥 − 2)𝑑𝑥 + ∫2 (𝑥 − 2) + + ∫1 −(𝑥 −
4
3) + ∫3 (𝑥 + 3) 1
For calculation 1
4
19
∫ [|𝑥 − 1| + |𝑥 − 2| + |𝑥 − 3|] 𝑑𝑥 = 1/2
1 2
30 𝑥+3
∫ 𝑑𝑥
√5 − 4𝑥 + 𝑥 2
𝑑 ½
X+3 = A (5 − 4𝑥 + 𝑥 2 ) + 𝐵
𝑑𝑥
A=-1/2 and B = 1
𝑥+3 ½
∫ 𝑑𝑥
√5 − 4𝑥 + 𝑥 2
1 (−4 − 2𝑥) 𝑑𝑥
= − ∫ 𝑑𝑥 + ∫
2 √5 − 4𝑥 + 𝑥 2 √5 − 4𝑥 + 𝑥 2 ½
1
For calculation
𝑥+3 𝑥+2
∫ 𝑑𝑥 = −√5 − 4𝑥 + 𝑥 2 + sin−1 +𝐶 1/2
√5 − 4𝑥 + 𝑥 2 3
31

½
Corner points (0,0), (0,3) , (2,0), (20/19 , 45/19)

Max. of Z = 235/19, at (20/19 , 45/19) ½


SECTION D
32 For labelled and correct diagram 2

4
Required area = ∫1 √𝑥𝑑𝑥
1/2
For calculation
14 2
= 1/2
3
33 To prove reflexive 1
To prove symmetric 1
To prove transitive 2
3 4 5 1
Proving T1 related with T3 = =
6 8 10
0R
Showing one one 3
Taking cases, x1,x2 both are even , x1,x2 both are odd and x1,x2
one is odd another is even 2
Showing onto by any method

34 (i) 2x +4y +3z = 29000 1


5x +2x +3x = 30500
x +y + z = 9500

(ii) matrix method AX=B


X=A-1B ½
IAI = -1 ½
−1 −1 6
adj A=[−2 −1 9 ] 2
3 2 −16 ½
calculation
X= 2500, y= 3000 and z =4000 ½

35 𝑥−1 𝑦−2 𝑧−3 𝑥−2 𝑦−4 𝑧−5


= = 𝑎𝑛𝑑 = =
2 3 4 3 4 5
⃗⃗⃗ ̂
a1 = 𝑖̂ + 2𝑗̂ + 3𝑘 , ⃗⃗⃗⃗
a2 = 2𝑖̂ + 4𝑗̂ + 5𝑘 ̂ 1
⃗⃗⃗⃗
b1 = 2𝑖̂ + 3𝑗̂ + 4𝑘̂ , ⃗⃗⃗⃗
b2 = 3𝑖̂ + 4𝑗̂ + 5𝑘̂

a1 = 𝑖̂ + 2𝑗̂ + 2𝑘̂
a⃗⃗⃗⃗2 − ⃗⃗⃗ ½
⃗⃗⃗⃗
b1 × ⃗⃗⃗⃗⃗ b2 = −𝑖̂ + 2𝑗̂ − 𝑘̂ 1

⃗⃗⃗⃗1 × ⃗⃗⃗⃗⃗
|b b2 | = √6
½
⃗⃗⃗⃗⃗⃗ b2 ) . ⃗⃗⃗⃗⃗⃗
(b1 × ⃗⃗⃗⃗⃗ (a2 − ⃗⃗⃗
a1 ) = 1

⃗⃗⃗⃗⃗⃗⃗ ⃗⃗⃗⃗⃗⃗ ) .(a


⃗⃗⃗⃗⃗⃗ −a
½
(b ×b ⃗⃗⃗⃗ )
S.D. = d = | 1 ⃗⃗⃗⃗2 ⃗⃗⃗⃗⃗⃗2 1 |
|b1 ×b2 |
1 ½
Shortest distance d =
√6 ½
The lines do not intersect 1/2
OR
Eq. of line 𝑟 = 𝑎 + ⅄𝑏⃗
Line passes through (1,2,-4) and let (a,b,c ) be the D,Ratio of ½
line then
Eq of line is
𝑟 = 𝑖̂ + 2𝑗̂ − 4𝑘̂ + ⅄(𝑎𝑖̂ + 𝑏𝑗̂ + 𝑐𝑘̂ )
Line is perpendicular to the lines 1/2
𝑥 − 8 𝑦 + 19 𝑧 − 10 𝑥 − 15 𝑦 − 29 𝑧 − 5
= = 𝑎𝑛𝑑 = =
3 −16 7 3 8 −5
𝑎 × 𝑏⃗ 𝑖𝑠 𝑝𝑒𝑟𝑝𝑒𝑛𝑑𝑖𝑐𝑢𝑙𝑎𝑟 𝑡𝑜 𝑎 𝑎𝑛𝑑 𝑏⃗ 𝑏𝑜𝑡ℎ

𝑖̂ 𝑗̂ 𝑘̂
𝑎 × 𝑏⃗ = |3 −16 7 |
3 8 −5 1
̂ 2
= 24𝑖̂ + 36𝑗̂ + 72𝑘 2
Hence D’ Ratio of line is (24,36,72)

Eq. of line
𝑥−1
=
𝑦−2
=
𝑧+4 ½
24 36 72

½
𝑟 = 𝑖̂ + 2𝑗̂ − 4𝑘̂ + ⅄(24𝑖̂ + 36𝑗̂ + 72𝑘̂ )
½
𝑟 = 𝑖̂ + 2𝑗̂ − 4𝑘̂ + ⅄(2𝑖̂ + 3𝑗̂ + 6𝑘̂ )
SECTION E
36 Let A be the event that the doctor visit the patient late and let
E1, E2, E3, E4 be the events that the doctor comes by cab,
metro, bike and other means of transport respectively.
P(E1)=0.3, P(E2)=0.2, P(E3)=0.1, P(E4)=0.4

P(A I E1) = Probability that the doctor arriving late when he


comes by cab = 0.25
Similarly, P ( A I E2) = 0.3, P (A I E3) = 0.35 and
P ( A I E4) = 0.1
A A A
P(A) = P (𝐸1 )P ( ) + P(𝐸2 )P ( ) + P(𝐸3 )P ( ) +
𝐸1 𝐸2 𝐸3
A
P(𝐸4 )P ( )
𝐸 4
P(A) = 0.25x0.3 + 0.3x0.2 + 0.1x0.35 + 0.4x0.1 = 0.21

(i)P(𝐸4 /𝐴) =
P (E4 )P(A/E4 )
=
4 1
A A A A
P (E1 )P( )+ P(E2 )P( )+P(E3 )P( )+P(E4 )P( ) 21
E1 E2 E3 E4
P (𝐸2 )P(A/𝐸2 ) 2
(i)P(𝐸2 /𝐴) = A A A A =
P (E1 )P( )+ P(E2 )P( )+P(E3 )P( )+P(E4 )P( ) 7
E1 E2 E3 E4 1

(iii) P(𝐸3 /𝐴) + P(𝐸4 /𝐴) =


A
P (𝐸3 )P( )+ P (𝐸4 )P(A/𝐸4 ) 5
𝐸3
A A A A = 2
P (E1 )P( )+ P(E2 )P( )+P(E3 )P( )+P(E4 )P( ) 14
E1 E2 E3 E4

OR
P(𝐸1 /𝐴) + P(𝐸2 /𝐴) =
A
P (𝐸1 )P(𝐸 )+ P (𝐸2 )P(A/𝐸2 ) 9
1
= 2
A A A A 14
P (E1 )P(E )+ P(E2 )P(E )+P(E3 )P(E )+P(E4 )P(E )
1 2 3 4
37 (i)we have ⃗⃗⃗⃗⃗
𝑂𝐴 = 8𝑖̂ 𝑘𝑚 ⃗⃗⃗⃗⃗
𝐴𝐵 = 6𝑗̂ 1
vector distance from Gitika’s house to school =8𝑖̂ + 6𝑗̂
(ii)vector distance from school to Aloke’s house
= 6cos300𝑖̂ + 6𝑠𝑖𝑛300 𝑗̂ 1
= 3√3𝑖̂ + 3𝑗̂
(iii)vector distance from Gitika’s house to Aloke’s house=
8𝑖̂ + 6𝑗̂ + 3√3𝑖̂ + 3𝑗̂ 2
=(8 + 3√3)+9𝑗̂
OR
The total distance travel by Gitika from her house to Aloke’s
2
house= 8 + 6 + 6 = 20 km
38 Let the increase of ₹ x in annual subscription of ₹ 300
maximize the profit of the company. Due to this increase of ₹
x, x subscriber will discontinue. Therefore
Number of subscriber = 500-x
Annual subscription = ₹ (300+x)
R be the total revenue =(500-x)(300-x)
𝑑𝑅 𝑑2𝑅 2
= 200 − 2𝑥 𝑎𝑛𝑑 2 = −2
𝑑𝑥 𝑑𝑥
𝑑𝑅
For critical point = 200 − 2𝑥 = 0 , x= 100
𝑑𝑥
𝑑2𝑅
< 0 𝑎𝑡 x=100 2
𝑑𝑥 2
So R is maximum at x=100 and maximum R = 400x400=160000

================================================================
=
( ) 1
( )
0.14 7 1
= =
0.38 19

Page
12
Practice Questions
Session 2022-23
Class XII
Mathematics (Code-041)

Time Allowed: 3 Hours Maximum Marks: 80

General Instructions:
1. This Question paper contains - five sections A, B, C, D and E. Each section is
compulsory. However, there are internal choices in some questions.
2. Section A has 18 MCQs and 02 Assertion-Reason based questions of 1 mark each.
3. Section B has 5 Very Short Answer (VSA)-type questions of 2 marks each.
4. Section C has 6 Short Answer (SA)-type questions of 3 marks each.
5. Section D has 4 Long Answer (LA)-type questions of 5 marks each.
6. Section E has 3 source based/case based/passage based/integrated units of
assessment (4 marks each) with sub parts.

SECTION A
(This section comprises of Multiple-choice questions (MCQ) of 1 mark each.)
Q.No Question Marks
Q.1 Shown below is the graph of a function f(x) whose domain is R – (-1,1). Some portion of
the graph is hidden behind the star. 1

Which of the following is f(x)?

A. tan-1 x
B. cot-1 x
C. sec-1 x
D. cosec-1 x
Q.2 P and Q are matrices such that both (P + Q) and (PQ) are defined. 1

Which of the following is true about P and Q?

A. P and Q can be any matrices but of the same order.


B. P and Q must be square matrices of the same order.
C. P and Q must be square matrices not necessarily of the same order.
D. Order of P and Q must be of the form m × k and k × n respectively, with no
condition on m and n.

Q.3 1

Under which of the following conditions will |A| be equal to 0?

i) a - 2p = b - 2q = c - 2r = 0
ii) x = y = z = 0
iii) a : b : c = x : y : z

A. only ii)
B. only i) and ii)
C. only i) and iii)
D. all - i), ii) and iii)

Q.4 If abc = 2, what is the value of the determinant below? 1

A. -48
B. -24
C. 48
D. (cannot be found without the values of a, b and c)

1
Q.5
For what value of k is the function f continuous at x = 0?

A. 4
B. 1
1
C. 4
1
D. 8

Q.6 What is the integral of the following expression? 1

A. − tan 𝑥 + 𝐴, where A is a constant.


1
B. − tan 𝑥 − 𝐵, where B is a constant.
1
C. tan 𝑥 + 𝐶, where C is a constant.
2
1 1
D. 𝑠𝑒𝑐 𝑥 𝑡𝑎𝑛 − 𝐷, where D is a constant.
𝑥

Q.7 What is the value of the following integral? 1

A. -4
B. -2
C. 0
D. 4

Q.8 Which of the following is CLOSEST to the area under the parabola given by y = 4x2, 1
bounded by the x-axis, and the lines x = (-1) and x = (-2)?

A. 6 sq units
B. 8 sq units
C. 9 sq units
D. 12 sq units

Q.9 Which of the following differential equation has an order of 2 and a degree of 3? 1
A.

B.

C.

D.

Q.10 Following is a differential equation. 1


𝑑𝑦
= 4𝑒 3𝑥
𝑑𝑥

7
If y(0) = , which of the following is a particular solution of the differential equation?
3

4 4
A. 𝑒 3𝑥 − 𝑒7
3 3
29
B. 12𝑒 3𝑥 − 3
4
C. 𝑒 3𝑥 + 1
3
5
D. 4𝑒 3𝑥 − 3

Q.11 1
A. only ii)
B. only iii)
C. only i) and ii)
D. all - i), ii) and iii)

Q.12 1

A.

B.

C.

D.

Q.13 1
A. 8
B. 4
C. 2
D. 0

1
Q.14 A line makes an angle of 135° with the positive direction of the x-axis, and an angle of
300° with the positive direction of the y-axis.

Which of the following could be the angle it makes with the negative direction of the z-
axis?

A. 45°
B. 60°
C. (Such a line does not exist.)
D. (A unique angle made with the z-axis cannot be determined.)

Q.15 1

A. -9
B. -8
C. 7
D. 8

Q.16 A linear programming problem (LPP) along with its constraints is given below. 1

Minimize: 𝑍 = 3x + 2y

Subject to:
5x − 10y ≥ 0
x + y ≤ 1
x ≤ 4
x ≥ 0, y ≥ 0

Which of the following is true about the above LPP?

A. It has no solution.
B. It has a unique solution.
C. It has two distinct solutions.
D. It has infinitely many solutions.

Q.17 M and N are two events such that P(M|N) = 0.3, P(M)= 0.2 and P(N) = 0.4. 1

Which of the following is the value of P(M ∩ N')?

A. 0.8
B. 0.12
C. 0.1
D. 0.08

1
Q.18 The constraints of a linear programming problem along with their graphs is shown below:

x + 2y ≥ 3
x ≥ 10
y ≥ 0
Which of the following objective functions has an optimal solution with respect to the
above set of constraints?

A. Minimise Z = x + y
B. Minimise Z = 0.5x + y
C. Maximise Z = x + y
D. Maximise Z = 2x + y

Q.19
1
Based on the above function, two statements are given below - one labelled Assertion (A)
and the other labelled Reason (R). Read the statements carefully and choose the option that
correctly describes statements (A) and (R).

Assertion (A): The function f is not onto.

Reason (R): 3 ∈ R (co-domain of f) has no pre-image in the domain of f.

A. Both A and R are true and R is the correct explanation of A.


B. Both A and R are true but R is not the correct explanation of A.
C. A is true but R is false.
D. Both A and R are false.

1
Q.20 Two statements are given below - one labelled Assertion (A) and the other labelled Reason
(R). Read the statements carefully and choose the option that correctly describes
statements (A) and (R).

Assertion (A): The function f(x) = |x − 6|(cos x) is differentiable in 𝑅 − {6}.

Reason (R): If a function f is continuous at a point c, then it is also differentiable at that


point.

A. Both (A) and (R) are true and (R) is the correct explanation for (A).
B. Both (A) and (R) are true but (R) is not the correct explanation for (A).
C. (A) is true but (R) is false.
D. (A) is false but (R) is true.
SECTION B

(This section comprises of very short answer type-questions (VSA) of 2 marks each.)

2
Q.21

1
Based on the above equation, find 𝑡𝑎𝑛−1 (𝑥) using the principal values of the inverse
trigonometric functions. Show your work.

OR

i) Find the domain of the function below.

ii) Find the range (principal value branch) of the function below.

Show your work.

Q.22

𝑑𝑦 2
Q.23 Find if 𝑦 = (𝑒 𝑠𝑒𝑐𝑥 + 𝑥)4 . Show your work.
𝑑𝑥
Q.24 2

OR

In the figure below, QRST and QRTP are parallelograms.

Using the vectors shown for RQ and RS, prove that the area of QRST is equal to the area
of QRTP.

Q.25 2

SECTION C
(This section comprises of short answer type questions (SA) of 3 marks each)

Q.26 Check whether the following statement is true or false. 3

Show your work with valid reason.

OR
1 3
Q.27 The anti-derivative of a function of the form (3x − 1)f(x), (x ≠ ), is given by
3
13 3
3𝑥 4 − 𝑥3 + 𝑥 2 + 𝐶, where C is the constant of integration.
3 2

Find the value of f(6). Show your steps.

Q.28 Evaluate the following definite integral and show your work. 3

Q.29 Find the particular solution when x = y = 0 for the following differential equation. 3

Show your steps.

OR

Find the general solution of the following differential equation.

Show your steps.

Q.30 Frame the below optimisation problem as a linear programming problem and determine its 3
feasible region graphically.

Bhavani Singh, a farmer, decides to raise hens and cows to make some extra money apart
from his agricultural income. He wants to raise no more than 16 animals including no
more than 10 hens. On an average it will cost him Rs 25 and Rs 75 per day to raise one
hen and one cow respectively. He will make an average profit of Rs 12 from each hen and
Rs 40 from each cow every day. He has a budget of Rs 900 per day to raise the animals.
How many of each type of animals should he raise to maximise his profit?
Q.31 Nikhil has a bag of marbles that contains exactly 8 green marbles and 6 red marbles. He 3
takes out three marbles successively without replacing any of the marbles.

What is the probability that all three marbles taken out are green in colour? Show your
steps.

OR

In a recreational event at a school, there were 8 students, 6 parents, and 4 teachers. To play
a game, two members were selected randomly one after the other.

Find the probability distribution of the number of students. Show your steps.

SECTION D
(This section comprises of long answer-type questions (LA) of 5 marks each)

Q.32 Sravan is a nutritionist. He wants to create a mixture of orange juice, beetroot juice and 5
kiwi juice that can provide 1860 mg of vitamin C, 22 mg of iron and 760 mg of calcium.
The quantity of each nutrient per litre of juice is shown below.

Using the matrix method, find how many litres of each juice Sravan should add into the
mixture. Show your work.
2 𝑦2
Q.33 Shown below is an ellipse whose equation is 𝑥 + = 1. 5
81 36

Find the area of the shaded region in terms of π. Show your steps.

Q.34 5
OR

Q.35 Integrate the given function and show your steps. 5

OR

Evaluate the integral and show your steps.


SECTION E
(This section comprises of 3 case-study/passage-based questions of 4 marks each
with two sub-questions. First two case study questions have three sub questions of
marks 1, 1, 2 respectively. The third case study question has two sub questions of 2
marks each.)

Q.36 Answer the questions based on the given information.

Port Blair, the capital city of Andaman and Nicobar Islands is directly connected to
Chennai and Vishakapatnam via ship route. The ships sail from Chennai/Vishakapatnam
to Port Blair and vice versa.

Swaraj Dweep and Shaheed Dweep are two popular tourist islands in Andaman Islands.
One has to take a ferry from Port Blair to reach these islands. There are ferries that sail
frequently between the three islands - Port Blair (PB), Swaraj Dweep (SwD) and
Shaheed Dweep (ShD).

Shown below is a schematic representation of the ship routes and ferry routes.

(Note: The image is for representation purpose only.)

X is the set of all 5 places and Y is the set of 3 places in Andaman Islands.

That is, X = {C, V, PB, SwD, ShD} and Y = {PB, SwD, ShD}.

A relation R defined on the set X is given by, R = {(𝑥1 , 𝑥2 ): there is a direct ship or
direct ferry from 𝑥1 to 𝑥2 }.

A function 𝑓: Y → X is defined by, 𝑓(𝑃𝐵) = 𝑉, 𝑓(𝑆𝑤𝐷) = 𝑃𝐵, 𝑓(𝑆ℎ𝐷) = 𝑆𝑤𝐷.


i) List all the elements of R. 1

ii) Is the relation R symmetric? Give a valid reason. 1

iii) Is the relation R transitive? Give a valid reason. 2

OR

Check whether the function f is one-one and onto. Give valid reasons.

Q.37 Answer the questions based on the given information.

A medicinal drug administered into a human body requires some time to produce its effect
on the body. The amount (in mg) of a certain medicinal drug in the bloodstream at t hours
after administering the drug to an individual is given by the function:

C(t) = −t 3 + 4.5t 2 + 54t , 0 ≤ t ≤ 10

Shown below is the graph of C(t) in the interval [0, 10].


i) Find the rate at which the amount of drug is changing in the bloodstream at 5 1
hours after administering the drug. Show your work.
ii) Show that the function C(t) is strictly increasing in the interval (3, 4).
2
OR

The amount of the drug in the bloodstream at which the action of the drug is
maximum is denoted by Cmax.
How long after administering the drug is Cmax attained? Show your work and
give valid reasons.

iii) Find the amount of drug in the bloodstream at the time when the effect of the
drug is maximum. Show your work. 1

Q.38 Answer the questions based on the given information.

A school conducted a survey of their school staff to find their beverage preferences. Each
of them picked either tea or coffee as their first preference and then with sugar or without
sugar as their second preference as shown in the below tree diagram.

Some of the insights from the survey are given below.

♦ 60% percent of the staff prefer coffee.


♦ 90% of those who prefer coffee prefer it with sugar.
♦ 20% of those who prefer tea prefer it without sugar.

i) What is the probability that a person selected randomly from the staff prefers a
2
beverage with sugar? Show your steps.

ii) What is the probability that a person from the staff selected at random prefers
2
coffee given that it is without sugar? Show your steps.
शिक्षा निदे शालय, राष्ट्रीय राजधानी क्षेत्र दिल्ली
Directorate of Education, GNCT of Delhi
Practice Paper
(2023-24)
कक्षा – XII
Class – XII
गणित (कोड: 041)
Mathematics (Code: 041)

समय: 3 घंटे अधिकतम अंक: 80


Time: 3 hours Maximum Marks: 80

सामान्य निर्देश:

1. इस प्रश्न पत्र में पाँच खंड हैं - ए ,बी ,सी ,डी तथा ई ,प्रत्येक खंड अनिवार्य है । यद्यपि कुछ प्रश्नों में
आंतरिक विकल्प दिया है ।
2. भाग ए में 18 प्रश्न ,बहुविकल्पीय तथा 2 अभिकथन-कारण प्रश्न प्रत्येक 1 अंक का है (20 अंक)
3. भाग बी में 5 अति लघु उत्तर (VSA) प्रकार के प्रश्न हैं जिनमें प्रत्येक के 2 अंक हैं (10 अंक )
4. भाग सी में 6 लघु उत्तर (SA) प्रकार के प्रश्न हैं जिनमें प्रत्येक के 3 अंक हैं (18 अंक )
5. भाग डी में 4 में दीर्घ उत्तर (LA)प्रकार के प्रश्न हैं जिनमें प्रत्येक के 5 अंक हैं (20 अंक )
5. भाग ई में 3 स्रोत आधारित /गद्यांश /क्षमता प्रकरण अध्ययन / एसेसमें ट की एकीकृत इकाई
पर आधारित प्रश्न जिसमें प्रत्येक प्रश्न (उपभाग सहित) 4 अंक का है (12 अंक)
General Instructions :

1. This Question paper contains - five sections A,B,C,D,E. Each section is compulsory.
However, there are internal choices in some questions.
2. Section A has 18 MCQ’s and 02 Assertion-Reason based questions of 1 mark each.(20
Marks )
3. Section Bhas 5 Very Short Answer (VSA)-type questions of 2 marks each.(10 Marks )
4. Section C has 6 Short Answer (SA)-type questions of 3 marks each.(18 Marks )
5. Section D has 4 Long Answer (LA)-type questions of 5 marks each.(20 Marks )
6. Section E has 3 Source based/Case based/passage based/integrated units of assessment
(4 marks each) with sub parts.(12 Marks )

खंड - ए
Section – A
प्रश्न संख्या 1-18 ,बहुविकल्पीय प्रकार के प्रश्न हैं ।प्रत्येक प्रश्न 1 अंक का है ।
Question Number 1-18 are of MCQ type question one mark each.
1. फलन cos−1 (2 x−1) का प्रांत है : 1

The domain of the function cos−1 (2 x−1) is :

(a) [0,1] (b ) [-1,1]

(c) (-1,1) (d) [0 , π ]

[ ]
2. 0 a b 1
यदि A = 2 1 c एक सममित आव्यह
ू है ती (a+b+c) का मान होगा ?
3 4 5
[ ]
0 a b
If A = 2 1 c is a symmetric matrix , then the value of (a+b+c) is ;
3 4 5

(a) 9 (b )8

(c) 7 (d) 6

3. यदि आव्यूह A=[ 0 1 2 ]1 x 3 है , तो आव्यूह A A T है (जहाँ A T आव्यूह A का परिवर्त 1


If a matrix A =[ 0 1 2 ]1 x 3 then the matrix A A T (where A T is transpose of A )
is:
(a) [ 0 ] (b ) [ 3 ]

[ ]
(c) [ 5 ] 0 1 2
(d) 0 1 2
0 1 2

[ ]
4. 2 0 0 1
यदि //If A= 0 3 0
0 0 1
तब |adj A| का मान होगा :
then the value of |adj A| is :
(a) 6 (b ) 1/6

(c) 31 (d) 216

यदि आव्यह
ू A ,B तथा C इस प्रकार हैं कि A p×4 . B q×5=C 2×5 तब मान होगा?
2 2
5. p −q 1
2 2
If matrices A ,B and C are such that A p×4 . B q×5=C 2×5 , then the value of p −q is :

(a) -12 (b ) 12

(c) 16 (d) -16

6. x⩽3 तथा y⩾3 का आलेख स्थित है : 1

The graph of x⩽3 and y⩾3 lie in :

अ (a){प्रथम और द्वितीय चतुर्थांश) ब (b ) ( द्वितीय और तति


ृ ये चतुर्थांश)

Ist and 2nd quadrant 2nd and 3rd quadrant

(c) तति
ृ ये और चतुर्थ चतुर्थांश (d) Ist and 4th quadrant

3rd and 4th quadrant प्रथम और चतर्थ


ु चतर्थां
ु श

7.
( )( )
3 1 1
d y dy
अवकल समीकरण 3
. 3
=0 के कोटि और भज
ु का योग है :
dx
3
dx 1

Sum of order and degree of differential equation is :

( )( )
1
d3 y 3 dy 1

3 . 3
=0
dx dx

(a) 6 (b ) 5

(c) 3 (d) 2
8. 1
sec−1 √ +sin−1 √
x +1 x +1
का x के सापेक्ष अवकलज है :
√ x−1 √ x−1
−1 √ x+1 −1 √ x+1
Derivative of sec +sin w.r.t x is:
√ x−1 √ x−1
(a) 0 (b ) 1

(c) x (d) x 2

9. 1
3
∫ x+x 1 dx बराबर है/is equal to :
x2 x3 x2 x3
(a) x+ + −log|1−x|+C (b ) x + − −log|1−x|+C
2 3 2 3

(c) (d)
x2 x 3 x 2 x3
x− − −log|1+ x|+C x− + −log|1+ x|+C
2 3 2 3

10. dy 2 1
Integrating factor of x +2 y=x is :
dx

(a) x3 (b) x 2

4
(c) x (d) x

[ ]
11. 2 3 2 1
dy d y
अवकल समीकरण की घात है :
2
1+( ) =
dx dx 2

[ ]
2 3 2
dy 2 d y
The degree of the differential equation 1+( ) = 2
is:
dx dx
(a) 4 (b)
3
2

(c) परिभाषित नहीं /Not defined (d) 2

12. सदिश 2 ^i +3 ^j−6 k^ का ^i−2 ^j+3 k^ पर प्रक्षेप है : 1


The projection of 2 ^i +3 ^j−6 k^ on the vector ^i−2 ^j+3 k^ is :
2 1
(a) (b )
√ 14 √ 14

3 (d)
(c) −2
√ 14
√ 14

13. एक समांतर चतुर्भुज जिसके विकर्ण ⃗


a =3 ^i + ^j−2 k^ तथा ⃗b= ^i−3 ^j+4 ^j हैं ,तो क्षेत्रफल होगा :
Area of the parallelogram whose diagonals are a =3 ^i + ^j−2 k^ and
⃗ 1

b= ^i−3 ^j+4 ^j is given by :

(a) 10 √ 3 (b) 5 √ 3

(c) 8 (d) 4
14. यदि = |⃗a + ⃗b|=|⃗a −⃗b| तब ⃗ a तथा ⃗ b के बीच का कोण होगा : 1
If |⃗a + ⃗b|=|⃗a −⃗b| then the angle between ⃗ a and ⃗ b is :

(a) π (b ) 0
2

(c) (d)
π π
4 6

15. किसी रे खा के दिक् - कोसाइन 1 ,3 ,5 हैं तो इसके दिक् - अनप


ु ात होंगे : 1
The direction ratios of line are 1 ,3 ,5 then its direction cosines are :
1 3 5 (b )
(a) , , 1 1 5
√ 35 √ 35 √ 35 , ,
9 3 9

(c) (d) इनमें से कोई नहीं/ None of these


5 3 1
, ,
√ 35 √ 35 √ 35
16. 2 2 1
दो स्वतंत्र घटनाओं A तथा B के लिए यदि P( A∪B)= , P( A )= हो तोP(B) का मान होगा
3 5
2 2
: For two independent events A & B P( A∪B)= , P( A)= , then P(B) is equal
3 5
to:

5 4
(a) (b)
9 9

2 3
(c) (d)
9 9

17. 1
व्यवरोधों x≤20 , y≥10 तथा x , y ≥0 , के अंतर्गत Z=x+y का न्यूनतम मान है :
The minimum value of Z=x+y subjet to the constraints x≤20 , y≥10 and
x , y≥0 , is :

(a) 0 (b ) 10

(c) 20 (d) 30

18. एक रै खिक प्रोग्रामन समस्या के उद्दे श्य फलन Z=11x+7y तथा सस


ु ंगत क्षेत्र के शीर्ष बिन्द ु
(3 ,2), (0, 5), (0 ,3) हैं तो Z का नूनतम मान होगा जिस बिन्द ु पर वह है : 1

If the objective function for the LPP is Z=11x+7y and the corner points of the
bounded feasible regions are (3 ,2), (0, 5), (0 ,3)then the minimum value of Z occurs
at :
(a) (3, 2) (b ) (0, 5)

(c) (0, 3) (d) does not exist


अभिकथन-कारण प्रश्न (ASSERTION-REASON BASED QUESTIONS )

निम्नलिखित प्रश्नों मे अभिकथन (A) के बाद कारण ( R ) दिया गया है । निम्नलिखित विकल्पों में
से सही विकल्प का चन ु ाव कीजिए ।
(a) A तथा R दोनों सत्य हैं और R , A की सही व्याख्या है ।
(b) A तथा R दोनों सत्य हैं परं तु R , A की सही व्याख्या नहीं है ।

(c) A सत्य है परं तु R असत्य है ।


(d)A असत्य है परं तु R सत्य है ।
In the following questions, a statement of assertion (A) is followed by a statement of
Reason (R). Choose the correct answer out of the following choices.

(a) Both A and R are true and R is the correct explanation of A.

(b) Both A and R are true but R is not the correct explanation of A.

(c) A is true but R is false.

(d) A is false but R is true.

19. 7π 5π 1
अभिकथन/Assertion( A) : c cos−1 (cos( ))=
6 6

कारण /Reason ( R) : cos−1 (cos x)=x for all x  (10 , π )

20. अभिकथन/Assertion( A) :If a line makes angles α , β , γ with the positive direction of
coordinate axes then cos 2 α +cos 2 β +cos 2 γ =−1 1

कारण /Reason ( R) :) : Sum of squares of direction cosines of a line is 1

खंड बी ( Section B)
इस खं ड में पाँ च अति लघ ु उत्त र वाले प्र श्न हैं प्र त्ये क प्र श्न 2 अं क का है ।
This section contains 5 Very Short Answer (VSA)-type questions of 2 marks each.
21. 2
एक प्रतिलोम त्रिकोणमितीय फलन f(x) का आलेख नीचे दिया गया है । आलेख का अवलोकन
कीजिए और निम्नलिखित प्रश्नों के उत्तर दीजिए।
−1
(i) f ( ) का मान क्या है ?
2
(ii)यदि f (x )= π ,हो तो x. का मान ज्ञात कीजिए ।
4

y=f(x)

The graph of an inverse trigonometric function f(x) is given below, observe the graph and
answer the following questions
−1
(i)What is the value of f ( ) ?
2
(ii)If f (x )= π , then find the value of x.
4
22.
दिखाईए कि f मेँ फलन A=R− {23 } जो f (x)= 64 x−4
x +3
द्वारा परिभाषित है एकेकी है :
2

A=R−{ } defined as f (x )=
2 4 x+3
Show that the function f in is one- one
3 6 x−4

23. 2
dy
यदि /If y=x y
,तब मान ज्ञात कीजिए/then find
dx
अथवा / OR
यदि /If
)+tan −1 ( √
2
1 1+ x −1 dy
y=sin−1 ( ) ज्ञात कीजिए / find
√ 1+x 2 x dx

24. 2
एक कण वक्र x 2 =2 y के साथ चलता है । किस बिंद ु पर, भुज के बढ़ने पर कोटि उसी दर से बढ़ती
है ?
A particle moves along the curve x 2 =2 y . At what point , ordinate increases at the same
rate as abscissa increases ?
25. logx 2
ज्ञात कीजिए /Find ∫ dx
(1+logx)2
अथवा / OR
मान ज्ञात कीजिए /Find the value of
1

∫ tan−1( 1+1−2
x−x
x
2
)dx
0
खंड सी / Section C
इस खं ड में छह : लघ ु उत्त र वाले प्र श्न हैं प्र त्ये क प्र श्न 3 अं क का है । This section
contains 6 Short Answer (SA)-type questions of 3marks each.
2
26 . d y 3
यदि /If x=a sin 2 θ , y=a cos2 θ , तब ज्ञात कीजिए /then find 2
dx
27. एक बैग A में 4 काली गें दें और 6 लाल गें दें हैं और बैग B में 7 काली और 3 लाल गें दें हैं। एक पासा 3
फेंका जाता है । यदि उस पर 1 या 2 दिखाई दे ते हैं, तो बैग A को चुना जाता है , अन्यथा बैग B को
चुना जाता है । यदि चयनित बैग से दो गें दों को यादृच्छिक रूप से (प्रतिस्थापन के बिना) निकाला
जाता है , तो उनमें से एक के लाल और दस ू रे के काले होने की प्रायिकता ज्ञात कीजिए।
अथवा
15 बल्बों के ढे र में से जिसमें 5 खराब हैं, दो बल्बों का एक नमूना यादृच्छिक रूप से (बिना
प्रतिस्थापन के) निकाला जाता है । खराब बल्बों की संख्या का प्रायिकता वितरण ज्ञात कीजिए।
A bag A contains 4 black balls and 6 red balls and bag B contains 7 black and 3 red balls. A
die is thrown . If 1 or 2 appear on it , then bag A is chosen , otherwise bag B . If two balls
are drawn at random (Without replacement ) from the selected bag , find the probability
of one of them being red and another black.
OR
From a lot of 15 bulbs which include 5 defectives , a sample of two bulbs is drawn at
random (without replacement).Find the probability distribution of the number of defective
bulbs.
π
28. 3 3
हल कीजिए /Solve ∫π 1+ √dxcotx
6

अथवा /OR
5

हल कीजिए /Solve ∫|x +2|dx


−5
π
4 3
ज्ञात कीजिए// Evaluate ∫ log (1+tan x)dx
0
OR
Find ∫ e . sinx dx
x

29. समीकरण (1+ x2 )dy+2 xy dx=cotx dx का सामान्य हल ज्ञात कीजिए । 3


Find the general solution of (1+ x2 )dy+2 xy dx=cotx dx
अथवा / OR

निम्न अवकल समीकरण को हल कीजिए /Solve following differential equation


2 2
(x − y ) dx+2 xy dy=0

30. व्ययवरोधों x+ y≤50,3 x+ y≤90 , x≥0 , y ≥0 के अंतर्गत Z=4x+y का अधिकतमिकरण 3


कीजिए । रे खिक प्रोग्रामन समस्या को ग्राफ विधि से हल कीजिए ।

Solve the following Linear programming problem graphically :


Maximize : Z=4x+y
subject to the constraints x+ y≤50,3 x+ y≤90 , x≥0 , y≥0

31. Find the interval in which function f (x)=2 x 3 −9 x2 +12 x+15 is strictly increasing and 3
strictly decreasing
खंड डी /(SECTION D )
इस खं ड में चार दीरध उत्त र वाले प्र श्न हैं । प्र त्ये क प्र श्न 5 अं क का है ।
This section contains four Long Answer (LA)-type questions of 5marks each.
32. वक्र 4 y=3 x 2 और रे खा 2y=3x+12 के बीच शामिल क्षेत्र का क्षेत्रफल ज्ञात कीजिए। 5
Find the area of the region included between the curves 4 y=3 x 2 and the line
2y=3x+12

33. यदि N सभी प्राकृत संख्याओं का समुच्चय है तथा R, NXN पर तुल्यता संबंध है जो 5
(a , b)∈R (c , d)
द्वारा परिभाषित है । यदि ad(b+c)=bc(a+d) हो तो सिद्ध कीजिए कि R एक तल्
ु यता संबध
ं है ।

Let N denote the set of all natural numbers and R be the relation on N x N defined by
(a , b)∈R (c , d) If ad(b+c)=bc(a+d). Prove that R is an equivalence relation.

34. निम्नलिखित दी गई रे खाओं ⃗r =(8+3 λ ) ^i−(9+16 λ ) ^j+(10+7 λ ) k^ तथा 5


^ μ (3 ^i +8 ^j−5 k^ ) के बीच की लघुत्तम दरू ी ज्ञात कीजिए।
⃗r =15 ^i +29 ^j+5 k+

Find the shortest distance between the lines given by


⃗r =(8+3 λ ) ^i−(9+16 λ ) ^j+(10+7 λ ) k^ and ⃗r =15 ^i +29 ^j+5 k^ + μ (3 ^i+8 ^j−5 k^ )

अथवा /OR
उस रे खा की सदिश तथा कार्तिये समीकरण ज्ञात कीजिए जो बिन्द ु (1, 1, 1) से गज ु रती हो तथा
x−1 y−2 z−3 x−1 y−2 z−3
रे खाओं = = तथा = = पर लम्बवत हो । दोनों रे खाओं के
2 3 4 2 3 4
बीच का कोण भी ज्ञात कीजिए।
Find the vector and cartesian equations of the line which is perpendicular to the lines with
equations and and passes through the point (1, 1, 1). Also find the angle between the given
lines.
35. गुणनफल AB का मान ज्ञात कीजिए, जहाँ : 5

Evaluate the product AB , where :

[ ] [ ]
1 −1 0 2 2 −4
A= 2 3 4 and B= −4 2 −4 अतः रै खिक समीकरण निकाये को हल कीजिए /
0 1 2 2 −1 5
Hence solve the system of linear equations

x-y=3
2x+3y+4z=17

y+2z=7
अथवा / OR

[ ]
2 0 1
यदि A= 2 1 3 हो तो A 2 −5 A +4 I ज्ञात कीजिए तथा आव्यहू X इस प्रकार ज्ञात
1 −1 0
कीजिए ताकि A −5 A +4 I + X=0 हो।
2

[ ]
2 0 1
If A= 2 1 3 , find A 2 −5 A +4 I and hence find a matrix X such that
1 −1 0
2
A −5 A +4 I + X=0
खंड ई (Section E)
स्रोत आधारित /गद्यांश /क्षमता प्रकरण अध्ययन / एसेसमें ट की एकीकृत इकाई प्रश्न
Source based/Case based/passage based/integrated units of assessment Questions

36. 1+1+2
x−6 2− y z−2
लाइन = = के किनारे सड़क पर एक बाइक दौड़ रही है
1 2 2
x+4 y z +1
जबकि एक हवाई जहाज़ लाइन = = के साथ अंतरिक्ष में उड़ रहा है
3 −2 −2
ऊपर दी गई जानकारी के आधार पर निम्नलिखित प्रश्नों के उत्तर दीजिए।
1(i) दोनों रे खाओं के समीकरण सदिश रूप में लिखिए।
(II) दी गई रे खाओं पर लंबवत एक सदिश ज्ञात कीजिए ।
(iii) दोनों विषमतलीय रे खाओं के बीच की न्यन ू तम दरू ी ज्ञात कीजिए।
अथवा
x−6 2− y z−2
(IIIi) λ के किस मान के लिए रे खाएं = = तथा
1 2 2
x+4 y z +1
= =
3 −2 −2
एक दस ू को काटती हैं?
रे
x−6 2− y z−2
A bike is running on the road along the line = =
1 2 2
x+4 y z +1
while an aeroplane is flying in the space along the line = =
3 −2 −2

Based on the information given above answer the following questions .


1(i) Write the equations of both the lines in vector form.
(II) Find a vector perpendicular to both the given lines .
(iii) Find shortest distance between both skew lines.
OR
x−6 2− y z−2 x+4 y z +1
(IIi) For which value of λ the lines = = and = =
1 2 2 3 −2 −2
Intersect each other

37.
स्मार्ट सिटी इंदौर में 100 घरों वाली एक आवासीय सोसायटी में 10-15 साल की उम्र के बीच के 1+1+2
60 बच्चे थे। उन्हें उनके शिक्षक ने बायोडिग्रेडब
े ल कचरे का पुनर्चक्रण सुनिश्चित करने के लिए
खाद बनाना शुरू करने के लिए प्रेरित किया था। इस उद्दे श्य के लिए प्रत्येक बच्चे को केवल अपने
घर के लिए ऐसा करने के बजाय रे जिडेंट्स वेलफेयर एसोसिएशन को एक सामाजिक पहल के रूप
में ऐसा करने के लिए राजी किया। इसके लिए उन्होंने एक स्थानीय पार्क में एक वर्गाकार क्षेत्र की
पहचान की। स्थानीय अधिकारियों ने जगह के लिए ₹50 प्रति वर्ग मीटर की दर से शुल्क लिया
ताकि जगह का दरु ु पयोग न हो और रे जिडेंट वेलफेयर एसोसिएशन इसे गंभीरता से लेता है ।
एसोसिएशन ने खद ु ाई के लिए एक मजदरू को 250 मीटर रुपये पर काम पर रखा और उसने
3

₹400 x या x (गहराई )2 शुल्क लिया। ( एसोसिएशन प्रति वर्ग मीटर न्यूनतम लागत रखना
चाहे गी।
ऊपर दी गई जानकारी के आधार पर निम्नलिखित का उत्तर दें :

(i)यदि वर्गाकार भूखंड की भज ु ा x मीटर है और गहराई h मीटर है तो गड्ढे की लागत C ज्ञात


कीजिये
dc
.(II) h का मान (मीटर में )जिसमे का मान ज्ञात कीजिए।ताकि =0
dh
2
d c
(iii) 2
का मूल्य क्या है ?
dh
अथवा
(iii)न्यूनतम लागत के लिए x का मान (मीटर में )ज्ञात कीजिए।

In a smart city Indore a residential society comprising of 100 houses, there were 60
childrens between the ages 10-15 yearsThey were inspired by their teacher to start
composting to ensure that biodegradable waste is recyled. For this purpose instead of
each child doing it for only his/her house childrens convinced the Residents welfare
association to do it as a society initiative. For this they identified a square area ina local
park . Local authorities charged amount of ₹50 per sq metre for space so that there is no
misuse of the space and Resident welfare association takes it seriously . Association hired
a labourer for digging out 250 m3 and he charged ₹400 x or X (depth)2 . Association
will like to have minimum cost .

Based on the information given above answer the following:


(i)If the side of square plot is x metre and its depth is h metre then find the cost C for the
pit
.
dc
(II) Find the value of h(in metre)for which =0 .
dh
d2 c
(iii) What is the value of ?
d h2
OR
(iii)Find the value of x)( in metre)for minimum cost.

38. रूस और यूक्रेन के बीच युद्ध में ब्रिटे न ने यूक्रेन को A और B नामक दो प्रकार की नई विमान भेदी 2+2
बंदक
ू ें प्रदान कीं जिनका उपयोग यक्रू े नी सेना ने रूस के 'आत्मघाती ड्रोन' को रोकने के लिए किया
था। उनसे दागे गए गोले के हवाई जहाज़ से टकराने की संभावनाएँ क्रमशः 0.3 और 0.2 हैं। दोनों
ने एक ही समय में एक हवाई जहाज पर एक-एक गोला दागा।
ऊपर दी गई जानकारी के आधार पर निम्नलिखित प्रश्नों के उत्तर दीजिए।
(i) इसकी क्या संभावना है कि उनमें से ठीक एक से निकला गोला विमान से टकराया?
(ii) यदि यह ज्ञात है कि उनमें से ठीक एक से दागा गया गोला विमान से टकराया है , तो इसकी
क्या संभावना है कि वह B से दागा गया था?

In a war between Russia and Ukraine , UK provided Ukraine two types of new anti-aircraft
guns named A and B which were used by Ukrainian forces to stop Russia's 'suicide
drones' . The probabilities that the shell fired from them hits an airplane are 0.3 and 0.2
respectively. Both of them fired one shell at an airplane at the same time.

Based on the information given above answer the following questions.


(i) What is the probability that the shell fired from exactly one of them hit the plane?
(ii) If it is known that the shell fired from exactly one of them hit the plane, then what is
the probability that it was fired from B?
शिक्षा निदे शालय, राष्ट्रीय राजधानी क्षेत्र दिल्ली
Directorate of Education, GNCT of Delhi
Annual Examination Practice Paper
(2022-23)
कक्षा – XII
Class – XII
गणित (कोड: 041)
Mathematics (Code: 041)

समय: 3 घंटे अधिकतम अंक: 80


Time: 3 hours Maximum Marks: 80

सामान्य निर्देश:

1. इस प्रश्न पत्र में पाँच खंड हैं - ए ,बी ,सी ,डी तथा ई ,प्रत्येक खंड अनिवार्य है । यद्यपि कुछ प्रश्नों मे
आंतरिक विकल्प दिया है ।
2. भाग ए में 18 प्रश्न ,बहुविकल्पीय तथा 2 अभिकथन-कारण प्रश्न प्रत्येक 1 अंक का है (20 अंक)
3. भाग बी में 5 अति लघु उत्तर (VSA) प्रकार के प्रश्न हैं जिनमें प्रत्येक के 2 अंक हैं (10 अंक )
4. भाग सी में 6 लघु उत्तर (SA) प्रकार के प्रश्न हैं जिनमें प्रत्येक के 3 अंक हैं (18 अंक )
5. भाग डी में 4 में दीर्घ उत्तर (LA)प्रकार के प्रश्न हैं जिनमें प्रत्येक के 5 अंक हैं (20 अंक )
5. भाग ई में 3 स्रोत आधारित /गद्यांश /क्षमता प्रकरण अध्ययन / एसेसमें ट की एकीकृत इकाई
पर आधारित प्रश्न जिसमें प्रत्येक प्रश्न (उपभाग सहित) 4 अंक का है (12 अंक)

General Instructions :

1. This Question paper contains - five sections A,B,C,D,E. Each section is compulsory.
However, there are internal choices in some questions.
2. Section A has 18 MCQ’s and 02 Assertion-Reason based questions of 1 mark each.(20
Marks )
3. Section Bhas 5 Very Short Answer (VSA)-type questions of 2 marks each.(10 Marks )
4. Section C has 6 Short Answer (SA)-type questions of 3 marks each.(18 Marks )
5. Section D has 4 Long Answer (LA)-type questions of 5 marks each.(20 Marks )
6. Section E has 3 Source based/Case based/passage based/integrated units of assessment
(4 marks each) with sub parts.(12 Marks )

खंड - ए
Section – A
प्रश्न संख्या 1-18 ,बहुविकल्पीय प्रकार के प्रश्न हैं ।प्रत्येक प्रश्न 1 अंक का है ।
Question Number 1-18 are of MCQ type question one mark each.
Q1.
If 2[ ][ ][ ]
1 3 + y 0 = 5 6
0 x 1 2 1 8
तब x +y बराबर होगा /then x+y equals
1

(a) 6 (b ) 12

(c) 9 (d) 18

[ ]
Q2 0 −1 3 x 1
यदि आव्यह
ू 1 y −5 एक विषम सममित आव्यह
ू है , तब :
−6 5 0
[ ]
0 −1 3 x
If the matrix 1 y −5 is a skew symmetric , then :
−6 5 0

(a) x=-2 , y=0 (b ) x=2 , y=0

(c) x=-1 , y=1 (d) x=2 , y= -1

Q3 यदि आव्यूह /matrix A =[ aij ]2 x 2 जहां where a ij =1 यदि /if i ≠ j तथा /and 0 if i= j then A 2 1
बराबर होगा /is equal to :
(a) I (b ) A

(c) 0 (d) None of these

Q4
यदि /If A=
a b
b a [ ]
और /and A =
2 α β
β α
, तब /then [ ] 1

(a) α =a2 +b 2 , β=ab (b ) α =a2 +b 2 , β=2 ab

(c) α =a2 +b 2 , β=a 2 − b 2 (d) α =2ab , β=a 2 +b2

Q5. यदि A ,3x3 कोटि का एक वर्ग आव्यूह इस प्रकार है कि |A|=5 तो |2A’| का मान है - 1
If A is any square matrix of order 3x3 such that|A|=5, then the value of |2A’| is :

(a) -10 (b ) 10

(c) -40 (d) 40

{
Q6 5x−4 0< x ≤ 1 1
यदि फलन f ( x )= 2 सतत हो सभी x ∈ ( 0,2 ) तो a का मान होगा :
4 x +3 ax 1< x <2

If f ( x )= { 5x−4
2
0< x ≤ 1
4 x +3 ax 1< x <2
is continuous for all x ∈ ( 0,2 ) then the value of a is

(a) 1 (b ) 0

(c) -1 (d) 1/3

x के सापेक्ष sec ( tan x )का अवकलन होगा । 1


−1
Q7

Derivative of sec ( tan x ) w . r .t . x is


−1

x 1
(a) (b )
1+x
2
√ 1+ x 2
x (d) x √ 1+ x 2
(c)
√ 1+ x 2

3
Q8 x 1
∫ x+1 dx बराबर है /is equal to :
2 3 2 3
x x x x
(a) x+ + − log|1− x|+C (b ) x+ − − log|1− x|+C
2 3 2 3

2 3 2 3
x x x x
(c) x − − − log|1+x|+C (d) x − + − log|1+ x|+C
2 3 2 3
π
4 1
∫ tanx dx बराबर है/is equal to :
Q9 0

1 1
(a) log 2 (b) log 2
3 2

(c) log 3 (d) 2 log 2

[ ( )]
Q10 2 3 2
dy d y
अवकल समीकरण 1+ की घात है :
2
=
dx dx2

[ ( )]
2 3
dy 2 d2 y
The degree of the differential equation 1+ = is
dx dx2
(a) 4 3
(b)
2

(c) परिभाषित नहीं /Not defined (d) 2

Q11 1
dy
अवकल समीकरण ( 1− y ) + yx=ay ( − 1< y <1 ) का समाकलन गण ु क होगा । :
2
dx
2 dy
The integrating factor of the differential Equation ( 1− y ) + yx=ay ( − 1< y <1 ) is :
dx
1 1
(a) 2 (b ) 2
y −1 √ y −1
−1 1
(c) (d)
√ 1− y 2
√ 1− y 2

Q12 सदिश 2 i+3


^ ^j − 6 k^ का प्रक्षेप बिंदओ
ु ं (3 ,4 2 ) तथा (5 ,6 3 ) को मिलाने वाली रे खा पर होगा : 1

^ ^j− 6 k^ on the line joining the points (3 ,4 2 ) and


The projection of the vectors 2 i+3
(5 ,6 3 ) is :

2 4
(a) (b )
3 3

−4 5
(c) (d)
3 3

Q13 ^ दो इकाई सदिश हैं तथा θ उनके बीच का कोण है तो |a^ − b^| होगा ।
यदि a^ तथा b 1
^ be two unit vectors and θ is the angle between them , then |a^ − b^| :
If a^ and b

θ θ
(a) sin (b ) 2 sin
2 2

θ θ
(c) cos (d) 2 cos
2 2

Q14 यदि =|⃗a + ⃗b|=|⃗a − ⃗b| तब ⃗a तथा ⃗b के बीच का कोण होगा : 1


If |⃗a + ⃗b|=|⃗a − ⃗b|then the angle between ⃗a and ⃗b is :

π (b )0
(a)
2
π π
(c) (d)
4 6

Q15 किसी रे खा के दिक् - कोसाइन 1 ,3 ,5 हैं तो इसके दिक् - अनुपात होंगे : 1


The direction ratios of line are 1 ,3 ,5 then its direction cosines are :
1 3 5 1 1 5
(a) , , (b ) , ,
√ 35 √ 35 √ 35 9 3 9

5 3 1 (d) None of these


(c) , ,
√ 35 √ 35 √ 35
Q16 एक उद्दे श्य फलन Z=5x+10y का अधिकत्तम मान 1
x+2 y ≤120 , x + y ≥ 60 , x − 2 y ≥ 0 , x ≥ 0 , y ≥ 0 प्रतिबंध पर होगा :
The maximum value of the object function Z=5x+10y subject to the constraints
x +2 y ≤120 , x+ y ≥ 60 , x − 2 y ≥ 0 , x ≥ 0 , y ≥ 0 is:
(a) 300 (b ) 600

(c) 400 (d) 800

Q17 3 4 1
यदि A और B दो स्वतंत्र घटनाएँ हैंजिनके और P ( A )= ,तथा P ( B )= तो P ( A ∩ B )बराबर है:
' '
5 9
3 4
If A and B are two independent events with P ( A )= and P ( B )= ,then P ( A ∩ B )
' '
5 9
equals :
4 8
(a) (b )
15 15

1 2
(c) (d)
3 9

Q18 यदि x + y ≤ 2 ,x , y ≥ 0 ,वह बिदं ु जिस पर 3x+2y का अधिकतम मान प्राप्त हुआ हो, होगा : 1
If x + y ≤ 2 ,x , y ≥ 0 ,the point at which maximum value of 3x+2y attained , will be :
(a) (0, 2) (b ) (0, 0)

(c) (2, 0)
(d ( 12 , 12 )

अभिकथन-कारण प्रश्न (ASSERTION-REASON BASED QUESTIONS )

निम्नलिखित प्रश्नों मे अभिकथन (A) के बाद कारण ( R ) दिया गया है । निम्नलिखित विकल्पों में
से सही विकल्प का चुनाव कीजिए ।

(a) A तथा R दोनों सत्य हैं और R , A की सही व्याख्या है ।


(b) A तथा R दोनों सत्य हैं परं तु R , A की सही व्याख्या नहीं है ।

(c) A सत्य है परं तु R असत्य है ।


(d)A असत्य है परं तु R सत्य है ।
In the following questions, a statement of assertion (A) is followed by a statement of
Reason (R). Choose the correct answer out of the following choices.

(a) Both A and R are true and R is the correct explanation of A.

(b) Both A and R are true but R is not the correct explanation of A.

(c) A is true but R is false.


(d) A is false but R is true.

Q19
अभिकथन/Assertion( A) : cos
−1
( ( ))
cos

6
=

6
1

कारण /Reason ( R) : cos −1 ( cos x )=x for all x  ( 10 , π )

Q20 अभिकथन/Assertion( A) : The position vector of a point say P(x ,y z ) is 1


⃗ ^ y ^j+ z k^
OP =r⃗ = x i+

and its magnitude is |⃗r|= √ x 2 + y 2 + z2

कारण /Reason ( R) : If r⃗ = x i+
^ y ^j+ z k^ , then coefficient of i^ , ^j , k^ in r⃗ ie; x ,y ,z are called
direction ratios of vector r⃗

Q19
अभिकथन/Assertion( A) : cos
−1
( ( ))
cos

6
=

6
1

कारण /Reason ( R) : cos −1 ( cos x )=x for all x  ( 10 , π )

खंड बी ( Section B)
इस खं ड में पाँ च अति लघ ु उत्त र वाले प्र श्न हैं प्र त्ये क प्र श्न 2 अं क का है ।
This section contains 5 Very Short Answer (VSA)-type
questions of 2 marks each.

( ) ( ) ( ( )) 2
Q 21 −1 1 −π
निम्न का मान ज्ञात कीजिए/Find the value of tan
−1
+cot −1 + tan −1 sin
√3 √3 2

अथवा /OR

{
n+1, य दि n वि षम हो
तो प्रदर्शित कीजिए कि फलन एकैकी आच्छादी है ?
यदि
f ( n )= n
, य दि nसम हो
2

Let f ( n )= {nn+1−1,, if nis odd


if nis even
Show that f is a bijection.

Q 22 2
एक कण वक्र x 2 =2 y के साथ चलता है । किस बिंद ु पर, भुज के बढ़ने पर कोटि उसी दर से बढ़ती है ?
A particle moves along the curve x 2 =2 y . At what point , ordinate increases at the same rate as
abscissa increases ?
Q 23 2
|⃗a| तथा |⃗b| ज्ञात कीजिए यदि |⃗a|=2|⃗b| और ( ⃗a + ⃗b ) . ( ⃗a − b⃗ )=12 हो ।

a| and |⃗b| if |a⃗|=2|⃗b| and ( ⃗a + ⃗b ) . ( ⃗a − b⃗ )=12


Find |⃗

अथवा / OR
2 x − 5 y+4 6 − z
Equation of a line is = = . Find the directions cosines of a line parallel to this line
4 3 6
.

Q 24
यदि /If y =sin
−1

1+ x (√ )1
2
+tan
−1
x ( √ 1+x 2 −1
) ,ज्ञात कीजिए /Find
dy
dx
.

Q 25 a|and|⃗b|, यदि/if ( ⃗a + ⃗b ) . ( ⃗a − b⃗ )=8 तथा /and |⃗a|=8|⃗b|


ज्ञात कीजिए /Find |⃗ 2
खंड सी / Section C
इस खं ड में छह : लघ ु उत्त र वाले प्र श्न हैं प्र त्ये क प्र श्न 3 अं क का है । This
section contains 6 Short Answer (SA)-type questions of
3marks each.
3

dx
ज्ञात कीजिए /Find ∫
Q26 √ 8+3 x − x 2

Q27 एक बैग A में 4 काली गें दें और 6 लाल गें दें हैं और बैग B में 7 काली और 3 लाल गें दें हैं। एक पासा फेंका 3
जाता है । यदि उस पर 1 या 2 दिखाई दे ते हैं, तो बैग A को चुना जाता है , अन्यथा बैग B को चुना जाता है ।
यदि चयनित बैग से दो गें दों को यादृच्छिक रूप से (प्रतिस्थापन के बिना) निकाला जाता है , तो उनमें से एक
के लाल और दस ू रे के काले होने की प्रायिकता ज्ञात कीजिए।
अथवा
15 बल्बों के ढे र में से जिसमें 5 खराब हैं, दो बल्बों का एक नमन
ू ा यादृच्छिक रूप से (बिना प्रतिस्थापन के)
निकाला जाता है । खराब बल्बों की संख्या का प्रायिकता वितरण ज्ञात कीजिए।
A bag A contains 4 black balls and 6 red balls and bag B contains 7 black and 3 red balls. A die is
thrown . If 1 or 2 appear on it , then bag A is chosen , otherwise bag B . If two balls are drawn at
random (Without replacement ) from the selected bag , find the probability of one of them being
red and another black.
OR
From a lot of 15 bulbs which include 5 defectives , a sample of two bulbs is drawn at random
(without replacement).Find the probability distribution of the number of defective bulbs.

π
Q28 3 3
dx
हल कीजिए /Solve ∫
π 1+ √ cotx
6

अथवा /OR
5

हल कीजिए /Solve ∫|x+2|dx


−5

Q29 Find the general solution of ( 1+ x2 ) dy+2 xy dx=cotx dx 3


OR
Solve following differential equation ( x 2 − y 2 ) dx+2 xy dy=0

Q30 व्ययवरोधों x+2 y ≥100,2 x − y ≤ 0 ,2 x + y ≤ 0 , x ≥ 0 , y ≥ 0के अंतर्गत Z=x+2y का अधिकतमिकरण 3


कीजिए । रे खिक प्रोग्रामन समस्या को ग्राफ विधि से हल कीजिए ।

Maximize : Z=x+2y
Subject to constraints :
x +2 y ≥100,2 x − y ≤ 0 ,2 x+ y ≤ 0 , x ≥ 0 , y ≥ 0
Solve the LPP graphically.

Q. 31 2x 3
ज्ञात कीजिए /Evaluate ∫( dx
x +1 )( x +3 )
2 2

खंड डी /(SECTION D )
इस खं ड में चार दीरध उत्त र वाले प्र श्न हैं । प्र त्ये क प्र श्न 5 अं क का है ।
This section contains four Long Answer (LA)-type questions of 5marks
each.
Q 32 Make a sketch of region {( x , y ) :0 ≤ y ≤ x 2 ,0 ≤ y ≤ x+2 ,0 ≤ x ≤ 3 }and find its area using 5
integration .

क्षेत्र {( x , y ) :0 ≤ y ≤ x ,0 ≤ y ≤ x+2 ,0 ≤ x ≤ 3 }का एक संभावित आकृति खींचिए तथा साथ ही


2

समाकलन विधि द्वारा इसका क्षेत्रफल ज्ञात कीजिए।

Q 33 यदि N सभी प्राकृत संख्याओं का समुच्चय है तथा R, NXN पर तुल्यता संबंध है जो ( a ,b ) ∈ R ( c , d ) 5


द्वारा परिभाषित है । यदि ad(b+c)=bc(a+d) हो तो सिद्ध कीजिए कि R एक तल्
ु यता संबंध है ।

Let N denote the set of all natural numbers and R be the relation on N x N defined by
( a ,b ) ∈ R ( c , d ) If ad(b+c)=bc(a+d). Prove that R is an equivalence relation.

अथवा /OR

मान लीजिए कि R , प्राकृत संख्याओं के समच् ु चय N में निम्नलिखित प्रकार से परिभाषित एक संबंध R={
( x , y ) : x ∈ N , y ∈ N ,2 x+ y=41} है ।
संबंध R का प्रांत तथा परिसर ज्ञात कीजिए । साथ ही सत्यापित (जाँच ) कीजिए कि क्या R स्वतुल्य ,
सममित तथा संक्रामक है ।
Let R be a relation defined on the set of natural numbers N as follows: {
( x , y ) : x ∈ N , y ∈ N ,2 x+ y=41} . Find the domain and range of the relation R . Also verify
whether R is reflexive , symmetric and transitive .

Q 34 निम्नलिखित दी गई रे खाओं ^ ( 9+16 λ ) ^j+ ( 10+7 λ ) k^ तथा


r⃗ = ( 8+3 λ ) i− 5
r⃗ =15 i^ +29 ^j+5 k^ +μ ( 3 i^ +8 ^j − 5 k^ )के बीच की लघुत्तम दरू ी ज्ञात कीजिए।

Find the shortest distance between the lines given by r⃗ = ( 8+3 λ ) i^ − ( 9+16 λ ) ^j+ ( 10+7 λ ) k^ and
^
r⃗ =15 i+29 ^j+5 k^ +μ ( 3 i+8
^ ^j − 5 k^ )

अथवा /OR
उस रे खा की सदिश तथा कार्तिये समीकरण ज्ञात कीजिए जो बिन्द ु (1, 1, 1) से गज
ु रती हो तथा रे खाओं
x −1 y −2 z − 3 x −1 y −2 z − 3
= = तथा = = पर लम्बवत हो । दोनों रे खों के बीच का कोण भी
2 3 4 2 3 4
ज्ञात कीजिए।
Find the vector and cartesian equations of the line which is perpendicular to the lines with
equations and and passes through the point (1, 1, 1). Also find the angle between the given lines.

[ ]
Q35 1 2 0 5
यदि A= − 2 − 1 − 2 , तो A − 1ज्ञात कीजिए । A − 1का प्रयोग करके रै खिक समीकरणों के निकाये x -2y
0 −1 1
=10 , 2x -y - z=8 तथा -2y + z=7 को हल कीजिए।

[ ]
1 2 0
If A= − 2 − 1 − 2 , then find the value of A − 1.
0 −1 1
using A − 1,solve the system of linear equations x-2y=10 , 2x-y-z=8 and -2y+z=7

खंड ई (Section E)
स्रोत आधारित /गद्यांश /क्षमता प्रकरण अध्ययन / एसेसमें ट की एकीकृत इकाई
प्रश्न
Source based/Case based/passage based/integrated units of assessment
Questions
एक कण बहुपद f ( x )= ( x − 1 )( x −2 ) द्वारा निरूपित वक्र के अनुदिश गति कर रहा है जैसा कि नीचे दिए
2
Q36
गए चित्र में दिखाया गया है । 4
उपरोक्त जानकारी के आधार पर निम्नलिखित प्रश्नों के उत्तर दीजिए।
(i)बहुपद f ( x )= ( x − 1 )( x −2 )2
के क्रातिक बिन्द ु ज्ञात कीजिए ।
(ii)वह अंतराल ज्ञात कीजिए जिसमें फलन निरं तर वर्धमान है ।
(iii)वह अंतराल ज्ञात कीजिए जिसमें फलन निरंतर ह्वासमान है ।
फलन f ( x )= ( x − 1 )( x −2 ) का स्थानिये उच्चत्तम मान क्या होगा?
2

2
A particle is moving along the curve represented by the polynomial f ( x )= ( x − 1 )( x −2 ) as shown
in the figure given below.
Based on the above information answer the following questions.
(i)Find Critical points of polynomial f ( x )= ( x − 1 )( x −2 )2. 1
(ii)Find the interval where f(x) is strictly increasing
(iii)Find the interval where f(x) is strictly decreasing. 1
OR 2
What is the point of local maxima of f ( x )= ( x − 1 )( x −2 )2 ?
To Get More material for
Maths IX - X Click below

CBSE MATHS
IX-X

To Get More Material for XI-


XII Click below

CBSE MATHS
XI-XII
Q 37 4

कंपनी का कुल लाभ फलन P ( x )=−5 x +125 x+37500 है , जहां x कंपनी का उत्पादन है ।
2

उपरोक्त जानकारी के आधार पर निम्नलिखित प्रश्नों के उत्तर दीजिए:


(i) जब उत्पादन 2 इकाई हो तो कंपनी को कितना लाभ होगा?
(ii) किस अंतराल में लाभ सख्ती से बढ़ रहा है ?
(iii) अधिकतम लाभ होने पर उत्पादन क्या होगा?
या
(iii) 38250/- का लाभ होने पर कंपनी का उत्पादन क्या होगा?

2
P ( x )=−5 x +125 x+37500 is the total profit function of a company, where x is a production of
the company.
Based on the above information answer the following questions:
(i)When the production is 2 units what will be the profit of the company?
(ii)In which interval profit is strictly increasing?
(iii)What will be the production when profit is maximum?
OR
(iii)What will be the production of the company when profit is Rs 38250/-?
2
P ( x )=−5 x +125 x+37500

Q 38 एक कारखाने में बोल्ट बनाने के लिए तीन मशीनें A, B और C हैं। मशीन A ,30% का निर्माण करती है , 2+2
मशीन B 20% का निर्माण करती है और मशीन C क्रमशः 50% बोल्ट का निर्माण करती है । उनके संबंधित
आउटपुट में से 5%,2% और 4% ख़राब हैं। कुल उत्पादन से यादृच्छिक रूप से एक बोल्ट निकाला जाता है
और यह दोषपूर्ण पाया जाता है
उपरोक्त जानकारी के आधार पर निम्नलिखित प्रश्नों के उत्तर दीजिए।
(i) खराब बोल्ट मशीन A द्वारा निर्मित होने की प्रायिकता ज्ञात कीजिए।
(ii) इसकी क्या प्रायिकता है कि दोषपूर्ण बोल्ट मशीन C द्वारा निर्मित है ?

A factory has three machines A, B, and C to manufacture bolts. Machine A manufacture


30% ,Machine B manufacture 20% and Machine C manufacture 50% of the bolts respectively. Out
of their respective outputs 5%,2%, and 4% are defective . A bolt is drawn at random from total
production and it is found to be defective
Based on the above information answer the following questions.
(i) Find the probability that defective bolt drawn manufactured by machine A.
(ii) What is the probability that defective bolt is manufactured by machine C?

You might also like